You are on page 1of 263

ΧΑΤΖΗΚΥΡΙΑΚΟΥ ΚΩΣΤΑΣ

Π.Τ.Δ.Ε.
Πανεπιστήμιο Θεσσαλίας

Ψυχαγωγικά Μαθηματικά
Από τις μαθηματικές σπαζοκεφαλιές
στις μαθηματικές θεωρίες
Ψυχαγωγικά Μαθηματικά

Συγγραφή
Χατζηκυριάκου Κώστας

Συντελεστές έκδοσης
Γλωσσική Επιμέλεια: Αθηνά Χατζηγεωργίου
Γραφιστική Επιμέλεια: Γιώργος Σκουλούδης
Copyright © 2023, ΚΑΛΛΙΠΟΣ, ΑΝΟΙΚΤΕΣ ΑΚΑΔΗΜΑΪΚΕΣ ΕΚΔΟΣΕΙΣ
(ΣΕΑΒ + ΕΛΚΕ-ΕΜΠ)

Το παρόν έργο αδειοδοτείται υπό τους όρους της άδειας Creative Commons Αναφορά Δημιουργού - Μη Εμπορική
Χρήση - Παρόμοια Διανομή 4.0. Για να δείτε ένα αντίγραφο της άδειας αυτής επισκεφτείτε τον ιστότοπο
https://creativecommons.org/licenses/by-nc-sa/4.0/deed.el

ΚΑΛΛΙΠΟΣ
Εθνικό Μετσόβιο Πολυτεχνείο
Ηρώων Πολυτεχνείου 9, 15780 Ζωγράφου

www.kallipos.gr

ISBN: 978-618-228-126-0

Βιβλιογραφική Αναφορά: Χατζηκυριάκου, Κ. (2023). Ψυχαγωγικά Μαθηματικά [Προπτυχιακό εγχειρίδιο].


Κάλλιπος, Ανοικτές Ακαδημαϊκές Εκδόσεις. http://dx.doi.org/10.57713/kallipos-361
Στη Ρίκα–
Χωρίς αυτήν ίσως να μην γραφόταν αυτό το σύγγραμμα
χωρίς αυτήν καθυστέρησε να ολοκληρωθεί
Πίνακας περιεχομένων

Πίνακας περιεχομένων ..................................................................................................................................... 11


Προλεγόμενα ..................................................................................................................................................... 15
Κεφάλαιο 1 Ποτάμια και περάσματα ............................................................................................................. 17
1.1 Ο πραματευτής, ο λύκος, το πρόβατο και το φορτίο λάχανα.................................................................... 18
1.2 Οι τρεις φίλοι και οι αδελφές τους ............................................................................................................ 19
1.3 Ο παχύς και η παχιά .................................................................................................................................. 20
1.4 Λίγη ιστορία και… ακόμη λιγότερη ανθρωπολογία ................................................................................. 21
1.5 Παραλλαγές και επεκτάσεις ...................................................................................................................... 23
1.6 Αποτίμηση του «προβλήματος της διακομετακομίσεως» ως διδακτικού εργαλείου ............................... 27
Απαντήσεις στα ερωτήματα του 1ου Κεφαλαίου............................................................................................ 29
1.1 Ο πραματευτής, ο λύκος, το πρόβατο και το φορτίο λάχανα.................................................................... 29
1.2 Οι τρεις φίλοι και οι αδελφές τους ............................................................................................................ 31
1.3 Ο παχύς και η παχιά .................................................................................................................................. 32
1.4 Λίγη ιστορία και… ακόμη λιγότερη ανθρωπολογία ................................................................................. 34
1.5 Παραλλαγές και επεκτάσεις ...................................................................................................................... 34
Βιβλιογραφία/Αναφορές .................................................................................................................................. 36
Κεφάλαιο 2 Γέφυρες και διαδρομές ............................................................................................................... 37
2.1 Οι επτά γέφυρες του Κένινγκσμπεργκ ...................................................................................................... 38
2.2 Η μονοκοντυλιά ........................................................................................................................................ 41
2.3 Δωμάτια..................................................................................................................................................... 45
2.4 Χειραψίες και γάμοι .................................................................................................................................. 47
2.5 Γραφήματα ................................................................................................................................................ 50
Απαντήσεις στα ερωτήματα του 2ου Kεφαλαίου............................................................................................ 55
2.1 Οι επτά γέφυρες του Κένινγκσμπεργκ ...................................................................................................... 55
2.2 Η μονοκοντυλιά ........................................................................................................................................ 58
2.3 Δωμάτια..................................................................................................................................................... 60
2.4 Χειραψίες .................................................................................................................................................. 60
2.5 Γραφήματα ................................................................................................................................................ 62
Βιβλιογραφία/Αναφορές .................................................................................................................................. 64
Κεφάλαιο 3 Πύργοι και διαδρομές ................................................................................................................. 65
3.1 Οι είκοσι στάσεις (The Icosian Game)...................................................................................................... 66
3.2 Τα πλατωνικά γραφήματα ......................................................................................................................... 67
3.3 Οι πύργοι του Ανόι.................................................................................................................................... 68
3.4 To πυργάκι που τρελαίνει (Instant Insanity Puzzle) ................................................................................. 69
Π ................................................................................................................................................................. 70
3.5 Γραφήματα ................................................................................................................................................ 71
Απαντήσεις στα ερωτήματα του 3ου Kεφαλαίου............................................................................................ 75
3.1 Οι είκοσι στάσεις (The Icosian Game)...................................................................................................... 75
3.2 Τα πλατωνικά γραφήματα ......................................................................................................................... 77
3.3 Οι πύργοι του Ανόι.................................................................................................................................... 78
3.4 To πυργάκι που τρελαίνει ......................................................................................................................... 80
3.5 Γραφήματα ................................................................................................................................................ 82
Βιβλιογραφία/Αναφορές .................................................................................................................................. 84
Κεφάλαιο 4 Δέντρα και δάση .......................................................................................................................... 85
4.1 Τυχαίο; ...................................................................................................................................................... 86
4.2 Φρεσκοφυτεμένα δένδρα και δάση ........................................................................................................... 87
4.3 Οι πυροσβεστικοί κρουνοί ........................................................................................................................ 88
4.4 Χρωματισμοί (κορυφών)........................................................................................................................... 89
4.5 Και λίγη... Χημεία: Αλκάνια ..................................................................................................................... 90
4.6 Γραφήματα ................................................................................................................................................ 92
Απαντήσεις στα ερωτήματα του 4ου Kεφαλαίου ............................................................................................ 93
4.1 Τυχαίο; ...................................................................................................................................................... 93
4.2 Φρεσκοφυτεμένα δένδρα και δάση ........................................................................................................... 94
4.3 Οι πυροσβεστικοί κρουνοί ........................................................................................................................ 95
4.4 Χρωματισμοί (κορυφών)........................................................................................................................... 97
4.5 Και λίγη… Χημεία: Αλκάνια .................................................................................................................... 99
4.6 Γραφήματα .............................................................................................................................................. 100
Βιβλιογραφία/Αναφορές ................................................................................................................................ 104
Κεφάλαιο 5 Ξεμπλέκουν οι ακμές τους; ....................................................................................................... 105
5.1 Γείτονες που αντιπαθιούνται ................................................................................................................... 106
5.2 Από τον Πλάτωνα στον Όυλερ και πίσω ................................................................................................ 107
5.3 To K3,3, το Κ5 και όλα τα μη επίπεδα γραφήματα ................................................................................... 112
Απαντήσεις στα ερωτήματα του 5ου Kεφαλαίου.......................................................................................... 115
5.1 Γείτονες που αντιπαθιούνται ................................................................................................................... 115
5.2 Από τον Πλάτωνα στον Όυλερ και πίσω ................................................................................................ 115
5.3 To K3,3, το Κ5 και όλα τα μη επίπεδα γραφήματα .................................................................................. 117
Βιβλιογραφία/Αναφορές ................................................................................................................................ 120
Κεφάλαιο 6 Χρωματισμοί .............................................................................................................................. 121
6.1 Χρωματίζοντας χάρτες ............................................................................................................................ 122
6.2 Χρωματίζοντας τα επίπεδα πλατωνικά γραφήματα ................................................................................ 124
6.3 Χρωματισμοί και χρωματικοί αριθμοί .................................................................................................... 127
6.4 Χρωματίζοντας ακμές ............................................................................................................................. 129
Απαντήσεις στα ερωτήματα του 6ου Kεφαλαίου.......................................................................................... 133
6.1 Χρωματίζοντας χάρτες ............................................................................................................................ 133
6.2 Χρωματίζοντας τα επίπεδα πλατωνικά γραφήματα ................................................................................ 140
6.3 Χρωματισμοί και χρωματικοί αριθμοί .................................................................................................... 142
6.4 Χρωματίζοντας ακμές ............................................................................................................................. 143
Βιβλιογραφία/Αναφορές ................................................................................................................................ 149
Κεφάλαιο 7 Γοητευτικά τετράγωνα, Ι .......................................................................................................... 151
7.1 Μαγικά τετράγωνα (περιττής τάξης) ...................................................................................................... 152
7.2 Μαγικά τετράγωνα (άρτιας τάξης).......................................................................................................... 158
Απαντήσεις στα ερωτήματα του 7ου Κεφαλαίου.......................................................................................... 161
7.1 Μαγικά τετράγωνα (περιττής τάξης) ...................................................................................................... 161
7.1 Μαγικά τετράγωνα (άρτιας τάξης).......................................................................................................... 166
Βιβλιογραφία/Αναφορές ................................................................................................................................ 167
Κεφάλαιο 8 Γοητευτικά τετράγωνα, ΙΙ ........................................................................................................ 169
8.1 Λατινικά τετράγωνα ................................................................................................................................ 170
8.2 Οι σπαζοκεφαλιές Sudokou και Kenken ................................................................................................. 175
Απαντήσεις στα ερωτήματα του 8ου Κεφαλαίου.......................................................................................... 179
8.1 Λατινικά τετράγωνα ................................................................................................................................ 179
8.2 Οι σπαζοκεφαλιές Sudokou και Kenken ................................................................................................. 185
Βιβλιογραφία/Αναφορές ................................................................................................................................ 188
Κεφάλαιο 9 Συμμετρίες, ομάδες, συγγένειες................................................................................................ 189
9.1 Οι συμμετρίες του τετραγώνου ............................................................................................................... 190
9.2 Οι συμμετρίες του ισόπλευρου τριγώνου................................................................................................ 192
9.3 Συγγένειες ............................................................................................................................................... 194
Απαντήσεις στα ερωτήματα του 9ου Κεφαλαίου.......................................................................................... 197
9.1 Οι συμμετρίες του τετραγώνου ............................................................................................................... 197
9.3 Συγγένειες ............................................................................................................................................... 199
Βιβλιογραφία/Αναφορές ................................................................................................................................ 201
Κεφάλαιο 10 Το ρολόϊ, η σπαζοκεφαλιά 14-15, Κωδωνοκρουσίες ............................................................ 203
10.1 Το ρολόι ................................................................................................................................................ 204
10.2 Ομάδες μικρής τάξης ............................................................................................................................ 205
10.3 H σπαζοκεφαλιά 14-15 ......................................................................................................................... 210
10.4 Κωδωνοκρουσίες .................................................................................................................................. 211
10.5 Ομάδες με άπειρα στοιχεία ................................................................................................................... 213
Απαντήσεις στα ερωτήματα του 10ου Κεφαλαίου........................................................................................ 215
10.1 To ρολόι ................................................................................................................................................ 215
10.2 Ομάδες μικρής τάξης ............................................................................................................................ 216
10.3 H σπαζοκεφαλιά 14-15 ......................................................................................................................... 219
10.4 Κωδωνοκρουσίες .................................................................................................................................. 222
10.5 Ομάδες με άπειρα στοιχεία ................................................................................................................... 225
Βιβλιογραφία/Αναφορές ................................................................................................................................ 227
Κεφάλαιο 11 Νούμερα.................................................................................................................................... 229
11.1 Ζυγίσματα ............................................................................................................................................. 230
11.2 Μεταγγίσεις ........................................................................................................................................... 234
11.3 Μάντεψε!............................................................................................................................................... 236
11.4 Κρυπτάριθμοι ........................................................................................................................................ 237
Απαντήσεις στα ερωτήματα του 11ου Κεφαλαίου........................................................................................ 239
11.1 Ζυγίσματα ............................................................................................................................................. 239
11.2 Μεταγγίσεις ........................................................................................................................................... 240
11.3 Μάντεψε!............................................................................................................................................... 240
11.4 Κρυπτάριθμοι ........................................................................................................................................ 240
Βιβλιογραφία/Αναφορές ................................................................................................................................ 242
Κεφάλαιο 12 Παιχνίδια τύχης ....................................................................................................................... 243
12.1 Ζαριές .................................................................................................................................................... 244
12.2 Χαρτοπαίγνια ........................................................................................................................................ 248
12.3 Γενέθλια ................................................................................................................................................ 250
12.4 Από την ψυχαγωγία στην κριτική μαθηματική αγωγή.......................................................................... 251
Απαντήσεις στα ερωτήματα του 12ου Κεφαλαίου........................................................................................ 253
12.1 Ζαριές .................................................................................................................................................... 253
12.2 Χαρτοπαίγνια ........................................................................................................................................ 258
`12.3 Γενέθλια ............................................................................................................................................... 259
12.4 Από την ψυχαγωγία στην κριτική μαθηματική αγωγή.......................................................................... 259
Βιβλιογραφία/Αναφορές ................................................................................................................................ 261
Προλεγόμενα

Με το παρόν σύγγραμμα επιχειρώ να αξιοποιήσω την εμπειρία που αποκόμισα διδάσκοντας το μάθημα
Διασκεδαστικά Μαθηματικά και Επίλυση Προβλημάτων επί δύο δεκαετίες, στο Παιδαγωγικό Τμήμα
Δημοτικής Εκπαίδευσης του Πανεπιστημίου Θεσσαλίας.

Η κεντρική ιδέα του συγγράμματος περιγράφεται από τον υπότιτλό του: Από τις μαθηματικές
σπαζοκεφαλιές στις μαθηματικές θεωρίες. Σκοπός μου είναι να δείξω πώς ορισμένες σπαζοκεφαλιές ή
ορισμένα προβλήματα παιγνιώδους χαρακτήρα που έχουν απλή διατύπωση όντως οδήγησαν ή μπορούν να
οδηγήσουν σε όμορφες, ενδιαφέρουσες, βαθιές και χρήσιμες μαθηματικές θεωρίες, όπως είναι,
παραδείγματος χάριν, η θεωρία γραφημάτων, η θεωρία των πεπερασμένων ομάδων, η συνδυαστική, η θεωρία
των πιθανοτήτων και η θεωρία των αριθμών. Πρόκειται, δηλαδή, για βασικές θεωρίες των σύγχρονων
Μαθηματικών που συχνά ταξινομούνται στην ευρύτερη περιοχή των Διακριτών Μαθηματικών.

Στον πυρήνα του συγγράμματος βρίσκονται δεκαέξι «πακέτα φύλλων εργασίας», τα οποία
χρησιμοποιούσα ως διδακτικό υλικό στο μάθημα Διασκεδαστικά Μαθηματικά και Επίλυση Προβλημάτων. Τα
πακέτα αυτά μετεξελίχθηκαν στα δώδεκα κεφάλαια του συγγράμματος.

Ειδικότερα, στο πρώτο κεφάλαιο, Ποτάμια και Περάσματα, μέσω της οριζόντιας μαθηματικοποίησης
εισάγεται το χρήσιμο μαθηματικό μοντέλο του απλού γραφήματος, ενώ το δεύτερο κεφάλαιο, Γέφυρες και
Διαδρομές, επεκτείνεται στη μαθηματική δομή του γραφήματος. Η ενεργοποίηση της κατακόρυφης
μαθηματικοποίησης στο κεφάλαιο αυτό οδηγεί στη μελέτη διαφόρων ειδών γραφημάτων και των ιδιοτήτων
τους. Η μελέτη αυτή συνεχίζεται στο τρίτο κεφάλαιο Πύργοι και Διαδρομές. Στο τέταρτο κεφάλαιο, Δένδρα
και Δάση, μελετώνται οι ιδιότητες αυτών των γραφημάτων και η χρήση τους σε και η σχέση τους με πεδία
πέραν των Μαθηματικών, όπως λόγου χάριν η Χημεία. Στο πέμπτο κεφάλαιο, Ξεμπλέκουν οι Ακμές τους;
εισάγεται η έννοια του επίπεδου γραφήματος και ο τύπος του Euler γι’ αυτά και για τα κυρτά πολύεδρα.
Τέλος, στο έκτο κεφάλαιο, Χρωματισμοί, ολοκληρώνεται η θεωρία των γραφημάτων με τη μελέτη του
κλασικού προβλήματος του χρωματισμού του χάρτη. Στο έβδομο κεφάλαιο, Γοητευτικά Τετράγωνα, Ι,
μελετώνται τα μαγικά τετράγωνα περιττής και άρτιας τάξης, ενώ στο όγδοο κεφάλαιο Γοητευτικά Τετράγωνα,
ΙΙ, μελετώνται τα λατινικά τετράγωνα και η σχέση τους με τα μαγικά τετράγωνα. Στο ένατο κεφάλαιο,
Συμμετρίες, ομάδες, συγγένειες, με αφορμή το κανονικό μαγικό τετράγωνο και τις «ισοδύναμες» μορφές του,
μέσω των συμμετριών του τετραγώνου που είδαμε στο έβδομο κεφάλαιο, πρώτα μελετάμε τη δομή ομάδας
των συμμετριών του τετραγώνου και του ισόπλευρου τριγώνου και στη συνέχεια βλέπουμε πώς οι δύο αυτές
δομές σχετίζονται με τις δομές συγγένειας δύο φυλών που έχουν μελετήσει ανθρωπολόγοι. Στο δέκατο
κεφάλαιο, Το Ρολόι, η Σπαζοκεφαλιά 14-15, Κωδωνοκρουσίες, συνεχίζουμε τη μελέτη της δομής της ομάδας
εκκινώντας από την «αριθμητική του ρολογιού» και διερευνώντας και άλλες ομάδες μικρής τάξης. Με τη
βοήθεια της θεωρίας των ομάδων μελετάμε τη σπαζοκεφαλιά 14-15 και ορισμένες βασικές μεθόδους
κωδωνοκρουσίας. Στο ενδέκατο κεφάλαιο, Νούμερα, μελετάμε ορισμένες κλασικές αριθμητικές
σπαζοκεφαλιές, ενώ στο δωδέκατο κεφάλαιο, Παιχνίδια Τύχης, εκκινώντας από παιχνίδια τύχης με ζάρια ή
τραπουλόχαρτα, μελετάμε τα βασικά στοιχεία της θεωρίας των πιθανοτήτων. Το κεφάλαιο τελειώνει με μια
σύντομη επίσκεψη στην κριτική μαθηματική εκπαίδευση.

Στη διεθνή βιβλιογραφία, όπως ανακάλυψα αφού αποφάσισα να συγγράψω το παρόν σύγγραμμα,
υπάρχουν βιβλία συγγενικά στο «πνεύμα» του. Ενδεικτικά αναφέρω τα βιβλία των Averbach B., & Chein O.,
Problem solving through recreational mathematics, Gary Chartrand, Introductory Graph Theory, Kraitchik,
M., Mathematical Recreations, Hunter, J. A. H. & Madachy, J. S. Mathematical Diversions.

Ψυχαγωγικά Μαθηματικά 15
Η προσέγγιση, ωστόσο, που ακολουθώ στο σύγγραμμα, η «διδασκαλία μέσω επίλυσης προβλήματος»,
[βλ. Van De Walle, J. A. et al. (2017). Μαθηματικά από το Νηπιαγωγείο ως το Γυμνάσιο. Διδασκαλία με
επίκεντρο το παιδί και την ανάπτυξή του. Gutenberg.] τα διαφοροποιεί από αυτά. Με την προσέγγιση αυτή,
επιχειρώ να δείξω πώς ενδομαθηματικές και εξωμαθηματικές προβληματικές καταστάσεις οδηγούν στην
επινόηση νοητικών εργαλείων, όπως είναι οι μαθηματικές έννοιες και μέθοδοι. Ειδικότερα, επιχειρώ να
αναδείξω δύο βασικές πτυχές της μαθηματικής δραστηριότητας, την οριζόντια μαθηματικοποίηση (ή
μοντελοποίηση) και την κατακόρυφη μαθηματικοποίηση, πτυχές που αναδείχθηκαν από την προσέγγιση των
«ρεαλιστικών μαθηματικών». Με την οριζόντια μαθηματικοποίηση, η προβληματική κατάσταση παράγει ένα
μαθηματικό πρόβλημα, η επίλυση του οποίου απαιτεί τη χρήση νοητικών εργαλείων που οι λύτ(ρι)ες
επινοούν ή γνωρίζουν από προγενέστερες εμπλοκές τους με παρόμοιες καταστάσεις. Η περαιτέρω,
«κατακόρυφη», επεξεργασία των μαθηματικών εννοιών και μεθόδων που παράγονται έτσι οδηγεί σε νέες
μαθηματικές έννοιες και μεθόδους ή και σε νέες, μαθηματικές πλέον, προβληματικές καταστάσεις που
παράγουν νέα μαθηματικά ερωτήματα προς επίλυση. Κειμενικά, η υλοποίηση της προσέγγισης αυτής γίνεται
με τον χωρισμό κάθε κεφαλαίου σε δύο μέρη: το πρώτο αποτελείται από φύλλα εργασίας (στα παρεχόμενα
σελιδικά κενά των οποίων ο αναγνώστης ή η αναγνώστρια μπορεί να επιχειρήσει να απαντήσει στα
ερωτήματά τους) και το δεύτερο από τις απαντήσεις στα ερωτήματα, μέσω των οποίων αναπτύσσεται
σταδιακά η σχετική μαθηματική θεωρία.

Όπου είναι εφικτό, η επίλυση του προβλήματος γίνεται με τρόπο που να αναδεικνύονται οι τέσσερις
φάσεις που διακρίνει ο George Polya στο βιβλίο του How to Solve it: Κατανόηση του προβλήματος,
σχεδιασμός μιας στρατηγικής επίλυσης του προβλήματος, υλοποίηση της επίλυσης, αναστοχασμός.
Ειδικότερα, σε όλο το σύγγραμμα επιμένω να δείχνω τη σπουδαιότητα της τέταρτης φάσης στην ανάπτυξη
της μαθηματικής σκέψης.

Στο σύγγραμμα, οι προβληματικές καταστάσεις τοποθετούνται, κατά το δυνατόν, μέσα στα ιστορικά,
κοινωνικά και πολιτισμικά πλαίσια στα οποία αναδύθηκαν. Έτσι, αρκετές δραστηριότητες ζητούν από τον
αναγνώστη ή την αναγνώστρια να μάθει για τις μαθηματικές φυσιογνωμίες που συνέβαλαν στην επίλυση
πολλών παρουσιαζόμενων προβλημάτων επινοώντας μαθηματικές έννοιες και μεθόδους. Ενώ με κάποια
άλλα ερωτήματα, οδηγώ τον αναγνώστη ή την αναγνώστρια σε γνωριμία με τα Εθνομαθηματικά.

Πέρα από τη συνήθη πρακτική τεκμηρίωσης, η σχετική βιβλιογραφία στο τέλος κάθε κεφαλαίου
αποσκοπεί επιπροσθέτως στο να αναδείξει την κειμενική διάσταση που έχει η μαθηματική μελέτη και να
προτρέψει τον αναγνώστη ή την αναγνώστρια να προχωρήσει σε περαιτέρω μελέτη ανατρέχοντας στα βιβλία
ή τα άρθρα που παρατίθενται.

Κώστας Χατζηκυριάκου

16 Χατζηκυριάκου Κώστας
Κεφάλαιο 1
Ποτάμια και περάσματα

Σύνοψη
Η σπαζοκεφαλιά της διακομετακόμισης και η αξιοποίησή της στη διδασκαλία και στη μάθηση της θεωρίας των
γραφημάτων.

Προαπαιτούμενη γνώση
Γραφή, ανάγνωση και αριθμητική.

Ψυχαγωγικά Μαθηματικά 17
1.1 Ο πραματευτής, ο λύκος, το πρόβατο και το φορτίο λάχανα
Ένας πραματευτής ταξιδεύει, έχοντας μαζί του έναν λύκο, ένα πρόβατο και ένα φορτίο με λάχανα. Κάποια
μέρα βρίσκεται στην όχθη ενός ποταμού και το μοναδικό μέσο για να περάσει απέναντι είναι μια βαρκούλα
που χωρά μόνο αυτόν και ένα από τα ζώα ή το φορτίο με τα λάχανα. Είναι φανερό ότι ο έμπορος δεν μπορεί
να αφήσει μόνο του τον λύκο με το πρόβατο ούτε το πρόβατο με το φορτίο με τα λάχανα.

Μπορεί να περάσει απέναντι με όλα τα υπάρχοντά του; Αν ναι, πώς;

18 Χατζηκυριάκου Κώστας
1.2 Οι τρεις φίλοι και οι αδελφές τους
Τρεις φίλοι, καθένας με την αδελφή του, φθάνουν στην όχθη ενός ποταμού. Η βάρκα με την οποία μπορούν
να περάσουν στην απέναντι όχθη χωρά μόνο δύο ανθρώπους. Ωστόσο, κάθε φίλος εποφθαλμιά την αδελφή
των άλλων.

Είναι δυνατό να περάσουν απέναντι υπό αυτές τις συνθήκες; Αν ναι, πόσες διαδρομές απαιτούνται;

Ψυχαγωγικά Μαθηματικά 19
1.3 Ο παχύς και η παχιά
Ένας παχύς άνδρας με την παχιά σύζυγό του και τα δύο παιδιά τους φτάνουν στην όχθη ενός ποταμού. Η
μικρή βάρκα με την οποία μπορούν να περάσουν στην απέναντι όχθη χωρά μόνο τον άνδρα ή μόνο τη
γυναίκα ή μόνο τα δύο παιδιά.

Είναι δυνατό να περάσουν απέναντι υπό αυτές τις συνθήκες; Αν ναι, πόσες διαδρομές απαιτούνται;

20 Χατζηκυριάκου Κώστας
1.4 Λίγη ιστορία και… ακόμη λιγότερη ανθρωπολογία
Τα τρία προηγούμενα προβλήματα είναι τo 18ο, το 17ο και το 19ο πρόβλημα της συλλογής προβλημάτων που
είναι γνωστή ως Propositiones ad acuendos juvenes (Προτάσεις για την άσκηση της οξύνοιας των νέων), η
κατάρτιση της οποίας αποδίδεται στον Αλκουίνο (738-804) και έχει χαρακτηριστεί ως το πρώτο κείμενο
«διασκεδαστικών μαθηματικών».

Παραλλαγές του 18ου προβλήματος βρίσκουμε και σε πολλούς άλλους μη δυτικούς πολιτισμούς. Το
πρόβλημα αυτό, μάλιστα, έχει ταξινομηθεί με τον κωδικό H 506.3 (τεστ/δοκιμασία ευρηματικότητας) ως
μοτίβο δημώδους αφήγησης στον σχετικό κατάλογο του Stith Thompson.

Tο 17ο πρόβλημα απαντά σε χειρόγραφα του 13ου, 14ου και 15ου αιώνα, αλλά και σε βιβλία του 16ου
αιώνα όπως το General Trattato του Nicolo Tartaglia (1505;-1557). Ωστόσο, σε αυτά οι τρεις αδελφοί και οι
αδελφές τους έχουν γίνει τρία ανδρόγυνα, με τους συζύγους, όμως, το ίδιο ζηλιάρηδες. Σύμφωνα με τον
Singmaster, o Luca Pacioli (1447;-1517) διατύπωσε για πρώτη φορά το ίδιο πρόβλημα με περισσότερα
ζευγάρια και έδειξε ότι τρία ή τέσσερα ζευγάρια μπορούν να περάσουν απέναντι αν η βάρκα χωρά τρεις
ανθρώπους.

I. Αφού συμβουλευτείς μια εγκυκλοπαίδεια ή μια Ιστορία των Μαθηματικών ή ένα βιογραφικό λεξικό, ή
έγκυρο ιστότοπο, γράψε ένα σύντομο βιογραφικό (η έκταση του οποίου να μην ξεπερνά τις διακόσιες
λέξεις):
i. του Αλκουίνου,
ii. του Luca Pacioli,
iii. του Nicolo Tartaglia.

Ψυχαγωγικά Μαθηματικά 21
II. Αναζήτησε περισσότερες πληροφορίες για τη συλλογή Propositiones ad acuendos juvenes. Τι είδους
είναι τα υπόλοιπα προβλήματα που περιέχει;

22 Χατζηκυριάκου Κώστας
1.5 Παραλλαγές και επεκτάσεις
I. Σχημάτισε το γράφημα του προβλήματος για δύο ζευγάρια (των οποίων οι άνδρες δεν διανοούνται να
αφήσουν τη γυναίκα τους με τον άλλον άνδρα) και διθέσια βάρκα.
i. Μπορούν να περάσουν στην απέναντι όχθη;
ii. Αν ναι, πόσα περάσματα απαιτούνται;

Ψυχαγωγικά Μαθηματικά 23
II. Δείξε ότι ο Tartaglia έσφαλε όταν ισχυρίστηκε ότι τέσσερα ζευγάρια (με τους συζύγους το ίδιο
ζηλιάρηδες) μπορούν να περάσουν στην απέναντι όχθη με διθέσια βάρκα.

24 Χατζηκυριάκου Κώστας
III. Δείξε όμως ότι, όπως πρότεινε ο μαθητής λυκείου Μ. Cadet de Fonteney το 1879, είναι δυνατό να
περάσουν απέναντι αν στο μέσον του ποταμού υπάρχει ένα νησάκι. Πόσες διαδρομές απαιτούνται αν:
i. είναι υποχρεωτικό να πιάσει η βάρκα στο νησάκι;
ii. δεν είναι υποχρεωτικό να πιάσει η βάρκα στο νησάκι;

Ψυχαγωγικά Μαθηματικά 25
IV. Μελέτησε το ίδιο πρόβλημα αν τα ζευγάρια ήταν τέσσερα ή πέντε (οι σύζυγοι εξίσου ζηλιάρηδες) και η
βάρκα είναι τριθέσια.

26 Χατζηκυριάκου Κώστας
1.6 Αποτίμηση του «προβλήματος της διακομετακομίσεως» ως διδακτικού εργαλείου
Ο μαθηματικός και ιστορικός των μαθηματικών G. Β. Loria (1862-1954), σχολιάζοντας τη συλλογή
Propositiones ad acuendos juvenes, γράφει:

Πρόκειται περί μιας συλλογής από την οποίαν δεν ελλείπουν ζητήματα ξένα προς τα
επιστημονικά μαθηματικά, είτε διότι αποτελούν απήχησιν αρχαίου μυστικισμού, είτε διότι
ανήκουν εις την τάξιν των παραδόξων εκείνων αινιγμάτων, τα οποία ακόμη και σήμερον
προσφέρουν ευχαρίστησιν εις τους αργόσχολους. Μεταξύ των τελευταίων είναι και το
πρόβλημα της διακομετακομίσεως... (Ιστορία των Μαθηματικών, 1928).

Τώρα που έχεις εργαστεί με τα προβλήματα αυτά, τα έχεις, ίσως, συζητήσει με άλλους και έχεις δει τις
παραλλαγές, τις επεκτάσεις και τις χρήσεις τους, συμφωνείς με την άποψη του G. B. Loria; Αιτιολόγησε την
απάντησή σου.

Ψυχαγωγικά Μαθηματικά 27
Απαντήσεις στα ερωτήματα του 1ου Κεφαλαίου

1.1 Ο πραματευτής, ο λύκος, το πρόβατο και το φορτίο λάχανα


Πρώτη Λύση Δεύτερη Λύση

Π, Λ, λ, π Π, Λ, λ, π
𝜫, 𝝅 𝜫, 𝝅
Λ, λ → Λ, λ →
𝜫 𝜫
Π, Λ, λ ← π Λ, λ ← π
𝜫, 𝜦 𝜫, 𝝀
λ → π Λ → π
𝜫, 𝝅 𝜫, 𝝅
λ ← Λ Λ ← λ
𝜫, 𝝀 𝜫, 𝜦
π → Λ π → λ
𝜫 𝜫
π ← Λ, λ π ← Λ, λ
𝜫, 𝝅 𝜫, 𝝅
→ Λ, λ → Λ, λ
Π, Λ, λ, π Π, Λ, λ, π

Επομένως, για να περάσει ο πραματευτής απέναντι με όλα του τα υπάρχοντα απαιτούνται 7 διαδρομές.

Τις δύο παραπάνω λύσεις θα μπορούσαμε να τις βρούμε ταυτόχρονα και αλλιώς, αν είχαμε επιλέξει μια
διαφορετική στρατηγική επίλυσης, μια διαφορετική αναπαράσταση, αυτήν που προτείνεται στην άσκηση 4.1,
σ. 118 του C.L.Liu, Elements of Discrete Mathematics, ως εξής: Παρατηρούμε ότι οι παρακάτω θέσεις του
πραματευτή (Π), του προβάτου (π), του λύκου (Λ) και του φορτίου λάχανα (λ) είναι οι μόνες αποδεκτές.

Α. (Π, π, Λ, λ│∅)
Β. (Π, Λ, λ│π)
Γ. ( Π, π, λ│Λ)
Δ. (Π, π, Λ│λ)
Ε. (Λ, λ│π, Π)
Ζ. (Π, π│Λ, λ)
Η. (π│Π, Λ, λ)
Θ. (Λ│Π, π, λ)
Ι. (λ│Π, π, Λ)
Κ. (∅│Π, π, Λ, λ)

Επιπλέον, Α↔Ε, Β↔Ε, Β↔Θ, Β↔Ι, Γ↔Η, Γ↔Ι, Δ↔Η, Δ↔Θ, Ζ↔Η, Ζ↔Κ, υπό την έννοια ότι οι
ισοδύναμες θέσεις προκύπτουν η μια από την άλλη με ένα πέρασμα του πραγματευτή. Επομένως, οι δύο
τρόποι να περάσει ο πραματευτής στην απέναντι όχθη με όλα τα υπάρχοντά του σώα και ασφαλή είναι οι:

Α↔Ε↔Β↔Θ↔Δ↔Η↔Ζ↔Κ και Α↔Ε↔Β↔Ι↔Γ↔Η↔Ζ↔Κ

Ψυχαγωγικά Μαθηματικά 29
Τα παραπάνω είναι δυνατό να τα αναπαραστήσουμε με τη βοήθεια ενός μαθηματικού αντικειμένου,
του απλού γραφήματος, που συνολοθεωρητικά μπορεί να οριστεί ως εξής:

Απλό γράφημα G ονομάζουμε ένα διατεταγμένο ζευγάρι (V, E), στο οποίο το V είναι ένα μη κενό
σύνολο σημείων που λέγονται κορυφές και το Ε είναι ένα σύνολο διμελών υποσυνόλων του V που
ονομάζονται ακμές.

Στο πρόβλημά μας, το σύνολο των κορυφών V = {Α, Β, Γ, Δ, Ε, Ζ, Η, Θ, Ι, Κ} είναι το σύνολο των
αποδεκτών θέσεων και το σύνολο των ακμών Ε = {{Α,Ε}, {Ε, Β}, {Β, Θ}, {Β, Ι}, {Ι, Γ}, {Γ, Η}, {Θ, Δ}, {Δ,
Η}, {Η, Ζ}, {Ζ, Κ}} είναι οι αποδεκτές θέσεις που προκύπτουν η μια από την άλλη με ένα μόνο πέρασμα του
πραματευτή. Είναι φανερό ότι οι ακολουθίες κορυφών Α, Ε, Β, Θ, Δ, Η, Ζ, Κ και Α, Ε, Β, Ι, Γ, Η, Ζ, Κ είναι
οι μόνοι δύο δυνατοί τρόποι για να περάσει ο πραματευτής στην απέναντι όχθη με όλα τα υπάρχοντά του σώα
και ασφαλή.

Σχήμα 1.1

Παρατήρησε ότι η αναστοχαστική ερώτηση «Μπορείτε να βρείτε το αποτέλεσμα με διαφορετικό τρόπο;» που
προτείνει ο Polya στο τέταρτο βήμα της μεθόδου του για την επίλυση προβλημάτων, στο βιβλίο του Πώς να
το λύσω, μας οδήγησε στην αναζήτηση και εύρεση μιας άλλης προσέγγισης που μας έδωσε την επιπλέον
πληροφορία ότι αυτές οι δύο λύσεις είναι και οι μοναδικές.

Παρατήρησε, επίσης, ότι κατά την επίλυσή μας κάναμε δύο κινήσεις: μία οριζόντια (μοντελοποιώντας
μαθηματικά ένα πρόβλημα) και μία κατακόρυφη (επινοώντας ή χρησιμοποιώντας μαθηματικά αντικείμενα ή
μαθηματικές έννοιες, μαθηματικά εργαλεία, που εντέλει η ύπαρξή τους ανεξαρτητοποιείται από το ιδιαίτερο

30 Χατζηκυριάκου Κώστας
πλαίσιο μέσα στο οποίο επινοήθηκαν). Σχετικά με τη σημαντική, για τη μαθηματική εκπαίδευση, διάκριση
μεταξύ οριζόντιας και κατακόρυφης μαθηματικοποίησης, βλ. σ. 41-44 στο βιβλίο του Hans Freudenthal
(2002) Revisiting the Mathematics Education, The China Lectures, Kluwer Academic Publishers.

1.2 Οι τρεις φίλοι και οι αδελφές τους

Αα Ββ Γγ
𝑨, 𝜶
ΒβΓγ →
𝜜
ΒβΓγ ← α
𝛽, 𝛾
ΑΒΓ → α
𝛼
ΑΒΓ ← βγ
𝛣, 𝛤
Αα → βγ
𝛣, 𝛽
Αα ← Γγ
𝛢, 𝛣
αβ → Γγ
𝛾
αβ ← ΑΒΓ
𝛾, 𝛽
α → ΑΒΓ
𝛢
α ← ΒβΓγ
𝛢, 𝛼
→ ΒβΓγ
ΑαΒβΓγ

Όπως επισημαίνει ο Polya στο δεύτερο βήμα της μεθόδου του, αναγνωρίζοντας την ομοιότητα του
προβλήματος αυτού με το προηγούμενο, είναι εύλογο να επιχειρήσουμε να το λύσουμε φτιάχνοντας πάλι ένα
μεγάλο απλό γράφημα του οποίου οι κορυφές θα ήταν οι αποδεκτές θέσεις. Έτσι, κάνοντας τις κατάλληλες
προσαρμογές, με (3,3,1) θα συμβολίζουμε την αρχική θέση όταν και τα τρία ζευγάρια και η βάρκα βρίσκονται
στην αρχική όχθη. Είναι φανερό ότι η τελική θέση είναι η (0,0,0). Παρατήρησε ότι για να είναι καταρχήν
αποδεκτή μία τριάδα, πρέπει ο αριθμός των γυναικών να μην είναι μεγαλύτερος από τον αριθμό των ανδρών
ούτε στη μία ούτε στην άλλη όχθη. Επομένως, οι θέσεις (2,3,0), (2,3,1), (1,3,0), (1,3,1), (1,0,1), (1,0,0),
(2,0,1), (2,0,0) δεν είναι αποδεκτές, αφού σε αυτές τουλάχιστον μία από τις τρεις γυναίκες δεν συνοδεύεται
στην αρχική όχθη ή στην απέναντι όχθη.

Παρατήρησε, επίσης, ότι η τριάδα (0,1,1) μπορεί να αντιστοιχεί μόνο σε μία θέση που η προηγούμενή
της είναι η τελική (0,0,0), δηλαδή το πέρασμα έχει ολοκληρωθεί με επιτυχία, ενώ οι προηγούμενες θέσεις της
(0,3,0) είναι η (1, 3, 1) ή η (2, 3, 1), οι οποίες δεν είναι εφικτές.

Οι αποδεκτές θέσεις είναι τελικά οι ακόλουθες:


Α(3,3,1), Β(3,2,1), Γ(3,2,0), Δ(3,1,1), Ε(3,1,0), Ζ(3,0,1), Η(3,0,0), Θ(2,2,1), Ι(2,2,0), Κ(1,1,1), Λ(1,1,0),
Μ(0,3,1), Ν(0,2,1), Ξ(0,2,0), Ο(0,1,0), Π(0,0,0).

Τώρα, οι ισοδύναμες θέσεις είναι οι ακόλουθες:

Α↔Ε, Α↔Ι, Α↔Γ, Ι↔Β, Ε↔Β, Β↔Η, Η↔Δ, Δ↔Λ, Λ↔Θ, Θ↔Ξ, Ξ↔Μ, Μ↔Ο, Ο↔Κ, Ο↔Ν,
Κ↔Π, Ν↔Π.

Επομένως, είναι δυνατό να πάμε από την Α στην Π ακολουθώντας την ακολουθία Α, Ε, Β, Η, Δ, Λ, Θ,
Ξ, Μ, Ο, Κ, Π ή την Α, Ι, Β, Η, Δ, Λ, Θ, Ξ, Μ, Ο, Κ, Π ή την ακολουθία Α, Ι, Β, Η, Δ, Λ, Θ, Ξ, Μ, Ο, Ν, Π ή

Ψυχαγωγικά Μαθηματικά 31
την ακολουθία Α, Ε, Β, Η, Δ, Λ, Θ, Ξ, Μ, Ο, Ν, Π. Οποιαδήποτε από αυτές απαιτεί 11 περάσματα, από τα
οποία το πρώτο και το τελευταίο είναι το πέρασμα ενός ζευγαριού ή δύο γυναικών.

Σχήμα 1.2

Στο βήμα του αναστοχασμού, τώρα, παρατήρησε ότι η λύση αυτή μπορεί να παραχθεί μηχανικά. Ξεκίνα από
την αρχική θέση (3,3,1) και προχώρα σε νέες αποδεκτές θέσεις αφαιρώντας από και προσθέτοντας σε
προηγούμενες αποδεκτές θέσεις εναλλάξ τις διατεταγμένες τριάδες (1,1,1), (2,0,1), (1,0,1), (0,2,1), (0,1,1) έως
ότου προκύψει η τελική θέση (0,0,0).

Τη μέθοδο αυτή χρησιμοποίησε o Saul Amarel (Θεσσαλονίκη 1928-Princeton 2002) σε ένα άρθρο που
έγραψε, το 1968, για τη μηχανική μάθηση, On presentations of problems of reasoning about actions, το οποίο
άσκησε μεγάλη επίδραση στον κλάδο της Τεχνητής Νοημοσύνης.

Διαδικτυακή δραστηριότητα: Αναζήτησε στο διαδίκτυο πληροφορίες για τη ζωή του Saul Amarel.

1.3 Ο παχύς και η παχιά


Α, Γ →
𝛼, 𝜅

Α, Γ ←
𝛼 κ

Α, α →
𝛤 κ

Α, α ←
𝜅 Γ

Α →
𝛼, 𝜅 Γ

Α ←
𝛼 Γ, κ

α →
𝛢 Γ, κ

α ←
𝜅 Α, Γ


𝛼, 𝜅 Α, Γ

Α, Γ, α, κ

32 Χατζηκυριάκου Κώστας
Είναι φανερό ότι υπάρχουν οκτώ διαφορετικές λύσεις.

Θα μπορούσαμε να σχηματίσουμε και πάλι ένα μεγάλο, απλό γράφημα, του οποίου οι κορυφές είναι οι
αποδεκτές θέσεις και οι κορυφές που συνδέονται είναι αυτές που προκύπτουν η μια από την άλλη με ένα μόνο
πέρασμα.

Σχήμα 1.3

Α(2,2,1)
↙↗ ↓↑ ↘↖
Β(1,2,0) Γ(2,1,0) Δ(2,0,0)
↓↑
Ε(2,1,1)
↓↑
Ζ(1,1,0)
↓↑
Η(1,2,1)
↙↗ ↓↑
Θ(0,2,0) Ι(1,0,0)
↙↗ ↓↑
Κ(2,0,1) Λ(1,1,1)
↓↑
Μ(0,1,0)
↓↑
Ν(0,2,1)
↓↑
Ξ(0,0,0)

Ψυχαγωγικά Μαθηματικά 33
Από το γράφημα με σύνολο κορυφών {Α, Β, Γ, Δ, Ε, Ζ, Η, Θ, Ι, Κ, Λ, Μ, Ν, Ξ} και σύνολο ακμών {ΑΒ, ΑΓ,
ΑΔ, ΔΕ, ΕΖ, ΖΗ, ΗΘ, HI, ΙΚ, ΙΛ, ΛΜ, ΜΝ, ΝΞ} βλέπουμε ότι η διαδρομή Α, Δ, Ε, Ζ, Η, Ι, Λ, Μ, Ν, Ξ είναι
η ζητούμενη.

Στο αναστοχαστικό βήμα, παρατήρησε ότι το γράφημά μας έχει τις εξής ιδιότητες :

i. Έχει 14 κορυφές και 13 ακμές.


ii. Δεν έχει «κυκλώματα» («κλειστές διαδρομές»), όπως π.χ. το κύκλωμα ΒΘΔΗΓΙΒ στο γράφημα
του Σχήματος 1.2 ή το κύκλωμα ΑΕΒΙΑ στο γράφημα του Σχήματος 1.4.
iii. Οποιεσδήποτε δύο κορυφές του συνδέονται με έναν μοναδικό τρόπο, ενώ στα προηγούμενα
γραφήματα (του Σχήματος 1.2 ή του Σχήματος 1.3 αυτό δεν ισχύει).

Με τι θα παρομοίαζες το σχήμα του γραφήματος;

1.4 Λίγη ιστορία και… ακόμη λιγότερη ανθρωπολογία


Εάν δεν τον έχεις ανακαλύψει ήδη, ένας πολύ έγκυρος ιστότοπος για βιογραφικά μαθηματικών και θεμάτων
σχετικών με την ιστορία των Μαθηματικών είναι ο https://mathshistory.st-andrews.ac.uk/

Πολύ καλό άρθρο για τη συλλογή προβλημάτων Propositiones ad acuendos juvenes του Αλκουίνου
είναι το άρθρο των Hadley J. και Singmaster D., Problems to Sharpen the Young.

1.5 Παραλλαγές και επεκτάσεις


I. Οι κορυφές του γραφήματος στο πρόβλημα αυτό θα είναι πάλι τριάδες της μορφής (Α, Γ, β), όπου Α
πλήθος ανδρών, Γ πλήθος γυναικών στην αρχική όχθη και β = 0 ή 1 ανάλογα με το αν η βάρκα είναι εκεί
ή όχι. Τώρα, οι αποδεκτές τριάδες είναι Α(2, 2, 1), Β(2, 1, 1), Γ(1, 1, 1), Δ(0, 2, 1), Ε(2, 1, 0), Ζ(2, 0, 0),
Η(1, 1, 0), Θ(0, 2, 0), Ι(0, 1, 0), Κ(0, 0, 0).
Οι ισοδύναμες θέσεις είναι οι A↔Ε, A↔Ζ, Α↔Η, Α↔Θ, Β↔Ι, Β↔Ζ, Β↔Η, Γ↔Κ, Γ↔Ι, Δ↔Κ,
Δ↔Ι. Επομένως, έχουμε τέσσερις εναλλακτικές λύσεις, την Α, Ζ, Β, Ι, Γ, Κ και την Α, Η, Β, Ι, Δ, Κ, την
Α, Η, Β, Ι, Γ, Κ και την Α, Ζ, Β, Ι, Δ, Κ.

II. Η βασική παρατήρηση είναι ότι σε όλα τα προβλήματα των περασμάτων, εκτός από το τελευταίο
πέρασμα, όλα τα υπόλοιπα από την αρχική όχθη στην απέναντι προσθέτουν, εφόσον προσθέτουν, μόνο
ένα άτομο (κάποιος/κάποια πρέπει να επιστρέψει τη βάρκα στην αρχική όχθη για να συνεχιστούν τα
περάσματα). Επομένως, σε κάποια φάση των περασμάτων θα πρέπει να υπάρχουν πέντε άτομα στην
απέναντι όχθη. Τέσσερις καταστάσεις είναι καταρχήν δυνατές:
a. 4 άνδρες, 1 γυναίκα ή b. 3 άνδρες, 2 γυναίκες ή

c. 2 άνδρες, 3 γυναίκες ή d. 1 άνδρας, 4 γυναίκες.

Η 3η και η 4η δεν είναι όμως δυνατές, επειδή οι γυναίκες είναι περισσότερες και άρα, τουλάχιστον
μία δεν είναι μαζί με τον άνδρα της ενώ άλλοι άνδρες είναι παρόντες στην απέναντι όχθη, ενώ η 2η δεν
είναι δυνατή γιατί αυτό συμβαίνει στην αρχική όχθη (2 γυναίκες, 1 άνδρας). Η πρώτη δεν είναι επίσης
δυνατή τελικά γιατί προέκυψε είτε από την άφιξη ενός ζευγαριού (οπότε στην άλλη όχθη βρίσκονταν ο
άνδρας του ζευγαριού και 3 ξένες γυναίκες μόνες τους, που δεν γίνεται) είτε από την άφιξη δύο ανδρών
(οπότε στην άλλη όχθη βρίσκονταν δύο άνδρες με 3 γυναίκες, που πάλι δεν γίνεται).

III. Ο Fonteney πρότεινε μια λύση με 24 περάσματα, στην οποία η στάση στο νησάκι δεν ήταν υποχρεωτική.
Η καλύτερη τέτοια απαιτεί, όμως, μόνο 16 περάσματα. Ωστόσο, εάν η στάση στο νησάκι είναι

34 Χατζηκυριάκου Κώστας
υποχρεωτική, τότε απαιτούνται 26 περάσματα για μια λύση.

IV. 11 περάσματα για 4 ζευγάρια και τριθέσια βάρκα, 13 περάσματα για 5 ζευγάρια και τριθέσια βάρκα, για ν
ζευγάρια (ν > 5) με τετραθέσια βάρκα απαιτούνται 2∙ν +1 περάσματα.
Για λεπτομέρειες, γενικεύσεις και παραλλαγές μελέτησε το άρθρο των Pressman, I. & Singmaster, D.
“The Jealous Husbands” and “The Missionaries and Cannibals”.

Ψυχαγωγικά Μαθηματικά 35
Βιβλιογραφία/Αναφορές

Amarel, S. (1968). On presentations of problems of reasoning about actions. Machine Intelligence 3, 131–171.
Freudenthal, H. (2002). Revisiting the Mathematics Education, The China Lectures. Kluwer Academic
Publishers.
Hadley, J., & Singmaster, D. (1992). Problems to Sharpen the Young. The Mathematical Gazette, 76(475),
102–126.
Liu, C. L. (1977). Elements of Discrete Mathematics. McGraw-Hill computer science series.
Loria, G. (1971, 1972, 1974). Ιστορία των Μαθηματικών (Τόμοι 3). Ελληνική Μαθηματική Εταιρεία -
Εκδόσεις Παπαζήση.
Polya, G. (1998). Πώς να το λύσω. Εκδόσεις Καρδαμίτσα.
Pressman, I., & Singmaster, D. (1989). The Jealous Husbands & The Missionaries and Cannibals. The
Mathematical Gazette, 73(464), 73–81.
Hadley, J., & Singmaster, D. (1992). Problems to Sharpen the Young. The Mathematics Gazette, 76(475),
102–126.

36 Χατζηκυριάκου Κώστας
Κεφάλαιο 2
Γέφυρες και διαδρομές

Σύνοψη
Εισαγωγή στη θεωρία γραφημάτων: η έννοια του γραφήματος, του υπογραφήματος, του ισομορφισμού
γραφημάτων, ενδιαφέροντα και σημαντικά γραφήματα, oϋλεριανά και ημι-οϋλεριανά γραφήματα. Τα
Εθνομαθηματικά.

Προαπαιτούμενη γνώση
Γραφή, ανάγνωση και αριθμητική.

Ψυχαγωγικά Μαθηματικά 37
2.1 Οι επτά γέφυρες του Κένινγκσμπεργκ

Εικόνα 2.1
https://en.wikipedia.org/wiki/Seven_Bridges_of_K%C3%B6nigsberg#/media/File:Konigsberg_bridges.png

Τον 18ο αιώνα η πόλη Κένιγκσμπεργκ, που σήμερα ονομάζεται Καλίνιγκραντ και ανήκει στη Ρωσία, ήταν
σπουδαίο πνευματικό κέντρο της Πρωσίας. Εκεί γεννήθηκε, έζησε, έγραψε και δίδαξε στο Πανεπιστήμιο

Αλμπερτίνα ο σπουδαίος φιλόσοφος Εμμάνουελ Καντ (Immanuel Kant, 1724-1804).

I. Αφού συμβουλευτείς μια εγκυκλοπαίδεια ή ένα βιογραφικό λεξικό ή έναν έγκυρο ιστότοπο, γράψε ένα
σύντομο βιογραφικό (η έκταση του οποίου να μην ξεπερνά τις διακόσιες λέξεις) του Εμμάνουελ Καντ.

38 Χατζηκυριάκου Κώστας
II. Την πόλη Κένιγκσμπεργκ διασχίζει ο ποταμός Πρέγκελ, και τον 18ο αιώνα τα δύο νησάκια του
συνδέονταν με τις δύο όχθες του με τις εφτά γέφυρες που είναι σημαδεμένες στον παραπάνω χάρτη. Για
τις εφτά αυτές γέφυρες του Κένιγκσμπεργκ είχε επινοηθεί η εξής περίφημη σπαζοκεφαλιά:
Μπορεί ένας κάτοικος να ξεκινήσει από οποιαδήποτε όχθη του ποταμού και να ολοκληρώσει τη
βόλτα του επιστρέφοντας σε αυτήν έχοντας διασχίσει ακριβώς μία φορά κάθε γέφυρα; Ναι, όχι, γιατί;

Ψυχαγωγικά Μαθηματικά 39
III. Την σπαζοκεφαλιά έλυσε ο σπουδαίος μαθηματικός Λέοναρντ Όυλερ (Leonhard Euler, 1707-1783) το
1735.
Αφού συμβουλευτείς μια εγκυκλοπαίδεια ή μια Ιστορία των Μαθηματικών ή ένα βιογραφικό λεξικό
ή έναν έγκυρο ιστότοπο, γράψε ένα σύντομο βιογραφικό (η έκταση του οποίου να μην ξεπερνά τις
διακόσιες λέξεις) του Όυλερ.

40 Χατζηκυριάκου Κώστας
2.2 Η μονοκοντυλιά
I. Σε καθένα από τα παρακάτω δύο σχήματα, μπορείς να ξεκινήσεις από το σημείο που υποδεικνύεται και
σέρνοντας το μολύβι σου πάνω στη συνεχή γραμμή τους, χωρίς να το σηκώσεις καθόλου, να καταλήξεις
πάλι στο σημείο από το οποίο ξεκίνησες; Είναι, δηλαδή, τα παρακάτω σχήματα μονοκοντυλιές;

Σχήμα 2.2

Ψυχαγωγικά Μαθηματικά 41
II. Στο βιβλίο του Brian Bolt, Μαθηματικές Σπαζοκεφαλιές, υπάρχει και η ακόλουθη σπαζοκεφαλιά:
Το παρακάτω σχήμα αναπαριστά ένα οδικό δίκτυο. Ο κατασκευαστής του πριν το παραδώσει στην
κυκλοφορία θέλει να το διασχίσει μία φορά ο ίδιος, ξεκινώντας από τη θέση Α, επιστρέφοντας σε αυτήν και
περνώντας από κάθε τμήμα του (ακμή) μόνο μία φορά. Μπορεί να το κάνει αυτό; Αν ναι, πώς; Αν όχι, γιατί;

Σχήμα 2.3

42 Χατζηκυριάκου Κώστας
III. Μπορείς να ξεκινήσεις από οποιαδήποτε κορυφή του παρακάτω γραφήματος και σέρνοντας το μολύβι
σου πάνω στη συνεχή γραμμή τους, χωρίς να το σηκώσεις καθόλου, να καταλήξεις πάλι στο ίδιο σημείο;
Γράψε την κλειστή διαδρομή που θα ακολουθήσεις εάν το σχήμα είναι μονοκοντυλιά.

Σχήμα 2.4

Ψυχαγωγικά Μαθηματικά 43
IV. Είναι το παρακάτω σχήμα μονοκοντυλιά; Ναι, όχι, γιατί; (Αν είναι, μπορείς να δείξεις τη διαδρομή;)

Εικόνα 2.5
https://books.google.com/books?id=12cLAAAAYAAJ&hl=el&pg=PA62#v=onepage&q&f=false

44 Χατζηκυριάκου Κώστας
2.3 Δωμάτια
I. Ένας κατάσκοπος εισέβαλε σε ένα κυβερνητικό κτήριο υψίστης ασφαλείας. Οι κρυμμένες κάμερες
ασφαλείας απέδειξαν ότι ο κατάσκοπος, αναζητώντας πληροφορίες, πέρασε μία μόνο φορά από κάθε
πόρτα του κτηρίου, πριν συλληφθεί την ώρα που προσπαθούσε να διαρρήξει το χρηματοκιβώτιο.
Σε ποιο δωμάτιο ήταν το χρηματοκιβώτιο; Γιατί;

[Πηγή: Bolt, B. (1990). Μαθηματικές Σπαζοκεφαλιές. Κάτοπτρο].

Α Β

Γ Δ Ε

Σχήμα 2.6

Ψυχαγωγικά Μαθηματικά 45
II. Ξεκινώντας από οποιαδήποτε από τα πέντε δωμάτια ή την αυλή του παρακάτω σπιτιού, μπορείς να
περάσεις από όλες τις πόρτες του ακριβώς μία φορά και να επιστρέψεις ακριβώς εκεί από όπου ξεκίνησες;

Σχήμα 2.7

46 Χατζηκυριάκου Κώστας
2.4 Χειραψίες και γάμοι
I. Πέντε μαθηματικοί συναντιούνται σε ένα συνέδριο θεωρίας γραφημάτων και ανταλλάσσουν χειραψίες
όλοι με όλους. Πόσες χειραψίες ανταλλάχτηκαν συνολικά; Πόσες θα ήταν οι αντίστοιχες χειραψίες, αν οι
μαθηματικοί ήταν έξι; Αν ήταν 𝜈;

Ψυχαγωγικά Μαθηματικά 47
II. Στο διάλειμμα για καφέ, τρεις από τους παραπάνω μαθηματικούς γνωρίζονται με τρεις συναδέλφισσές
τους και ανταλλάσσουν χειραψίες όλοι με όλες. Πόσες χειραψίες ανταλλάχτηκαν συνολικά; Πόσες θα
ήταν οι αντίστοιχες χειραψίες αν οι άνδρες μαθηματικοί ήταν 𝜇 και οι γυναίκες μαθηματικοί 𝜈;

48 Χατζηκυριάκου Κώστας
III. Καθένας από τους άνδρες α, β, γ, δ γνωρίζει κάποια ή κάποιες από τις γυναίκες Α, Β, Γ, Δ, Ε. Ειδικότερα,
ο α γνωρίζει τις Α, Δ, Ε, ο β την Α, ο γ τις Β, Γ, Δ και ο δ τις Β, Δ. Είναι δυνατόν καθένας από τους
άνδρες να παντρευτεί μία γνωστή του; (Η κοινωνία στην οποία ζουν όλοι και όλες είναι μονογαμική).

Ψυχαγωγικά Μαθηματικά 49
2.5 Γραφήματα
I. Δείξε (σχεδιάζοντάς τα) ότι υπάρχουν ακριβώς τέσσερα μη ισόμορφα απλά γραφήματα με τρεις κορυφές
και ένδεκα μη ισόμορφα με τέσσερις κορυφές. Πόσα μη ισόμορφα υπάρχουν με πέντε κορυφές;

50 Χατζηκυριάκου Κώστας
II. Υπάρχει γράφημα με έξι κορυφές των οποίων οι βαθμοί είναι 5, 5, 5, 5, 3, 3; Ναι, όχι, γιατί; Υπάρχει
απλό γράφημα τέτοιο; Υπάρχει γράφημα με έξι κορυφές των οποίων οι βαθμοί είναι 5, 5, 4, 3, 3, 2;
Υπάρχει απλό γράφημα τέτοιο;

Ψυχαγωγικά Μαθηματικά 51
III. Δείξε ότι το γράφημα του Petersen και το παρακάτω γράφημα είναι ισόμορφα.

Σχήμα 2.8

52 Χατζηκυριάκου Κώστας
IV. Δείξε ότι τα παρακάτω δύο γραφήματα δεν είναι ισόμορφα.

Σχήμα 2.9

Ψυχαγωγικά Μαθηματικά 53
V. Σχεδίασε τα έξι υπογραφήματα του παρακάτω γραφήματος που σχηματίζονται αφαιρώντας μία μόνο
κορυφή τη φορά, και δείξε ότι δεν είναι ισόμορφα μεταξύ τους.

Σχήμα 2.10

54 Χατζηκυριάκου Κώστας
Απαντήσεις στα ερωτήματα του 2ου Kεφαλαίου

2.1 Οι επτά γέφυρες του Κένινγκσμπεργκ


I. Έγκυρoς ιστότοπος που αφορά τη φιλοσοφία είναι ο https://plato.stanford.edu/, όπου μπορείτε να
μελετήσετε το άρθρο για τον Καντ.

II. Την αρνητική απάντηση στη σπαζοκεφαλιά έδωσε ο σπουδαίος μαθηματικός Λέοναρντ Όυλερ
(Leonhard Euler, 1707-1783) το 1735. Την απάντησή του μπορούν να τη μελετήσουν οι λατινομαθείς στο
άρθρο του Solutio problematis ad geometriam situs pertinentis, το οποίο δημοσιεύτηκε το 1741 και
θεωρείται το γενέθλιο κείμενο της θεωρίας των γραφημάτων, τα βασικά σημεία της οποίας θα δούμε
παρακάτω.
Λέγοντας γράφημα (γράφο) G εννοούμε ένα πεπερασμένο σύνολο σημείων V (τις κορυφές του
γραφήματος) μαζί με ένα πολυσύνολο (multiset) γραμμών E (τις ακμές του γραφήματος) που έχουν άκρα όλα
ή ορισμένα από τα παραπάνω σημεία. Είναι επίσης δυνατό να υπάρχει ακμή που συνδέει μια κορυφή με τον
εαυτό της (βρόχος). Ο βρόχος σε μια κορυφή Α γράφεται ΑΑ.

Μπορούμε να περιγράψουμε μαθηματικά την προβληματική κατάσταση που αντιμετωπίζουμε στη


σπαζοκεφαλιά των επτά γεφυρών του Κένιγκσμπεργκ, δημιουργώντας ένα μαθηματικό μοντέλο της. Το
μαθηματικό αντικείμενο που προκύπτει είναι ένα γράφημα G, το οποίο, όμως, δεν είναι απλό, καθώς
υπάρχουν κορυφές που συνδέονται με περισσότερες από μία ακμές. Στο γράφημα αυτό, οι δύο όχθες είναι οι
κορυφές Α, Β και τα δύο νησάκια είναι οι κορυφές C, D, ενώ οι ακμές είναι οι επτά γέφυρες (Σχήμα 2.11).

Σχήμα 2.11

Επομένως, G = ({Α, Β, C, D}, {AB, AB, AC, AC, AD, CD, BD}).

Αν δεν υπάρχουν σημεία που συνδέονται με περισσότερες από μία ακμές, ούτε βρόχοι, τότε το
γράφημα λέγεται απλό. Απόλυτα ασύνδετο λέμε κάθε γράφημα που το σύνολο των ακμών του είναι κενό. Το
σύνολο των κορυφών είναι πάντα μη κενό σύνολο.

Σε πιο αυστηρή μαθηματική γλώσσα, γράφημα G ονομάζουμε ένα διατεταγμένο ζευγάρι (V, E), στο
οποίο το V είναι ένα μη κενό σύνολο σημείων τα οποία λέγονται κορυφές και το Ε είναι ένα πολυσύνολο
(multiset) διμελών υποπολυσυνόλων του V τα οποία ονομάζονται ακμές.

Ψυχαγωγικά Μαθηματικά 55
Βαθμός μιας κορυφής είναι o αριθμός των ακμών που την έχουν άκρο. Ειδικότερα, κάθε βρόχος
συνεισφέρει δύο μονάδες στον βαθμό της κορυφής. Μια κορυφή που έχει βαθμό μηδέν λέγεται
απομονωμένη.

Είναι φανερό ότι το άθροισμα των βαθμών των κορυφών είναι ίσο με τον διπλάσιο αριθμό των ακμών,
αφού κάθε ακμή συνεισφέρει δύο μονάδες στο άθροισμα αυτό. Η πρόταση αυτή αποκαλείται συχνά στη
βιβλιογραφία λήμμα των χειραψιών.

Ένα γράφημα μπορεί να αναπαρασταθεί «γραφιστικά» με διαφορετικούς τρόπους. Η μαθηματική


έννοια της ισομορφίας έρχεται να συλλάβει το ίδιο, πίσω από τις διαφορετικές αναπαραστάσεις. Δύο
γραφήματα λέγονται ισόμορφα εάν υπάρχει 1-1 αντιστοιχία φ μεταξύ των κορυφών τους, έτσι ώστε {u, v}
είναι ακμή του ενός αν και μόνο αν (ανν) {φ(u), φ(v)} είναι ακμή του άλλου.

Ένα γράφημα G1 λέγεται υπογράφημα ενός γραφήματος G2 αν το σύνολο των κορυφών του G1 είναι
υποσύνολο του συνόλου των κορυφών του G2 και το πολυσύνολο των ακμών του G1 είναι υποπολυσύνολο
των ακμών του G2.

A B
Θ
Α Β
Ε Ζ

Δ Γ
Δ Γ Η
G2
G1
G
Σχήμα 2.12

Το γράφημα G (στο Σχήμα 2.12) έχει 4 κορυφές, τις Α, Β, Γ, Δ και 8 ακμές από τις οποίες η μία είναι
βρόχος. Προφανώς, το γράφημα δεν είναι απλό. Ο βαθμός της κορυφής Α είναι 3, των κορυφών Β και Γ
είναι 4, ενώ της Δ είναι 5.

Άθροισμα βαθμών του G = 3 + 4+ 4+ 5 = 16 = 2·8.

To G2 (στο Σχήμα 2.12) είναι απλό γράφημα, υπογράφημα του G, ενώ το G1 στο ίδιο Σχήμα είναι
ισόμορφο με το G (η αντιστοιχία Α ↔ E, Β↔Z, Γ↔H, Δ↔Θ είναι ισομορφία).

Λέγοντας διαδρομή από την κορυφή 𝑢0 στην κορυφή 𝑢𝑛 ενός γραφήματος εννοούμε μια ακολουθία
από διαφορετικές ακμές 𝑒0 = 𝑢0 𝑢1 , 𝑒1 = 𝑢1 𝑢2 , . . ., 𝑒𝑛−1 = 𝑢𝑛−1 𝑢𝑛 , που συνδέει τις κορυφές 𝑢0 και 𝑢𝑛 .
Λέγοντας μονοπάτι εννοούμε μια διαδρομή στην οποία όλες οι κορυφές 𝑢0 , 𝑢1 , 𝑢2 , . . ., 𝑢𝑛−1 , 𝑢𝑛 (εκτός ίσως
από την πρώτη και την τελευταία) είναι διαφορετικές. Αν η πρώτη και η τελευταία κορυφή μιας διαδρομής
ταυτίζονται, τότε η διαδρομή λέγεται κλειστή, ενώ το μονοπάτι λέγεται κλειστό (ή κύκλωμα).

Συνεκτικό λέγεται ένα γράφημα αν οποιεσδήποτε δύο κορυφές του συνδέονται με ένα τουλάχιστον
μονοπάτι. Επομένως, συνεκτικά είναι τα γραφήματα που αποτελούνται από ένα μόνο κομμάτι (μία
συνιστώσα).

Προς τιμήν του Όυλερ έχουν δοθεί οι ακόλουθοι δύο ορισμοί: Ένα συνεκτικό γράφημα λέγεται
οϋλεριανό αν υπάρχει κλειστή διαδρομή που περιέχει όλες τις ακμές του γραφήματος. Λέγεται ημι-
οϋλεριανό αν η διαδρομή αυτή δεν είναι κατ’ ανάγκη κλειστή.

56 Χατζηκυριάκου Κώστας
Επομένως, κάθε οϋλεριανό γράφημα είναι ημιοϋλεριανό. Το αντίστροφο δεν ισχύει: υπάρχουν
ημιοϋλεριανά γραφήματα που δεν είναι οϋλεριανά, π.χ. το γράφημα G2 (υπάρχει οϋλεριανή διαδρομή με άκρα
τις κορυφές Γ, Δ).

Ο Όυλερ έδειξε με δύο διαφορετικούς τρόπους ότι η απάντηση στη σπαζοκεφαλιά Οι εφτά γέφυρες του
Κένιγκσμπεργκ είναι αρνητική, δηλαδή, σε γραφοθεωρητική γλώσσα, ότι το γράφημα G δεν είναι οϋλεριανό.

Στη δεύτερη λύση του επιχειρηματολογεί κατ’ ουσίαν ως εξής: Εάν υπήρχε κλειστή οϋλεριανή
διαδρομή, αυτή κάθε φορά που περνά από μια κορυφή συνεισφέρει δύο μονάδες στον βαθμό της. Αλλά κάθε
ακμή υπάρχει ακριβώς μία φορά σε αυτήν τη διαδρομή, άρα κάθε κορυφή έχει κατ’ ανάγκην άρτιο βαθμό. Οι
βαθμοί των κορυφών, όμως, στο γράφημα που αντιστοιχεί στη σπαζοκεφαλιά μας είναι όλοι περιττοί, Α(5)
και Β(3), C(3), D(3). Επομένως, το γράφημα δεν είναι οϋλεριανό.

Είναι φανερό εξάλλου ότι το G δεν είναι ούτε ημιοϋλεριανό, αφού αν ήταν, θα υπήρχε οϋλεριανή
διαδρομή που δεν θα ήταν όμως κλειστή. Για να συνέβαινε αυτό, είναι φανερό ότι θα έπρεπε να υπάρχουν
ακριβώς δύο κορυφές με περιττό βαθμό και οι υπόλοιπες να είχαν άρτιο βαθμό, και φυσικά το γράφημά μας
δεν είναι ούτε τέτοιο.

Έχουμε δείξει, λοιπόν, ότι αν ένα συνεκτικό γράφημα είναι οϋλεριανό, τότε κάθε κορυφή του έχει
άρτιο βαθμό, ενώ αν είναι ημιοϋλεριανό έχει ακριβώς δύο κορυφές με περιττό βαθμό (και όλες οι
υπόλοιπες έχουν άρτιο βαθμό).

Ισχύουν και οι αντίστροφες προτάσεις, δηλαδή:

• Ένα συνεκτικό γράφημα του οποίου όλες οι κορυφές έχουν άρτιο βαθμό είναι οϋλεριανό.
• Ένα συνεκτικό γράφημα που έχει ακριβώς δύο κορυφές με περιττό βαθμό (οι υπόλοιπες έχουν
άρτιο) είναι ημιοϋλεριανό.

Ας δούμε πώς μπορούμε να αποδείξουμε την πρώτη από αυτές. Η απόδειξη θα γίνει με τη μέθοδο της
(πλήρους ή ισχυρής) μαθηματικής επαγωγής για τον αριθμό των ακμών, σύμφωνα με την οποία μια
πρόταση 𝑝(𝑘) που αναφέρεται στον φυσικό αριθμό 𝑘 ισχύει για όλους τους φυσικούς αριθμούς ν ≥
𝑘0 , εφόσον, υποθέτοντας ότι η πρόταση ισχύει για όλους τους φυσικούς 𝜈 < 𝑘0 , μπορούμε να δείξουμε ότι
ισχύει και για τον 𝑘0 .

Η αποδεικτική μέθοδος της (πλήρους ή ισχυρής) μαθηματικής επαγωγής είναι ισοσθενής μορφή της
απλής μαθηματικής επαγωγής (σύμφωνα με την οποία μία πρόταση 𝑝(𝑘) που αναφέρεται στον φυσικό αριθμό
𝑘 ισχύει για όλους τους φυσικούς αριθμούς ν ≥ 𝑘0 , εφόσον ισχύει για 𝜈 = 𝑘0 , και υποθέτοντας ότι η
πρόταση ισχύει για οποιονδήποτε φυσικό αριθμό 𝜈, μπορούμε να δείξουμε ότι ισχύει και για τον επόμενο
φυσικό αριθμό 𝜈 + 1). Η αποδεικτική μέθοδος της επαγωγής βασίζεται στη βασική μας αριθμητική
διαίσθηση, σύμφωνα με την οποία ένα σύνολο φυσικών που περιέχει τον φυσικό αριθμό 𝑘 και αποδεδειγμένα
σε αυτό ανήκει ο επόμενος οποιουδήποτε φυσικού που περιέχει δεν μπορεί παρά να αποτελείται από όλους
τους φυσικούς αριθμούς μεγαλύτερους από ή ίσους με τον 𝑘.

Είναι φανερό ότι κάθε κορυφή του γραφήματος έχει βαθμό τουλάχιστον δύο (αφού το γράφημα είναι
συνεκτικό). Ένα τέτοιο γράφημα αναγκαστικά διαθέτει κύκλωμα. Γιατί; Διότι αν δεν είναι απλό, αυτό είναι
προφανές. Αν είναι απλό, ξεκίνα από μια οποιαδήποτε κορυφή του, πήγαινε σε μια οποιαδήποτε διπλανή του,
από εκεί πήγαινε σε μια οποιαδήποτε διπλανή του (μόνο το πισωγύρισμα απαγορεύεται, και επειδή ο βαθμός
της είναι τουλάχιστον δύο, αυτό είναι πάντα εφικτό). Κάποτε, όμως, οι κορυφές θα εξαντληθούν και,
επομένως, θα πρέπει να επιλέξουμε κάποια κορυφή που ήδη έχει επιλεγεί, οπότε έχουμε το κύκλωμά μας.

Το κύκλωμα αυτό C ή περιέχει όλες τις ακμές, οπότε τελειώσαμε, ή δεν τις περιέχει. Σε αυτήν την
περίπτωση, σβήνουμε τις ακμές του κυκλώματος και σχηματίζουμε ένα νέο γράφημα Η, το οποίο έχει
λιγότερες ακμές από το αρχικό και είναι πιθανόν μη συνεκτικό. Αλλά κάθε συνιστώσα του Η έχει, λόγω της

Ψυχαγωγικά Μαθηματικά 57
υπόθεσή μας, οϋλεριανή κλειστή διαδρομή. Βρίσκουμε την οϋλεριανή κλειστή διαδρομή για το αρχικό
γράφημα ξεκινώντας από μια κορυφή του κυκλώματος C, ακολουθώντας το παρακαμπτήριο κύκλωμα
κάποιας συνιστώσας που εκκινεί από κάποια μη απομονωμένη κορυφή του Η, μετά ξαναμπαίνοντας στο C
κ.ο.κ., έως ότου καταλήξουμε στην αρχική κορυφή.

Την απόδειξη των αντίστροφων δεν την έκανε ο Όυλερ, αλλά ένας άλλος Γερμανός μαθηματικός, ο
Karl Hierholzer (1840-1871), 135 χρόνια αργότερα. Η απόδειξη που μόλις είδες είναι αυτή που προτείνει ο R.
J. Wilson στο βιβλίο του Introduction to Graph Theory.

Αν θέλεις να μάθεις ποιος είναι ο άλλος τρόπος με τον οποίο ο Όυλερ έλυσε το πρόβλημα, αλλά και
ενδιαφέρουσες πληροφορίες γύρω από την ιστορία της σπαζοκεφαλιάς των επτά γεφυρών του
Κένιγκσμπεργκ, μελέτησε το άρθρο του R. J. Wilson, An Eulerian Trail Through Königsberg.

Ένα πολύ γνωστό και χρήσιμο κανονικό γράφημα που δεν είναι οϋλεριανό, αφού όλες οι κορυφές του
έχουν περιττό βαθμό, είναι το γράφημα του Petersen (Σχήμα 2.13).

Σχήμα 2.13

III. Για τον Euler μπορείς να διαβάσεις στον διαδικτυακό τόπο https://mathshistory.st-
andrews.ac.uk/Biographies/Euler/

2.2 Η μονοκοντυλιά
I. Βλέποντας τόσο το σχήμα a όσο και το σχήμα b του Σχήματος 2.2 ως γραφήματα των οποίων οι
κορυφές είναι οι «κόμβοι», διαπιστώνουμε ότι και τα δύο είναι οϋλεριανά, αφού κάθε κορυφή τους
έχει βαθμό 4. Γραφήματα των οποίων όλες οι κορυφές έχουν τον ίδιο βαθμό λέγονται κανονικά. Είναι
φανερό ότι όλα τα κανονικά γραφήματα στα οποία ο βαθμός κάθε κορυφής είναι άρτιος είναι
οϋλεριανά.
Οι μονοκοντυλιές σε αυτό το φύλλο εργασίας είναι σχήματα που χαράζουν τελετουργικά στην άμμο
οι άνδρες στο νησί Μαλεκούλα. Η Martha Ascher μελετά τις μονοκοντυλιές αυτές αλλά και άλλες στο
άρθρο της Graphs in Culture: A study in Ethnomathematics, αλλά και στο βιβλίο της Ethnomathematics,
A Multicultural View of Mathematical Ideas.

Τα Εθνομαθηματικά (Ethnomathematics) είναι ένα κίνημα στο πεδίο της μαθηματικής


εκπαίδευσης που επιχειρεί να αναδείξει και να αξιοποιήσει διδακτικά τις μαθηματικές ιδέες που
εμφωλιάζουν σε πρακτικές που επιτελούν διάφορες «κουλτούρες» ή «υποκουλτούρες» (οι όροι
χρησιμοποιούνται με την ανθρωπολογική τους σημασία).

58 Χατζηκυριάκου Κώστας
Aν θέλεις να μάθεις περισσότερα για τα Εθνομαθηματικά, μπορείς να ξεκινήσεις μελετώντας τα
D’Ambrosio, U., Ethnomathematics and its place in the history and pedagogy of mathematics και Gerdes,
P., Reflections on Ethnomathematics.

II. Το γράφημα του οδικού δικτύου στο Σχήμα 2.3 έχει 5 κορυφές. Είναι επίσης κανονικό, δηλαδή όλες οι
κορυφές του έχουν τον ίδιο βαθμό, εδώ κάθε κορυφή έχει βαθμό 4. Επομένως, είναι οϋλεριανό και η
απάντηση στο ερώτημα είναι ναι. Μια οϋλεριανή κλειστή διαδρομή είναι το ΑΒ, ΒΓ, ΓΔ, ΔΕ, ΕΑ, ΑΓ,
ΓΕ, ΕΒ, ΒΔ, ΔΑ. Το γράφημα αυτό είναι απλό, συνεκτικό και έχει ακόμη μια ενδιαφέρουσα ιδιότητα:
κάθε κορυφή συνδέεται με οποιαδήποτε άλλη. Ένα τέτοιο γράφημα λέγεται πλήρες. Το πλήρες γράφημα
με ν κορυφές συμβολίζεται Κν. Επομένως, το γράφημα του οδικού δικτύου είναι το Κ5. Έχει 10 ακμές.
Πόσες ακμές έχει το Κν; Αφού για κάθε ακμή χρειάζονται δύο κορυφές, άρα όλες οι ακμές είναι
τόσες όσοι είναι όλοι οι διαφορετικοί τρόποι να σχηματίσουμε δυάδες με 𝜈 πράγματα, ή σε πιο
μαθηματική γλώσσα όσα είναι όλα τα διμελή υποσύνολα ενός συνόλου με πληθικό αριθμό 𝜈 ή, ακόμη,
όσοι είναι οι συνδυασμοί 𝜈 πραγμάτων ανά δύο, δηλ. νΣ2.

Εάν γνωρίζεις, λοιπόν, πώς να λύσεις αυτό το πρόβλημα, θα έλεγε ο Polya (Πώς να το λύσω), τότε
έχεις την απάντηση και για το πλήθος των ακμών του Κν. Αλλά:

𝜈∙(𝜈−1)
ν Σ2 = .
2

Γιατί; Διότι με 𝜈 πράγματα μπορώ να σχηματίσω 𝜈 ∙ (𝜈 − 1) διατεταγμένα ζεύγη [𝜈 επιλογές για την
πρώτη θέση και (𝜈 − 1) για τη δεύτερη], αλλά δεν με ενδιαφέρει η σειρά σχηματισμού, οπότε πρέπει να
διαιρέσω δια δύο για να βρω το πλήθος των διμελών υποσυνόλων ενός συνόλου με 𝜈 στοιχεία.

Σε αυτό το σημείο, ο Polya θα μας συμβούλευε και πάλι να αναρωτηθούμε: «Μπορούμε να βρούμε
αυτόν τον αριθμό και αλλιώς»; Ναι. Πήγαινε σε μια τυχούσα κορυφή και γράψε όλες τις ακμές που τη
συνδέουν με τις υπόλοιπες ν. Πόσες είναι; Είναι 𝜈. Πήγαινε στη διπλανή της και γράψε όλες τις ακμές που
τη συνδέουν με τις υπόλοιπες, πλην της αρχικής. Πόσες είναι; Είναι 𝜈 − 1. Κάνε το ίδιο ώσπου να
καταλήξεις στην προπροτελευταία και την τελευταία κορυφή, ανάμεσα στις οποίες υπάρχει μόνο μία
ακμή. Οπότε, το σύνολο των ακμών είναι:

𝜈∙(𝜈−1)
1 + 2 + 3 + . . . + (𝜈 − 1) = 2

χάρις στην περίφημη ταυτότητα του Gauss. Παρεμπιπτόντως, αυτός είναι ένας άλλος τρόπος υπολογισμού
του νΣ2.

Ο βαθμός κάθε κορυφής του Κν είναι 𝜈 − 3. Επομένως, τα οϋλεριανά πλήρη γραφήματα είναι εκείνα
που το πλήθος των κορυφών τους 𝜈 είναι 3 + 2 ∙ ρ, δηλ. 𝜈 = 3, 5, 7, 9, . . . .

Ως μαθηματικό αντικείμενο της ευκλείδειας γεωμετρίας, το Κ ν αναπαριστά ένα ν-γωνο με


σχεδιασμένες όλες τις διαγωνίους του. Πόσες είναι αυτές; Προφανώς:

𝜈∙(𝜈−1) 𝜈∙(𝜈−3)
−𝜈 =
2 2
Αξίζει να παρατηρήσουμε εδώ ότι μια σπαζοκεφαλιά όπως αυτή του οδικού δικτύου μας έδωσε την
ευκαιρία να συνδέσουμε μαθηματικές έννοιες από τέσσερις διαφορετικές μαθηματικές περιοχές: τη
θεωρία γραφημάτων, τη θεωρία συνόλων, τη συνδυαστική, αλλά και την ευκλείδεια γεωμετρία. Δύσκολα
μπορεί να αμφισβητήσει κανείς ότι οι πολλαπλές αυτές αναπαραστάσεις (σε ορολογία γνωστικής
ψυχολογίας) ή γεφυρώσεις (σε ορολογία διδακτικής) συντελούν στη βαθύτερη κατανόηση και πιο στέρεα
μάθηση των μαθηματικών εννοιών που υπεισέρχονται στην επίλυση της προβληματικής κατάστασης που
αντιμετωπίσαμε.

Ψυχαγωγικά Μαθηματικά 59
III. Όλες οι κορυφές του γραφήματος στο Σχήμα 2.4 έχουν άρτιο βαθμό, επομένως το γράφημα είναι
οϋλεριανό. Μπορούμε να βρούμε μια κλειστή οϋλεριανή διαδρομή, γι’ αυτό με τον εξής αλγόριθμο
(αλγόριθμος του Fleury):
• Ξεκίνα από οποιαδήποτε κορυφή του και καθώς κινείσαι, σβήνε κάθε ακμή που περνάς, καθώς
και κάθε απομονωμένη κορυφή που προκύπτει έτσι. Καθώς κινείσαι, επιλέγεις ακμή ισθμό μόνο
αν δεν έχεις άλλη επιλογή.
• Αν θέλεις να πειστείς ότι ο αλγόριθμος αυτός είναι αποτελεσματικός, μελέτησε τη σχετική
απόδειξη στο R. J. Wilson, Introduction to Graph Theory.
Επομένως, μια οϋλεριανή κλειστή διαδρομή είναι η ΑΔ, ΔΕ, ΕΘ, ΘΖ, ΖΗ, ΗΕ, ΕΖ, ΖΓ, ΓΒ, ΒΔ, ΔΓ, ΓΑ.

IV. Το γράφημα του Σχήματος 2.5 έχει μόνο δύο κορυφές περιττού βαθμού (5), οι οποίες είναι τα άκρα του
οριζόντιου άξονα συμμετρίας του. Οι άλλες κορυφές είναι άρτιου βαθμού, επομένως το γράφημα είναι
ημι-οϋλεριανό και υπάρχει οϋλεριανή διαδρομή από το ένα ως το άλλο η οποία, φυσικά, δεν είναι
κλειστή. To γράφημα αυτό υπάρχει στο έργο του J. B. Listing, Vorstudien zur Topdogie (1846), στο οποίο
ο εισηγητής του όρου «τοπολογία» μελετά και μονοκοντυλιές. (Βλ. R. J. Wilson, An Eulerian Trail
Through Königsberg.)

2.3 Δωμάτια
I. Τώρα που γνωρίζουμε λίγη θεωρία γραφημάτων, θα μπορούσαμε να τη χρησιμοποιήσουμε για να
φτιάξουμε ένα μοντέλο της κατάστασης που θα μας βοηθήσει να λύσουμε αυτό το πρόβλημα. Υπάρχουν
έξι περιοχές στο σχέδιο του Σχήματος 2.6 οι οποίες (όπως στο μοντέλο του Όυλερ για τις 7 γέφυρες του
Κένιγκσμπεργκ) μπορούν να αναπαρασταθούν με 6 κορυφές Α, Β, Γ, Δ, Ε, Ζ. Οι ακμές στο γράφημα Χ
που θα φτιάξουμε θα δηλώνουν όλους τους δυνατούς τρόπους με τους οποίους μπορούμε να πάμε άμεσα
από τη μια περιοχή στην άλλη μέσω μιας πόρτας. Επομένως:
Χ = ({Α, Β, Γ, Δ, Ε, Ζ, Η}, {ΑΖ, ΑΖ, ΑΒ, ΑΓ, ΑΔ, ΑΔ, ΒΖ, ΒΖ, ΒΕ, ΓΔ, ΓΖ, ΓΖ, ΔΖ, ΔΕ, ΕΖ, ΕΖ}).

Παρατηρούμε ότι οι βαθμοί των κορυφών Α, Β, Γ, Δ, Ε, Ζ είναι 6, 4, 4, 4, 5, 4, 9. Επομένως, ο


κατάσκοπος που μπαίνει στον διάδρομο Ζ συλλαμβάνεται στο δωμάτιο Δ όπου βρίσκεται και το
χρηματοκιβώτιο.

II. Είναι φανερό πια ότι κατ’ αναλογίαν προς τα προηγούμενα προβλήματα, και αυτή η σπαζοκεφαλιά
μπορεί να λυθεί με το φτιάξιμο ενός γραφήματος με κορυφές τις έξι περιοχές και ακμές ανάμεσα σε
εκείνες τις κορυφές που αντιστοιχούν σε περιοχές που επικοινωνούν άμεσα μέσω μιας πόρτας. Εάν Α, Β,
Γ είναι οι κορυφές που αναπαριστάνουν τα πάνω τρία δωμάτια (αριστερό, μεσαίο και δεξιό αντίστοιχα),
Δ, Ε είναι οι κορυφές που αναπαριστάνουν τα άλλα δύο (αριστερό, δεξιό) και Ζ είναι η κορυφή που
αναπαριστά το εξωτερικό του διαμερίσματος (Σχήμα 2.7), τότε βλέπουμε εύκολα ότι οι αντίστοιχοι
βαθμοί τους είναι 4, 5, 4, 5, 5, 9. Επομένως, το γράφημα δεν είναι οϋλεριανό και η απάντηση στο
ερώτημα είναι αρνητική.

2.4 Χειραψίες
I. Αν μαθηματικοποιήσουμε οριζόντια, δηλαδή αν μοντελοποιήσουμε την προβληματική κατάστασή μας
στη θεωρία γραφημάτων, τότε μπορούμε να φανταστούμε ότι οι κορυφές αναπαριστάνουν ανθρώπους και
οι ακμές τις χειραψίες τους, οπότε τα ερωτήματά μας γίνονται: Πόσες είναι όλες οι ακμές: του Κ5, του Κ6,
του Κν;

60 Χατζηκυριάκου Κώστας
𝜈∙(𝜈−1)
Αλλά τις απαντήσεις σε αυτά τα ερωτήματα τις γνωρίζουμε: 10 ακμές, 15 ακμές, ακμές.
2
Φυσικά, θα μπορούσαμε να είχαμε λύσει το πρόβλημα κάνοντας τις ίδιες σκέψεις που κάναμε για να
λύσουμε το πρόβλημα του πλήθους των ακμών του Κν. Αυτό που αξίζει να παρατηρήσουμε είναι ότι αν
γνωρίζουμε πώς να λύσουμε το ένα πρόβλημα, μπορούμε να λύσουμε και το άλλο ανάγοντας το άγνωστο
στο γνωστό.

II. Αφού καθένας από τους τρεις άνδρες μαθηματικούς κάνει χειραψία με καθεμία από τις τρεις γυναίκες
μαθηματικούς (από τον ορισμό του πολλαπλασιασμού (!) ή μέσω της μεθόδου των τριών), το πλήθος
όλων των χειραψιών που ανταλλάσσονται είναι 3·3 = 9. Παρόμοια, αν οι άνδρες ήταν 𝜇 και οι γυναίκες 𝜈,
το πλήθος των χειραψιών θα ήταν 𝜇·𝜈.
Ξανακοιτάζοντας το πρόβλημα προτού το εγκαταλείψουμε, όπως μας προτείνει ο Polya, ας δούμε τι
γράφημα προκύπτει αν πάλι μοντελοποιήσουμε την προβληματική κατάστασή μας στη θεωρία
γραφημάτων και φανταστούμε ότι οι κορυφές αναπαριστάνουν ανθρώπους και οι ακμές τις χειραψίες
τους.

Παρατηρούμε ότι το γράφημα στο Σχήμα 2.14 είναι διμερές, δηλαδή το σύνολο των κορυφών του V
= {A, B, Γ, Δ, Ε, Ζ} μπορεί να γραφτεί ως ένωση δύο υποσυνόλων του M = {A, B, Γ} και W = {Δ, Ε, Ζ},
ξένων μεταξύ τους, με τέτοιο, μάλιστα, τρόπο ώστε τα άκρα οποιασδήποτε ακμής του γραφήματος να μην
ανήκουν στο ίδιο υποσύνολο. Επιπροσθέτως, όλες οι κορυφές του ενός υποσυνόλου συνδέονται άμεσα
μέσω μιας ακμής με κάθε κορυφή του άλλου υποσυνόλου. Ένα τέτοιο διμερές γράφημα λέγεται πλήρες.
Το γράφημα V συμβολίζεται με Κ3,3.

Σχήμα 2.14
Είναι φανερό ότι αφού το πλήθος των κορυφών ενός πλήρους διμερούς γραφήματος Κ ρ,σ είναι 𝜈 =
𝜌 + 𝜎 (όπου 𝜌, 𝜎 είναι οι πληθικοί αριθμοί των δύο υποσυνόλων στα οποία το σύνολο των κορυφών
διαμερίζεται), το πλήθος των ακμών του είναι 𝜌 ∙ 𝜎.

Ένα πλήρες διμερές γράφημα είναι οϋλεριανό ανν τόσο το 𝜌 όσο και το 𝜎 είναι άρτιοι αριθμοί.

Το γράφημα Κ3,3 δεν είναι προφανώς οϋλεριανό.

III. Είναι φανερό ότι τον τρόπο με τον οποίον γνωρίζονται οι άνδρες και οι γυναίκες μπορούμε να τον
αναπαραστήσουμε με το διμερές γράφημα που έχει απεικονιστεί μόνο με μαύρο χρώμα. Τότε, οι κόκκινες
ακμές του υπεργραφήματος που δημιουργήσαμε δείχνουν έναν τρόπο που μπορούν να γίνουν αυτοί οι
γάμοι.

Σχήμα 2.15

Ψυχαγωγικά Μαθηματικά 61
Αν αναστοχαστούμε την προβληματική αυτή κατάσταση και τη λύση της, είναι δυνατόν να θέσουμε
το εξής γαμήλιο ερώτημα: Σε μια μονογαμική κοινωνία, υπάρχει ικανή ή αναγκαία συνθήκη που πρέπει
να ισχύει ώστε, αν καθένας από κάποιους άνδρες γνωρίζει κάποιες γυναίκες, να μπορέσει καθένας να
παντρευτεί κάποια που γνωρίζει; Το ερώτημα αυτό το απάντησε ο Philip Hall το 1935 αποδεικνύοντας το
γαμήλιο θεώρημα.

Χρειαζόμαστε πρώτα έναν ορισμό. Αν το Γ = Γ(Κ1, Κ2) είναι διμερές γράφημα, μια 1-1 αντιστοίχιση
Κ1 ↔Κ ⊆ Κ2 λέγεται πλήρες ταίριασμα, αν οι κορυφές που αντιστοιχίζονται είναι τα δύο άκρα μιας
ακμής.

Έστω Γ = Γ(Κ1, Κ2) διμερές γράφημα και για κάθε Χ⊆Κ1 έστω φ(Χ) το σύνολο εκείνων των
κορυφών του Κ2 που συνδέονται με ακμή με τουλάχιστον μία κορυφή του Χ. Το Γ έχει πλήρες ταίριασμα
ανν για κάθε Χ⊆Κ1, |Χ| ≤ |φ(Χ)|. Η απόδειξη που ακολουθεί είναι των Halmos και Vaughan.

Είναι φανερό ότι η συνθήκη είναι αναγκαία, αφού αν δεν ίσχυε για κάποιο Χ⊆Κ1, τα στοιχεία του Χ
δεν θα ήταν δυνατόν να αντιστοιχιστούν με 1-1 τρόπο με στοιχεία του Κ2. Θα δείξουμε ότι η συνθήκη
είναι και ικανή, με τη βοήθεια της μαθηματικής επαγωγής. Έστω ότι το θεώρημα αληθεύει όταν |Κ1| < m
(για m = 1, το θεώρημα είναι ολοφάνερο), και έστω τώρα |Κ1| = m.

Θα διακρίνουμε δύο περιπτώσεις. Πρώτη περίπτωση: Για κάθε Χ⊆Κ1, με |Χ| = k < m, k+1 ≤ φ(Χ).
Αν αντιστοιχίσουμε μια οποιαδήποτε κορυφή του Κ1 με μια κορυφή του Κ2 με την οποία συνδέεται, τότε
η αρχική συνθήκη ισχύει για τις υπόλοιπες m–1 κορυφές του Κ1. Άρα, βάσει της υπόθεσης της
μαθηματικής επαγωγής, αυτές οι m–1 κορυφές του Κ1 αντιστοιχίζονται με πλήρες ταίριασμα με m–1
κορυφές του Κ2, οπότε σε αυτήν την περίπτωση, το θεώρημα ισχύει.

Δεύτερη περίπτωση: Έστω ότι υπάρχει σύνολο Χ⊆Κ1, με |Χ| = k < m, και k = φ(Χ). Βάσει της
μαθηματικής επαγωγής, οι κορυφές του Χ μπορούν να αντιστοιχιστούν με πλήρες ταίριασμα με k
κορυφές του Κ2 και απομένει ένα σύνολο L⊆Κ1, |L| = m–k. Αλλά για κάθε Υ⊆L, |Υ| = h ≤ m–k πρέπει να
ισχύει |Υ| ≤ |φ(Υ)|, αφού αν |Υ| > |φ(Υ)|, τότε φ(Υ∪Χ) = φ(Υ) + φ(Χ) < h + k ≤ m, ανισότητα που
αντιβαίνει στην υπόθεσή μας. Άρα, η υπόθεση της μαθηματικής επαγωγής ισχύει για το L, οπότε και οι
εναπομένουσες m–k κορυφές του Κ1 μπορούν να αντιστοιχιστούν με πλήρες ταίριασμα με κορυφές του
Κ2, και το θεώρημα αποδείχτηκε.

2.5 Γραφήματα
I. Τα τέσσερα μη ισόμορφα απλά γραφήματα με 3 κορυφές είναι τα: ({Α, Β, Γ}, ∅), ({Δ, Ε, Ζ}, {ΔΕ}),
({Θ, Η, Ι}, {ΘΗ, ΗΙ}), ({Κ, Λ, Μ}, {ΚΛ, ΛΜ, ΜΚ}) και τα βλέπουμε στο Σχήμα 2.16.

Σχήμα 2.16
Για να βρούμε όλα τα μη ισόμορφα απλά γραφήματα σε 4 κορυφές, μπορούμε να σκεφτούμε ως εξής: Ας
συμβολίσουμε (1,2,3,4) το απόλυτα ασύνδετο γράφημα με 4 κορυφές (0 ακμές). Τα άλλα 10 είναι αυτά που
έχουν: μία ακμή (1–2,3,4) (παύλα ανάμεσα σε δύο κορυφές θα σημαίνει ότι υπάρχει ακμή με αυτά τα άκρα), 2
ακμές (1–2, 2–3,4), (1–3, 2–4), τρεις ακμές (1–2, 2–3, 3–4), (1–2, 2–3, 3–1), (1–2, 2–3, 2–4), (1–2, 2–3, 3–4),
τέσσερις ακμές (1–2, 2–3, 3–4, 4–1), πέντε ακμές (1–2, 2–3, 3–4, 4–1, 2–4), έξι ακμές (1–2, 2–3, 3–4, 4–1, 1–
3, 2–4).

62 Χατζηκυριάκου Κώστας
II. Γράφημα με 6 κορυφές που οι βαθμοί τους είναι 5, 5, 5, 5, 3, 3 θα έχει 13 ακμές, αφού:
5 + 5 + 5 + 5 + 3 + 3 = 26 = 2 ∙13
Ωστόσο, απλό τέτοιο γράφημα δεν υπάρχει, αφού τρεις κορυφές με βαθμό 5 είναι συνδεδεμένες με
όλες τις άλλες και επομένως, δεν επιτρέπουν την ύπαρξη τέταρτης κορυφής με βαθμό 5. Από την άλλη,
υπάρχει απλό γράφημα (άρα και γράφημα) με 6 κορυφές των οποίων οι βαθμοί είναι 5, 5, 4, 3, 3, 2. Ένα
τέτοιο γράφημα έχει 11 ακμές, αφού:

5 + 5 + 4 + 3 + 3 + 2 = 22 = 2 ∙11 (βλ. Σχήμα 2.17).

Σχήμα 2.17
III. Υπάρχουν 34 μη ισόμορφα απλά γραφήματα με 5 κορυφές (αν έχεις χρόνο προσπάθησε να τα βρεις) και
από αυτά, τα 21 είναι συνεκτικά (μπορείς να προσδιορίσεις αυτά πρώτα).
Η παρακάτω 1-1 αντιστοιχία μεταξύ του γραφήματος του Petersen και του γραφήματος 1 του Φ.Ε. 4,
Α↔Α, Β↔Β, Γ↔Γ, Δ↔Θ, Ε↔Ζ, Η↔Ε, Θ↔Η, Λ↔Λ, Ν↔Ν, Ξ↔Δ, είναι φανερό ότι είναι ισομορφία.

IV. Είναι φανερό ότι τα δυο γραφήματα του Σχήματος 2.9 δεν είναι ισόμορφα, αφού στο πρώτο, δύο από τα
άκρα των μοναδικών δύο ακμών που συνδέουν κορυφές βαθμού 3 {ΒΖ, ΔΘ} δεν συνδέονται με ακμή,
ενώ οι αντίστοιχες (μέσω οποιασδήποτε 1-1 αντιστοιχίας) {ΜΞ, ΛΟ} συνδέονται με ακμή. Βρήκαμε,
δηλαδή, ένα γνώρισμα που δεν μοιράζονται τα δύο γραφήματα, άρα δεν μπορούν να είναι ισόμορφα.

V. Τα έξι ζητούμενα υπογραφήματα τα βλέπουμε στο Σχήμα 2.18.

Σχήμα 2.18

Ψυχαγωγικά Μαθηματικά 63
Βιβλιογραφία/Αναφορές

Ascher, M. (1988). Graphs in Culture: A study in Ethnomathematics. HISTORIA MATHEMATICA 15, 201-
227.
Ascher, M. (1991). A Multicultural View of Mathematics Ideas. Brooks/Cole Publishing Company.
Bolt, B. (1990). Μαθηματικές Σπαζοκεφαλιές (τ.1). Εκδόσεις Κάτοπτρο.
D’Ambrosio, U. (1985). Ethnomathematics and its place in the history and pedagogy of mathematics. For the
Learning of Mathematics, 5(1), 44-48.
Euler, L. (1736). Solutio problematis ad geometriam situs pertinentis. Comment. Acad. Sci. U. Petrop 8, 128–
40.
Gerdes, P. (1994). Reflections on Ethnomathematics. For the Learning of Mathematics, 14(2), 19–22.
Polya, G. (1998). Πώς να το λύσω. Εκδόσεις Καρδαμίτσα.
Wilson, R. J. (1979). Introduction to Graph Theory (2nd edition). Longman.
Wilson, R. J. (2006). An Eulerian Trail Through Königsberg. Journal of Graph Theory, 10(3), 265-275

64 Χατζηκυριάκου Κώστας
Κεφάλαιο 3
Πύργοι και διαδρομές

Σύνοψη
Και άλλα σημαντικά και ενδιαφέροντα γραφήματα, χαμιλτονιανά και ημι-χαμιλτονιανά γραφήματα. Οι πύργοι
του Ανόι.

Προαπαιτούμενη γνώση
Κεφάλαιο 2 του παρόντος συγγράμματος.

Ψυχαγωγικά Μαθηματικά 65
3.1 Οι είκοσι στάσεις (The Icosian Game)
Είναι δυνατόν ξεκινώντας από οποιαδήποτε κορυφή του κανονικού γραφήματος στο Σχήμα 3.1 να
επιστρέψεις σε αυτήν ακολουθώντας μια διαδρομή που περνά από κάθε κορυφή του γραφήματος ακριβώς μία
φορά; Αν ναι, γράψε τη (μια) διαδρομή, αν όχι, προσπάθησε να εξηγήσεις γιατί δεν γίνεται.

Σχήμα 3.1

66 Χατζηκυριάκου Κώστας
3.2 Τα πλατωνικά γραφήματα
Το προηγούμενο γράφημα είναι το πλατωνικό γράφημα που προκύπτει «προβάλλοντας» το κυρτό, κανονικό
(πλατωνικό) εικοσάεδρο στο επίπεδο μιας έδρας του. Οι κορυφές του γραφήματος αντιστοιχούν στις κορυφές
του στερεού και οι ακμές του στερεού στις ακμές του γραφήματος.

Υπάρχουν μόνο άλλα τέσσερα πλατωνικά στερεά (οι έδρες καθενός είναι ίσα, μεταξύ τους, κανονικά
πολύγωνα): το τετράεδρο (κανονική τριγωνική πυραμίδα), το εξάεδρο (κύβος), το οκτάεδρο και το
δωδεκάεδρο.

Σχεδίασε τα αντίστοιχα πλατωνικά γραφήματα και δείξε ότι είναι όλα χαμιλτονιανά.

Ψυχαγωγικά Μαθηματικά 67
3.3 Οι πύργοι του Ανόι

Εικόνα 3.2
https://commons.wikimedia.org/wiki/File:Tower_of_Hanoi.jpeg

Οι πύργοι του Ανόι είναι ένα μαθηματικό παιχνίδι που αποτελείται από τρεις κατακόρυφες ράβδους και
δίσκους στοιβαγμένους σε μία από αυτές, με τον δίσκο με τη μεγαλύτερη διάμετρο στη βάση και τον δίσκο με
τη μικρότερη διάμετρο στην κορυφή (όπως στην Εικόνα 3.2). Σκοπός του παιχνιδιού είναι να μεταφερθούν
όλοι οι δίσκοι σε κάποια άλλη ράβδο και να είναι στοιβαγμένοι με τον ίδιο τρόπο. Σε κάθε κίνηση,
επιτρέπεται να μετακινούμε μόνο έναν δίσκο τη φορά, τοποθετώντας τον σε μια κενή ράβδο ή στην κορυφή
μιας στοίβας ράβδων όπου ο δίσκος στην κορυφή της έχει διάμετρο μεγαλύτερη της διαμέτρου του
μετακινούμενου δίσκου.

i. Μπορούμε να πετύχουμε τον σκοπό του παιχνιδιού όσοι κι αν είναι οι δίσκοι;


ii. Αν όχι, γιατί; Αν ναι, ποιος είναι ο μικρότερος αριθμός κινήσεων με τον οποίο μπορούμε να
πετύχουμε τον στόχο μας;

68 Χατζηκυριάκου Κώστας
3.4 To πυργάκι που τρελαίνει (Instant Insanity Puzzle)
I. Σου δίνω τέσσερις κύβους των οποίων οι έδρες είναι κόκκινες (K), πράσινες (Π), ροζ (Ρ) ή μπλε (Μ).
Κάθε χρώμα εμφανίζεται τουλάχιστον μία φορά σε κάθε κύβο. Βάλε τον έναν πάνω στον άλλον ώστε να
φτιάξεις ένα τετραώροφο πυργάκι (ορθογώνιο παραλληλεπίπεδο με διαστάσεις 1, 1, 4), κάθε έδρα του
οποίου να περιέχει κάθε χρώμα ακριβώς μία φορά. Να τα τέσσερα αναπτύγματα των κύβων:

1 Κ 2 Κ 3 Π

Κ Ρ Π Μ Κ Ρ Μ Π Μ Ρ Κ Π

Κ Ρ Π

4 Μ

Π Ρ Κ Π

Ρ
Σχήμα 3.3
(Πηγή: Wilson J. Robin, Introduction to Graph Theory, 2nd ed. Longman, 1979).

Ψυχαγωγικά Μαθηματικά 69
II. Με τους τέσσερις παρακάτω κύβους να φτιάξεις πάλι ένα τετραώροφο πυργάκι, κάθε έδρα του οποίου
περιέχει κάθε χρώμα ακριβώς μία φορά. Να τα αναπτύγματά τους:

2 Π 3 Κ
1 Μ
Ρ Ρ Κ Π Μ Μ Ρ Π
Π Κ Μ Κ
Μ Κ
Ρ

4 Ρ

Κ Μ Π Ρ

Π
Σχήμα 3.4

70 Χατζηκυριάκου Κώστας
3.5 Γραφήματα
I. Ποια πλήρη γραφήματα είναι χαμιλτονιανά;

II. Ποια πλήρη διμερή γραφήματα είναι χαμιλτονιανά;

Ψυχαγωγικά Μαθηματικά 71
III. Δείξε ότι τα διμερή γραφήματα με περιττό πλήθος ακμών δεν είναι χαμιλτονιανά.

IV. Είναι το παρακάτω γράφημα χαμιλτονιανό;

Σχήμα 3.5

72 Χατζηκυριάκου Κώστας
V. Δείξε ότι το γράφημα του Petersen είναι ημιχαμιλτονιανό, αλλά όχι χαμιλτονιανό.

VI. Δείξε ότι το γράφημα του Petersen είναι η ένωση ενός 1-παράγοντος και ενός 2-παράγοντος.

Ψυχαγωγικά Μαθηματικά 73
VII. Βρες ένα γράφημα που να είναι οϋλεριανό και χαμιλτονιανό, ένα που να είναι οϋλεριανό αλλά όχι
χαμιλτονιανό, ένα που να είναι χαμιλτονιανό αλλά όχι οϋλεριανό και ένα που να μην είναι ούτε
οϋλεριανό ούτε χαμιλτονιανό.

74 Χατζηκυριάκου Κώστας
Απαντήσεις στα ερωτήματα του 3ου Kεφαλαίου

3.1 Οι είκοσι στάσεις (The Icosian Game)


Είναι δυνατόν. Να, μία τέτοια κλειστή διαδρομή (χαμιλτονιανό κύκλωμα): ΑΖ, ZΛ, ΛΡ, ΡΣ, ΣΤ, ΤΥ, ΥΠ,
ΠΟ, ΟΚ, ΚΞ, ΞΓ, ΓΝ, ΝΘ, ΘΜ, ΜΗ, ΗΕ, ΕΔ, ΔΓ, ΓΒ, ΒΑ. Υπάρχουν άλλα πενήντα εννέα τέτοια
χαμιλτονιανά κυκλώματα. Μπορείς να βρεις άλλη μία διαδρομή, όσο το δυνατόν «πιο διαφορετική»;

Το γράφημα αυτό έχει 20 κορυφές και 30 ακμές και είναι κανονικό, αφού κάθε κορυφή έχει βαθμό 3.

Τη σπαζοκεφαλιά αυτή επινόησε ο Sir William Rowan Hamilton (1805-1865) το 1857. Πούλησε την ιδέα του
για 25 λίρες σε μια εταιρεία κατασκευής παιχνιδιών που δημιούργησε ένα επιτραπέζιο παιχνίδι που λεγόταν
Ο Γύρος του Κόσμου. Αξίζει να σημειώσουμε εδώ ότι ένας άλλος μαθηματικός, ο Thomas Kirkman (1806-
1895), δύο χρόνια πρωτύτερα είχε υποβάλει στη Βασιλική Εταιρία (Royal Society) μια εργασία στην οποία
ήταν διατυπωμένο το ερώτημα: «Υπάρχει κύκλωμα που διέρχεται από κάθε κορυφή του»;

Δραστηριότητα

α. Αφού συμβουλευτείς μια εγκυκλοπαίδεια ή ένα βιογραφικό λεξικό, ή έναν έγκυρο ιστότοπο, γράψε ένα
σύντομο βιογραφικό (η έκταση του οποίου να μην ξεπερνά τις διακόσιες λέξεις):

i. του Sir William Rowan Hamilton,


ii. του Τhomas Kirsten.
β. Σε μια εγκυκλοπαίδεια ή έναν έγκυρο ιστότοπο αναζήτησε πληροφορίες για τη Βασιλική Εταιρεία.
Προς τιμήν του Hamilton, γραφήματα που διαθέτουν κλειστή διαδρομή που περνά από κάθε κορυφή
τους ακριβώς μία φορά, δηλαδή γραφήματα που διαθέτουν χαμιλτονιανό κύκλωμα, λέγονται χαμιλτονιανά,
ενώ γραφήματα που διαθέτουν διαδρομή που περνά από κάθε κορυφή τους ακριβώς μία φορά (αλλά όχι κατ’
ανάγκη κλειστή) λέγονται ημι-χαμιλτονιανά. Επομένως, κάθε χαμιλτονιανό γράφημα είναι ημιχαμιλτονιανό,
αλλά το αντίστροφο, προφανώς, δεν ισχύει.

Το γράφημα ({Α, Β, Γ, Δ}, {ΑΓ, ΒΓ, ΓΔ}) στο Σχήμα 3.6 δεν είναι ημιχαμιλτονιανό και άρα ούτε
χαμιλτονιανό, ενώ το γράφημα ({Ε, Ζ, Η, Θ}, {ΕΗ, ΗΖ, ΕΖ, ΗΘ}) στο ίδιο σχήμα είναι ημιχαμιλτονιανό,
αλλά όχι χαμιλτονιανό.

Σχήμα 3.6

Ψυχαγωγικά Μαθηματικά 75
Δεν έχει βρεθεί ικανή και αναγκαία συνθήκη για να είναι ένα γράφημα χαμιλτονιανό. Υπάρχουν, όμως,
ικανές συνθήκες. Να δύο:

Ικανή συνθήκη του Ore (1960): Εάν το Γ είναι απλό γράφημα με 𝜈 κορυφές (𝜈 ≥ 3) και το άθροισμα
των βαθμών κάθε ζεύγους μη διαδοχικών κορυφών είναι μεγαλύτερό του ή ίσο με 𝜈, τότε το Γ είναι
χαμιλτονιανό.

Θα αποδείξουμε ότι η πρόταση αυτή αληθεύει με τη μέθοδο της εις άτοπον απαγωγής. Έστω ότι ένα
τέτοιο γράφημα δεν είναι χαμιλτονιανό. Χωρίς περιορισμό της γενικότητας, μπορούμε να υποθέσουμε ότι
αρκεί η πρόθεση μίας μόνο ακμής για να γίνει. Αλλά τότε, υπάρχει μια διαδρομή κ1→κ2, . . ., κν-1→κν που
περιέχει κάθε κορυφή. Αλλά επειδή το γράφημα δεν είναι χαμιλτονιανό, οι κορυφές κ ν-1, κν δεν είναι
διαδοχικές· άρα, το άθροισμα των βαθμών τους είναι μεγαλύτερο από ή ίσο με 𝜈. Επομένως, πρέπει να
υπάρχει κάποια κορυφή κi-1 που είναι διαδοχική τόσο της κ1 όσο και της κν. Αλλά τότε, η διαδρομή κ1→κ2, . .
., κi-1→κν→κν-1 →. . . →κi+1 →κi →κ1 είναι χαμιλτονιανό κύκλωμα, άτοπο.

Σχήμα 3.7
Ικανή συνθήκη του Dirac (1952): Εάν το Γ είναι απλό γράφημα με 𝜈 κορυφές (𝜈 ≥ 3) και ο βαθμός κάθε
𝜈
κορυφής είναι μεγαλύτερός του ή ίσος με 2, τότε το Γ είναι χαμιλτονιανό.

Παρατήρησε ότι η προγενέστερη συνθήκη έπεται από τη μεταγενέστερη. Παρατήρησε, επίσης, ότι
σίγουρα και οι δύο συνθήκες δεν μπορούν να είναι και αναγκαίες, αφού το γράφημα οποιουδήποτε 𝜈 -γωνου
𝜈
(𝜈 > 4) είναι χαμιλτονιανό και ταυτόχρονα κάθε κορυφή έχει βαθμό 2 < 2 .

Η εύρεση ενός χαμιλτονιανού κυκλώματος είναι προτυπικά δύσκολο πρόβλημα (NP-complete). Ούτε
λίγο ούτε πολύ, βρίσκεται μόνο με «έξυπνο» ψάξιμο.

Ένα διαβόητα δύσκολο πρακτικό πρόβλημα είναι το πρόβλημα του πλανόδιου πωλητή, ο οποίος
θέλει να ξεκινήσει από κάποια πόλη για να επισκεφτεί κάποιες άλλες πόλεις και να επιστρέψει στην αφετηρία
του έτσι ώστε να διανύσει τα λιγότερα χιλιόμετρα και να ελαχιστοποιήσει τα έξοδα των καυσίμων του. Το
πρόβλημα μπορεί να αναπαρασταθεί με ένα γράφημα, οι κορυφές του οποίου είναι οι πόλεις και οι ακμές του
οι δρόμοι μεταξύ των πόλεων. Οι ακμές του γραφήματος μπορούν να έχουν διαφορετικό βάρος. Εάν το
γράφημα είναι χαμιλτονιανό (και πάντα μπορούμε να υποθέσουμε πως είναι, κάνοντάς το πλήρες,
προσθέτοντας ακμές πολύ μεγάλου βάρους ανάμεσα σε κορυφές που δεν είναι άκρα ακμών), το πρόβλημα
είναι να βρούμε χαμιλτονιανό κύκλωμα που θα έχει το μικρότερο δυνατό βάρος. Είναι φανερό ότι και το
πρόβλημα αυτό είναι NP-complete. Παρατήρησε, επίσης, ότι μια χαμιλτονιανή διαδρομή (ή κύκλωμα) H ενός
γραφήματος G αποτελεί υπογράφημα του G, που περιέχει, όμως, όλες τις κορυφές του. Ένα τέτοιο
υπογράφημα H λέγεται παράγον υπογράφημα (spanning subgraph) του γραφήματος G.

76 Χατζηκυριάκου Κώστας
3.2 Τα πλατωνικά γραφήματα
Εκτός από το γράφημα του εικοσαέδρου, υπάρχουν άλλα τέσσερα πλατωνικά γραφήματα που αντιστοιχούν
στα υπόλοιπα τέσσερα πλατωνικά στερεά. Για τους λόγους που υπάρχουν μόνο πέντε πλατωνικά στερεά,
μπορείς να συμβουλευτείς το βιβλίο του Κώστα Χατζηκυριάκου, Μαθηματικά για τη Δασκάλα και τον
Δάσκαλο, Αριθμοί, Σύνολα. Το θέμα θα μας απασχολήσει και στο 5ο Κεφάλαιο του παρόντος συγγράμματος.

Σχήμα 3.8
Στο Σχήμα 3.8 βλέπουμε το γράφημα του κανονικού τετραέδρου (της κανονικής τριγωνικής πυραμίδας).

Το γράφημα αυτό έχει 4 κορυφές και 6 ακμές. Είναι κανονικό, αφού ο βαθμός κάθε κορυφής είναι 3.
4
Είναι χαμιλτονιανό (ικανοποιεί τη συνθήκη του Dirac, 3 > 2 = ). Να ένα χαμιλτονιανό κύκλωμά του: ΑΒ,
2
ΒΓ, ΓΕ, ΕΑ.

Στο Σχήμα 3.9 βλέπουμε το γράφημα του κανονικού εξαέδρου (του κύβου).

Σχήμα 3.9
Το γράφημα αυτό έχει 8 κορυφές και 10 ακμές. Είναι κανονικό, αφού ο βαθμός κάθε κορυφής είναι 3. Είναι
χαμιλτονιανό. Να ένα χαμιλτονιανό κύκλωμά του: ΑΒ, ΒΗ, ΗΘ, ΘΓ, ΓΔ, ΔΕ, ΕΖ, ΖΑ. Παρατήρησε ότι είναι
χαμιλτονιανό, αλλά ούτε η συνθήκη του Ore ούτε η συνθήκη του Dirac ισχύουν (είναι ικανές, όχι αναγκαίες).

Ψυχαγωγικά Μαθηματικά 77
Στο Σχήμα 3.10 βλέπουμε το γράφημα του κανονικού οκταέδρου.

Σχήμα 3.10

Το γράφημα αυτό έχει 6 κορυφές και 12 ακμές. Το γράφημα είναι κανονικό, αφού κάθε κορυφή έχει βαθμό 4.
6
Είναι χαμιλτονιανό, αφού ικανοποιεί τη συνθήκη του Dirac (4 > 3 = ). Να ένα χαμιλτονιανό κύκλωμά του:
2
ΑΒ, ΒΓ, ΓΗ, ΗΔ, ΔΕ, ΕΑ.

Στο Σχήμα 3.11 βλέπουμε το γράφημα του κανονικού δωδεκαέδρου.

Σχήμα 3.11
Το γράφημα του κανονικού δωδεκαέδρου έχει 12 κορυφές και 30 ακμές. Το γράφημα είναι κανονικό, αφού
κάθε κορυφή του έχει βαθμό 5. Είναι χαμιλτονιανό. Να ένα χαμιλτονιανό κύκλωμά του: AB, BΓ, ΓΔ, ΔΛ,
ΛΕ, ΕΖ, ΖΜ, ΜΚ, ΚΙ, ΙΘ, ΘΗ, ΗΑ.

3.3 Οι πύργοι του Ανόι


Ας πειραματιστούμε με μικρό αριθμό δίσκων, όπως μας προτείνει ο Polya. Αν ο δίσκος είναι ένας (1), αρκεί
να τον μετακινήσουμε σε μία διαφορετική ράβδο. Ο μικρότερος αριθμός κινήσεων είναι 2 1–1 = 1. Αν οι
δίσκοι είναι δύο (2), μεταφέρουμε τον πάνω δίσκο στη δεύτερη ράβδο, τον κάτω δίσκο στην τρίτη και στη
συνέχεια τοποθετούμε τον δίσκο της δεύτερης ράβδου πάνω από τον δίσκο που βρίσκεται στην τρίτη ράβδο.
Ο μικρότερος αριθμός κινήσεων είναι 22–1 = 3.

Τι συμβαίνει αν οι δίσκοι είναι τρεις (3); Προφανώς, οι δύο μικρότεροι βρίσκονται στη σωστή σειρά,
και για να μεταφερθούν σε κάποια άλλη ράβδο, απαιτούνται, σύμφωνα με όσα είδαμε, τρεις κινήσεις. Με μία
κίνηση μετακινούμε τον μεγαλύτερο σε μια κενή ράβδο, και τώρα με τρεις κινήσεις πάλι μπορούμε να
μετακινήσουμε τους άλλους δύο (που είναι στη σωστή σειρά) πάνω του. Ο μικρότερος αριθμός κινήσεων
είναι 3 + 1 + 3 = 2·3 + 1 = 2·22 – 1 = 23 – 1 = 7.

78 Χατζηκυριάκου Κώστας
Αρχίζουμε να υποψιαζόμαστε ότι ανακαλύψαμε ένα μοτίβο και ότι είναι εύλογο να διατυπώσουμε την
εξής εικασία: Αν οι πύργοι είναι ν, ο μικρότερος αριθμός κινήσεων για να επιτύχουμε τον στόχο μας είναι
2𝜈 –1. Η μαθηματική επικύρωση της ορθότητας της εικασίας μας θα γίνει με τη βοήθεια της μαθηματικής
επαγωγής. Είναι φανερό, από το πέρασμα από τους δύο στους τρεις δίσκους, ότι το επαγωγικό βήμα από 𝜈 σε
𝜈 + 1 ισχύει.

Πράγματι, έστω ότι η εικασία μας αληθεύει για 𝜈 δίσκους. Θα δείξουμε ότι τότε αληθεύει και για 𝜈 +
1 δίσκους. Για να το αποδείξουμε, θα μιμηθούμε το πέρασμα από τους δύο στους τρεις δίσκους. Προφανώς,
οι 𝜈 μικρότεροι βρίσκονται στη σωστή σειρά, και για να μεταφερθούν σε κάποια άλλη ράβδο απαιτούνται,
σύμφωνα με την υπόθεσή μας, κινήσεις 2𝜈 – 1. Με μία κίνηση μετακινούμε τον μεγαλύτερο (τον κατώτερο
της στοίβας) σε μια κενή ράβδο και τώρα με 2𝜈 – 1 κινήσεις, πάλι, μπορούμε να μετακινήσουμε τους άλλους
𝜈 (που είναι στη σωστή σειρά) πάνω του. Ο μικρότερος αριθμός κινήσεων είναι 2𝜈 – 1 + 1 + 2𝜈 – 1 = 2·2𝜈 –
1 = 2𝜈+1 – 1.

Ο τρόπος που κάναμε τις κινήσεις για να επιτύχουμε τον στόχο μας εύλογα μας κάνει να
αναρωτηθούμε αν μπορούμε να αναπαραστήσουμε τις διαδοχικές θέσεις με κάποιο γράφημα, να κάνουμε,
δηλαδή, κάτι σαν κι αυτό που κάναμε στις σπαζοκεφαλιές των περασμάτων του ποταμού.

Να ένας τρόπος αναπαράστασης: Η διατεταγμένη τριάδα (χ, ψ, ω) να δηλώνει ότι ο μικρότερος δίσκος
1 βρίσκεται στη ράβδο χ, ο μεσαίος δίσκος 2 βρίσκεται στη ράβδο ψ και ο μεγαλύτερος δίσκος 3 βρίσκεται
στη ράβδο ω. Οι μεταβλητές χ, ψ, ω παίρνουν τιμές στο σύνολο {α, β, γ}, όπου α η πρώτη ράβδος, β η
δεύτερη ράβδος και γ η τρίτη ράβδος. Έτσι, π.χ. η διατεταγμένη τριάδα (α, α, α) αναπαριστά τη θέση στην
οποία και οι τρεις δίσκοι είναι στην πρώτη ράβδο, η διατεταγμένη τριάδα (γ, γ, γ) αναπαριστά τη θέση στην
οποία και οι τρεις δίσκοι είναι στην τρίτη ράβδο, η διατεταγμένη τριάδα (α, α, β) αναπαριστά τη θέση στην
οποία οι δίσκοι 1, 2 είναι στην πρώτη ράβδο (στη σωστή σειρά) και ο δίσκος 3 είναι στη δεύτερη ράβδο, ενώ
η διατεταγμένη τριάδα (α, α, α) αναπαριστά τη θέση στην οποία και οι τρεις δίσκοι είναι στην πρώτη ράβδο,
ενώ η διατεταγμένη τριάδα (γ, β, α) αναπαριστά τη θέση στην οποία ο δίσκος 1 είναι στην τρίτη ράβδο, ο
δίσκος 2 είναι στη δεύτερη ράβδο και ο δίσκος 3 είναι στην πρώτη ράβδο.

Οι κορυφές του παρακάτω γραφήματος αναπαριστάνουν τις διάφορες δυνατές θέσεις. Οι ακμές
συνδέουν θέσεις που η μία μετασχηματίζεται στην άλλη με μία μόνο κίνηση.

Παρατήρησε (στο Σχήμα 3.12) ότι το γράφημα που προκύπτει είναι χαμιλτονιανό (το χαμιλτονιανό κύκλωμα
αποτελείται από τις ακμές που αναπαριστάνονται διακεκομμένες). Οι 7 κινήσεις που δηλώνονται σε κάθε
πλευρά του ισόπλευρου τριγώνου αναπαριστάνουν τις λύσεις με τον μικρότερο αριθμό βημάτων ανάλογα με
τη ράβδο στην οποία βρίσκονται στην αρχή και οι τρεις δίσκοι.

Σχήμα 3.12

Ψυχαγωγικά Μαθηματικά 79
Προσθέτοντας και άλλους δίσκους είναι φανερό ότι προκύπτουν όμοια μεγαλύτερα τέτοια τρίγωνα με
όμοια εσωτερική δομή. Αυτή η πολύ χαρακτηριστική ιδιότητα της αυτοομοιότητας μας κάνει να
αντιληφθούμε ότι βρισκόμαστε μπροστά στο αρχικό τμήμα ενός φράκταλ, παρόμοιου με το τρίγωνο
Sierpinski. Για τα φράκταλ και το τρίγωνο Sierpinski, βλ. το βιβλίο του Robert L. Devaney, Chaos,
Fractals and Dynamic, Computer Experiments in Mathematics.

3.4 To πυργάκι που τρελαίνει


I. Για να βρούμε τη λύση στην προβληματική μας κατάσταση, θα κατασκευάσουμε ένα μαθηματικό
μοντέλο της προβληματικής μας κατάστασης στο πλαίσιο της θεωρίας γραφημάτων, θα επιλύσουμε εκεί
το μαθηματικό πρόβλημα που προκύπτει και θα επιστρέψουμε στην αρχική μας προβληματική
κατάσταση, εκφράζοντας τη λύση στην καθημερινή γλώσσα στην οποία είναι διατυπωμένη η
σπαζοκεφαλιά.
Η ιδέα είναι να αναπαραστήσουμε κάθε κύβο με ένα γράφημα που θα έχει τέσσερις (όσα και τα
διαφορετικά χρώματα) κορυφές Κ, Μ, Π, Ρ και ακμές ανάμεσα σε κορυφές που αντιστοιχούν σε χρώματα
που βρίσκονται σε απέναντι έδρες του κύβου. Π.χ. στον πρώτο κύβο οι ακμές θα είναι KΠ, ΚΚ, ΜΡ.

Σχήμα 3.13

80 Χατζηκυριάκου Κώστας
Μπορούμε να σχηματίσουμε τώρα ένα γράφημα G που θα αναπαριστά ταυτόχρονα τις παραπάνω
πληροφορίες, δηλαδή ποια χρώματα βρίσκονται σε απέναντι έδρες και στους τέσσερις κύβους.

Σχήμα 3.14
Στο παραπάνω γράφημα βλέπουμε δύο παράγοντα υπογραφήματα του γραφήματος G: το υπoγράφημα G1
({Κ, Μ, Ρ, Π}, {ΚΠ(1), ΚΜ(2), ΜΡ(3), ΡΠ(4)}) και το υπογράφημα G2 ({Κ, Μ, Ρ, Π}, {ΚΠ(4), ΚΜ(3),
ΜΡ(1), ΡΠ(2)}). Ο βαθμός κάθε κορυφής τους είναι 2. Αφού θέλουμε κάθε έδρα του πρίσματός μας να έχει
ακριβώς μία φορά κάθε χρώμα και αφού οι ακμές συνδέουν τα χρώματα που βρίσκονται στις απέναντι έδρες,
μπορούμε να διαβάσουμε μια λύση στο παραπάνω γράφημα, χρωματίζοντας την μπρος και πίσω έδρα του
πρίσματος με τον τρόπο που δηλώνουν οι ακμές του G1 και χρωματίζοντας την αριστερή και τη δεξιά έδρα
του με τον τρόπο που υποδηλώνουν οι ακμές του G2: Βάζουμε τον πρώτο κύβο με την πράσινη έδρα μπροστά
(και άρα την κόκκινη πίσω), τη ροζ αριστερά (και επομένως τη μπλε δεξιά), τον δεύτερο κύβο πάνω στον
πρώτο με την κόκκινη έδρα μπροστά (και άρα τη μπλε πίσω), την πράσινη αριστερά (και τη ροζ δεξιά), τον
τρίτο κύβο πάνω στον δεύτερο με τη μπλε έδρα μπροστά (και άρα τη ροζ πίσω), τη μπλε αριστερά (και την
κόκκινη δεξιά), τον τέταρτο κύβο στην κορυφή με τη ροζ έδρα μπροστά (και άρα την πράσινη πίσω), την
κόκκινη αριστερά (και άρα την πράσινη δεξιά).

Η μοντελοποίηση (οριζόντια μαθηματικοποίηση) που κάναμε μας βοήθησε να λύσουμε


αποτελεσματικά ένα πρόβλημα το οποίο αλλιώς θα μας «τρέλαινε» με την πολυπλοκότητά του, αλλά
επιπροσθέτως μας οδηγεί, μέσω της αναστοχαστικής μελέτης της λύσης, σε ενδιαφέροντα βήματα
κατακόρυφης μαθηματικοποίησης (Βλ. Freudenthal, H. (2002). Revisiting the Mathematics Education, The
China Lectures. Kluwer Academic Publishers).

Το ζητούμενο πρίσμα

Υπογράφημα G1 Υπογράφημα G2

Σχήμα 3.15

Ψυχαγωγικά Μαθηματικά 81
Έστω G ένα γράφημα. Ένα υπογράφημά του θα λέγεται 2-παράγον αν είναι παράγον υπογράφημα του G και
κάθε κορυφή του έχει βαθμό 2. Παραδείγματος χάριν, τα γραφήματα G1, G2 είναι 2-παράγοντα γραφήματα.
Παρατήρησε ότι η έννοια του 2-παράγοντος ενός γραφήματος G (που λόγω του θεωρήματος του Όυλερ δεν
είναι παρά μια ένωση κυκλωμάτων, των οποίων τα σύνολα κορυφών αποτελούν διαμέριση του συνόλου των
κορυφών του G) γενικεύει την έννοια του χαμιλτονιανού κυκλώματος.

Αντίστοιχα, μπορούμε να ορίσουμε ότι ένα υπογράφημα είναι k-παράγον του G, αν είναι παράγον
υπογράφημα του G και κάθε κορυφή έχει βαθμό k (δηλ. είναι κανονικό γράφημα). Ένα γράφημα G λέγεται k-
παραγοντοποιήσιμο αν το σύνολο των ακμών του μπορεί να γραφτεί ως διαμέριση (δηλαδή ένωση ξένων,
μεταξύ τους, συνόλων) των ακμών k-παραγόντων.

Τώρα μπορούμε να πούμε ότι η σπαζοκεφαλιά «πυργάκι που τρελαίνει» λύνεται ανν μπορούμε να
βρούμε δύο 2-παράγοντα υπογραφήματα των οποίων τα σύνολα των ακμών είναι ξένα μεταξύ τους και
καθένα περιέχει τέσσερις ακμές ονοματισμένες 1, 2, 3, 4.

II. Λύνεται παρόμοια. Δείξε ότι σε αυτήν την περίπτωση το γράφημα G που αναπαριστά ταυτόχρονα το
ποια χρώματα βρίσκονται σε απέναντι έδρες και στους τέσσερις κύβους είναι 2-παραγοντοποιήσιμο. Τι
σημαίνει αυτό για τη λύση της σπαζοκεφαλιάς; (Συμβουλέψου, αν θέλεις, το βιβλίο του Liu, Cung Laung,
Elements of Discrete Mathematics, από το οποίο προέρχεται αυτή η σπαζοκεφαλιά).

3.5 Γραφήματα
I. Είναι φανερό ότι όλα τα πλήρη γραφήματα Κν (𝜈 > 2) είναι χαμιλτονιανά.

2∙𝑟
II. Oποιοδήποτε πλήρες διμερές γράφημα Κr,r (𝑟 > 1) είναι χαμιλτονιανό, αφού 𝑟 = (συνθήκη Dirac).
2
Είναι φανερό εδώ ότι η συνθήκη είναι και ικανή (το χαμιλτονιανό κύκλωμα είναι ένα ζιγκ ζαγκ).

III. Έστω χαμιλτονιανό διμερές γράφημα. Ο χαμιλτονιανός κύκλος θα είναι, προφανώς, της μορφής α1, β1,
α2, β2, . . . ακ, βκ, α1. Επομένως, το πλήθος των κορυφών του δεν μπορεί παρά να είναι άρτιο. Άρα, δεν
υπάρχει χαμιλτονιανό διμερές γράφημα με περιττό αριθμό κορυφών.

IV. Το γράφημα του Σχήματος 3.5 είναι διμερές με 13 κορυφές και, άρα, δεν μπορεί να είναι χαμιλτονιανό.

V. To γράφημα του Petersen είναι ημιχαμιλτονιανό, αφού η διαδρομή AE, ΕΔ, ΔΘ, ΘΛ, ΛΞ, ΞΓ, ΓΒ, ΒΝ,
ΝΗ είναι χαμιλτονιανή.

VI. Θα δείξουμε ότι το γράφημα του Petersen δεν είναι χαμιλτονιανό με τη μέθοδο της εις άτοπον
απαγωγής. Έστω ότι υπάρχει κλειστή χαμιλτονιανή διαδρομή κ 1κ2, . . ., κ9,κ10κ1. Προκύπτουν, έτσι,
10 ακμές. Το γράφημά μας έχει όμως 15. Μπορούμε να τις βρούμε; Αν η κ 1 συνδεόταν, μέσω ακμής,
με την κ3 ή την κ9, τότε στο γράφημα του Petersen θα υπήρχε κύκλωμα τριών κορυφών, αλλά τέτοιο
δεν υπάρχει. Καθώς δεν υπάρχει και κύκλωμα τεσσάρων κορυφών, η κ 1 δεν συνδέεται μέσω ακμής
με την κ4 ούτε με την κ 8. Μπορεί, μήπως, να συνδέεται μέσω ακμής με την κ 6 ή την κ7 ή την κ8; Ας
υποθέσουμε ότι συνδέεται έτσι με την κ 6. Τότε είναι εύκολο να δούμε ότι η κ 7 ανήκει σε ένα
κύκλωμα 3 ή 4 κορυφών. Άτοπο. Με άλλα λόγια, η υπόθεση ότι υπάρχει χαμιλτονιανό κύκλωμα,
οδηγεί στην ύπαρξη κυκλώματος 3 ή 4 κορυφών που, φυσικά, δεν υπάρχουν στο γράφημα του
Petersen, αλλά καθώς αυτό δεν αληθεύει, η υπόθεση ότι υπάρχει χαμιλτονιανό κύλωμα οδηγεί σε
άτοπο.

VII. Το γράφημα του Petersen είναι η ένωση ενός 1-παράγοντος (του γραφήματος με ακμές ΑΛ, ΕΗ, ΔΘ, ΓΞ,
ΒΝ) και ενός 2-παράγοντος (του γραφήματος με ακμές ΑΕ, ΕΔ, ΔΓ, ΓΒ, ΒΑ, ΝΗ, ΗΞ, ΞΛ, ΛΘ, ΘΝ). Η
ένωση δυο γραφημάτων είναι ένα τρίτο γράφημα με σύνολο κορυφών την ένωση των συνόλων των

82 Χατζηκυριάκου Κώστας
κορυφών των δύο γραφημάτων και σύνολο ακμών την ένωση των συνόλων των ακμών των δύο
γραφημάτων (εννοείται ότι στα δύο γραφήματα δεν υπάρχουν ακμές με την ίδια ονομασία).

VIII. Στον παρακάτω Πίνακα 3.16 διπλής εισόδου καταγράφονται τα ζητούμενα γραφήματα:

Πίνακας 3.16
Γράφημα Οϋλεριανό Μη-οϋλεριανό
Χαμιλτονιανό Κ3 ({Α, Β, Γ, Δ}, {ΑΒ, ΒΓ, ΓΔ, ΔΑ, ΔΒ})

Μη-χαμιλτονιανό ({Α, Β, Γ, Δ, Ε},{ΑΔ, ΑΕ, ΑΒ, ΑΓ, ΒΓ}) Κ2,3

Ψυχαγωγικά Μαθηματικά 83
Βιβλιογραφία/Αναφορές

Devaney, R. L. (1986). Chaos, Fractals and Dynamic, Computer Experiments in Mathematics. Addison
Wesley Publishinh House.
Freudenthal, H. (2002). Revisiting the Mathematics Education, The China Lectures. Kluwer Academic
Publishers.
Polya, G. (1998). Πώς να το λύσω. Εκδόσεις Καρδαμίτσα.
Wilson, R. J. (1979). Introduction to Graph Theory (2nd edition). Longman.
Χατζηκυριάκου, Κ. (2017). Μαθηματικά για τη Δασκάλα και τον Δάσκαλο, Αριθμοί, Σύνολα, Σχήματα (2η
έκδοση). Εκδόσεις Σοφία.

84 Χατζηκυριάκου Κώστας
Κεφάλαιο 4
Δέντρα και δάση

Σύνοψη
Τα δένδρα, τα δάση και οι βασικές ιδιότητές τους.

Προαπαιτούμενη γνώση
Κεφάλαια 2, 3 του παρόντος συγγράμματος.

Ψυχαγωγικά Μαθηματικά 85
4.1 Τυχαίο;
Στη σπαζοκεφαλιά Ο παχύς και η παχιά του 1ου Κεφαλαίου αναπαραστήσαμε τις δυνατές θέσεις μέσω ενός
γραφήματος που έχει 14 κορυφές και 13 ακμές, δεν έχει κυκλώματα (κλειστές διαδρομές) και οποιεσδήποτε
δύο κορυφές του συνδέονται με ένα μοναδικό μονοπάτι. Είναι οι ιδιότητες αυτές ανεξάρτητες ή σχετίζονται
κάπως μεταξύ τους;

Δηλαδή, υπάρχει γράφημα με 𝜈 κορυφές που να έχει μία από τις τρεις ιδιότητες (ο αριθμός των ακμών
του είναι 𝜈 − 1, απουσία κυκλωμάτων, σύνδεση δύο οποιωνδήποτε κορυφών με ακριβώς ένα μονοπάτι), αλλά
να μην έχει κάποια από τις άλλες δύο;

86 Χατζηκυριάκου Κώστας
4.2 Φρεσκοφυτεμένα δένδρα και δάση
I. Σχεδίασε όλα τα μη ισόμορφα δένδρα με μία, δύο, τρεις, τέσσερις, πέντε, έξι και εφτά κορυφές.

II. Σχεδίασε δάση με τρεις, πέντε, επτά, εννέα, ένδεκα κορυφές.

Ψυχαγωγικά Μαθηματικά 87
4.3 Οι πυροσβεστικοί κρουνοί
Το παρακάτω γράφημα του Σχήματος 4.1 αναπαριστά τη θέση των πυροσβεστικών κρουνών σε μια
κωμόπολη. Οι ακμές δείχνουν τους μόνους δυνατούς τρόπους να ενωθούν μεταξύ τους οι κρουνοί στο δίκτυο
και οι αριθμοί δίπλα σε κάθε ακμή τις αποστάσεις σε μέτρα. Όσο πιο μικρό είναι το μήκος του δικτύου, τόσο
λιγότερο θα κοστίσει. Βρείτε το μικρότερο, σε μήκος, δίκτυο που ενώνει όλους τους κρουνούς.

[Πηγή: Bolt, B. (1990). Μαθηματικές Σπαζοκεφαλιές. Κάτοπτρο].

Σχήμα 4.1

88 Χατζηκυριάκου Κώστας
4.4 Χρωματισμοί (κορυφών)
I. Πόσα διαφορετικά δένδρα με 2 κορυφές υπάρχουν των οποίων οι κορυφές είναι χρωματισμένες με
διαφορετικά χρώματα;

II. Πόσα διαφορετικά δένδρα με 3 κορυφές υπάρχουν των οποίων οι κορυφές είναι χρωματισμένες με
διαφορετικά χρώματα;

III. Πόσα διαφορετικά δένδρα με 4 κορυφές υπάρχουν των οποίων οι κορυφές είναι χρωματισμένες με
διαφορετικά χρώματα;

IV. Πόσα διαφορετικά δένδρα με 5 κορυφές υπάρχουν των οποίων οι κορυφές είναι χρωματισμένες με
διαφορετικά χρώματα;

V. Μήπως τολμάς πια να διατυπώσεις μια εικασία για το πόσα διαφορετικά δένδρα με 𝜈 κορυφές υπάρχουν
των οποίων οι κορυφές είναι χρωματισμένες με διαφορετικά χρώματα; Μπορείς να την αποδείξεις;

Ψυχαγωγικά Μαθηματικά 89
4.5 Και λίγη... Χημεία: Αλκάνια
Τα αλκάνια είναι οργανικές ενώσεις των οποίων ο τύπος είναι CνΗ2ν+2, ν = 1, 2, . . . . Κάθε τέτοια ένωση
μπορούμε να την αναπαραστήσουμε με ένα γράφημα. Αφού το σθένος του άνθρακα (C) είναι 4 και το σθένος
του υδρογόνου (Η) είναι 1, οι κορυφές που αντιστοιχούν σε άνθρακα έχουν βαθμό 4 και οι κορυφές που
αντιστοιχούν σε υδρογόνο έχουν βαθμό 1.

I. Πόσες είναι οι κορυφές στο γράφημα του CνΗ2ν+2; Πόσες είναι οι ακμές;

90 Χατζηκυριάκου Κώστας
II. Να γράψεις τους τύπους των αλκανίων για ν = 1, 2, 3, και να αναπαραστήσεις τις χημικές ενώσεις με
γραφήματα. Τι παρατηρείς;

III. Να γράψεις τους τύπους των αλκανίων για ν = 4, 5, 6.

Ψυχαγωγικά Μαθηματικά 91
4.6 Γραφήματα
I. Δείξε ότι κάθε δένδρο με τουλάχιστον 2 κορυφές είναι διμερές γράφημα. Ποια δένδρα είναι πλήρη
διμερή γραφήματα;

II. Να βρεις παράγον δένδρο για το Κ5, το Κ3,3, για τα πλατωνικά γραφήματα και για το γράφημα του
Petersen.

92 Χατζηκυριάκου Κώστας
Απαντήσεις στα ερωτήματα του 4ου Kεφαλαίου

4.1 Τυχαίο;
Όπως μας συστήνει ο Polya στο βιβλίο του Πώς να το λύσω, ας μελετήσουμε γραφήματα με μικρό αριθμό
κορυφών. Ας δούμε ένα γράφημα με δύο κορυφές που έχει μόνο μία ακμή. Είναι φανερό πως αυτό είναι το
({Α,Β}, {ΑΒ}), δηλαδή είναι απλό, δεν έχει κύκλωμα, και υπάρχει μόνο ένα μονοπάτι (εδώ απλώς ακμή) από
το Α στο Β.

Ας δούμε ένα γράφημα με τρεις κορυφές που έχει δύο ακμές. Είναι φανερό πως πρόκειται για το
μοναδικό γράφημα του οποίου οι δύο κορυφές έχουν βαθμό 1 και η μία έχει βαθμό 2. Το γράφημα αυτό πάλι
είναι απλό, δεν έχει κύκλωμα, και υπάρχει μόνο ένα μονοπάτι από κορυφή σε κορυφή.

Ας δούμε ένα γράφημα με τέσσερις κορυφές που έχει τρεις ακμές. Θα μπορούσε να ήταν το ({Α, Β, Γ,
Δ}, {ΑΒ, ΒΓ, ΓΑ}), το οποίο έχει κύκλωμα αλλά δεν είναι συνεκτικό, δηλαδή δεν υπάρχει ούτε ένα μονοπάτι
από την κορυφή Α στην κορυφή Δ. Αν απαιτήσουμε να είναι συνεκτικό ή να μην έχει κύκλωμα, το γράφημα
αυτό αποκλείεται, και μένουμε με το μοναδικό γράφημα του οποίου δύο κορυφές έχουν βαθμό 1, δύο έχουν
βαθμό 2, και το οποίο πάλι είναι απλό, δεν έχει κύκλωμα, και υπάρχει μόνο ένα μονοπάτι από κορυφή σε
κορυφή.

Ας δούμε ένα γράφημα με πέντε κορυφές που έχει τέσσερις ακμές. Θα μπορούσε να ήταν το ({Α, Β, Γ,
Δ, Ε}, {ΑΒ, ΒΓ, ΓΔ, ΔΑ}), το οποίο έχει κύκλωμα αλλά δεν είναι συνεκτικό, δηλαδή δεν υπάρχει ούτε ένα
μονοπάτι από την κορυφή Α στην κορυφή Ε. Αν απαιτήσουμε να είναι συνεκτικό ή να μην έχει κύκλωμα, το
γράφημα αυτό αποκλείεται. Θα μπορούσε να ήταν το ({Α, Β, Γ, Δ, Ε}, {ΑΒ, ΒΓ, ΓΑ, ΔΕ}). Αν απαιτήσουμε
να είναι συνεκτικό ή να μην έχει κύκλωμα, και το γράφημα αυτό αποκλείεται. Μένουμε με δύο δυνατότητες:
Ένα γράφημα με δύο κορυφές βαθμού 1 και τρεις κορυφές βαθμού 2, το οποίο πάλι είναι απλό, δεν έχει
κύκλωμα, και υπάρχει μόνο ένα μονοπάτι από κορυφή σε κορυφή ή ένα γράφημα με τρεις κορυφές βαθμού 1,
μία κορυφή βαθμού 2 και μία κορυφή βαθμού 3, το οποίο πάλι είναι απλό, δεν έχει κύκλωμα, και υπάρχει
μόνο ένα μονοπάτι από κορυφή σε κορυφή.

Παρατήρησε ότι σε όλες τις περιπτώσεις που έχουμε αποκλείσει την ύπαρξη κυκλώματος, η πρόσθεση
μίας ακμής δημιουργεί ακριβώς ένα κύκλωμα.

Στο σημείο αυτό, ακόμη κι αν είναι τολμηρό, είναι εύλογο να διατυπώσουμε την εξής εικασία: Έστω
γράφημα Τ που έχει 𝜈 κορυφές. Αν αυτό έχει οποιαδήποτε από τις παρακάτω πέντε ιδιότητες, έχει και τις
άλλες τέσσερις:

i. Δεν έχει κυκλώματα και είναι συνεκτικό.


ii. Δεν έχει κυκλώματα και έχει 𝜈 − 1 ακμές.
iii. Είναι συνεκτικό και έχει 𝜈 − 1 ακμές.
iv. Οποιεσδήποτε δύο κορυφές του συνδέονται με ακριβώς ένα μονοπάτι.
v. Δεν έχει κυκλώματα και η προσθήκη μίας μόνο ακμής δημιουργεί ακριβώς ένα κύκλωμα.

Ας δούμε αν η εικασία μας αληθεύει, δηλαδή αν επινοήσαμε (Προτιμάς «ανακαλύψαμε»; Για


σκέψου...) ένα θεώρημα για τη θεωρία των γραφημάτων. Προφανώς αν 𝜈 = 1 ή 2, όλα τα παραπάνω είναι
τετριμμένα αληθή· οπότε, ας υποθέσουμε ότι 𝜈 > 3.

Ψυχαγωγικά Μαθηματικά 93
i ⇒ ii. Θα το δείξουμε επαγωγικά. Έστω γράφημα με 3 κορυφές που δεν έχει κυκλώματα και είναι συνεκτικό.
Είδαμε ήδη ότι αυτό δεν μπορεί παρά να είναι το ({Α, Β, Γ}, {ΑΒ, ΒΓ}), δηλαδή έχει 2 = 3 − 1 ακμές. Έστω
ότι η συνεπαγωγή μας αληθεύει για όλα τα γραφήματα με λιγότερες από 𝜈 κορυφές. Θα δείξουμε ότι
αληθεύει και για όλα τα γραφήματα με 𝜈 κορυφές. Από το τυχόν γράφημά μας Τ, που είναι συνεκτικό, χωρίς
κυκλώματα, αφαιρούμε μία ακμή. Μένουμε τότε με δύο συνεκτικά, χωρίς κυκλώματα, υπογραφήματα Τ1, Τ2
που καθένα έχει μικρότερο αριθμό κορυφών από το Τ και άρα ο αριθμός των ακμών του καθενός είναι ο
προηγούμενος του αριθμού των κορυφών τους. Αλλά τότε, ο αριθμός των ακμών του Τ είναι (αριθμός
κορυφών του Τ1 + αριθμός κορυφών του Τ2) – 2 + 1 = αριθμός κορυφών του Τ – 1.

ii ⇒ iii. Έστω ότι το Τ δεν είναι συνεκτικό. Προφανώς, κάθε συνιστώσα του είναι συνεκτική χωρίς
κυκλώματα. Αλλά τότε, ο αριθμός των ακμών κάθε συνιστώσας είναι ο προηγούμενος του αριθμού των
κορυφών της, οπότε ο συνολικός αριθμός ακμών του γραφήματος είναι το πολύ 𝜈 − 2, άτοπο.

iii ⇒ iv. Αφού είναι συνεκτικό, οποιεσδήποτε δύο κορυφές συνδέονται με ένα τουλάχιστον μονοπάτι. Αν
υπάρχει έστω και μία κορυφή που συνδέεται και με δεύτερη, τότε το γράφημα έχει τουλάχιστον ένα κύκλωμα.
Αφαιρώντας από κάθε κύκλωμα μία ακμή, καταλήγουμε σε ένα γράφημα συνεκτικό, χωρίς κυκλώματα, με
πλήθος ακμών μικρότερο από 𝜈 − 1, άτοπο.

iv ⇒ v. Δεν μπορεί να έχει κύκλωμα, γιατί τότε δύο κορυφές θα συνδέονται με τουλάχιστον δύο μονοπάτια,
άτοπο. Αν προσθέσουμε μία ακμή, τότε τα άκρα της είναι ήδη συνδεδεμένα στο γράφημά μας, οπότε
δημιουργείται κύκλωμα. Γιατί είναι μοναδικό; Αν δημιουργούνταν και άλλο κύκλωμα με αυτήν την ακμή,
τότε θα είχαμε δύο κυκλώματα που θα είχαν κοινή την ακμή που προσθέσαμε. Χρησιμοποιώντας τις άλλες
ακμές των κυκλωμάτων αυτών είναι φανερό ότι μπορούμε να σχηματίσουμε ένα κύκλωμα που δεν περιέχει
την ακμή που προσθέσαμε, επομένως ήδη υπήρχε διπλό μονοπάτι ανάμεσα σε δύο κορυφές, άτοπο.

v ⇒ vi. Αν το γράφημά μας είναι μη συνεκτικό, τότε η πρόσθεση μίας ακμής ανάμεσα σε δύο κορυφές
διαφορετικών συνιστωσών δεν δημιουργεί κύκλωμα, οπότε άτοπο. Αλλά τότε, το γράφημά μας είναι
συνεκτικό, χωρίς κυκλώματα.

Αξίζει να παρατηρήσουμε ότι η παραπάνω αλυσίδα ισοδυναμιών σχηματίστηκε με την εφαρμογή δύο
βασικών μαθηματικών μεθόδων, της μεθόδου της εις άτοπον απαγωγής και της μεθόδου της μαθηματικής
επαγωγής.

Είναι εύλογο να δώσουμε κάποια ονομασία στα γραφήματα που έχουν μία και, επομένως, όλες τις
παραπάνω ιδιότητες. Δεν μοιάζουν με δένδρα; Ας τα πούμε δένδρα. Και ας πούμε δάση τα γραφήματα που
δεν είναι συνεκτικά, αλλά δεν έχουν κυκλώματα (αφού κάθε συνιστώσα τους είναι συνεκτικό και χωρίς
κυκλώματα γράφημα, επομένως δένδρο).

Είναι φανερό, εξαιτίας της ιδιότητας ii, ότι ένα δάσος με 𝜈 κορυφές και 𝜅 συνιστώσες έχει 𝜈 – 𝜅 ακμές.

Παρατήρησε ότι το άθροισμα των βαθμών οποιουδήποτε δένδρου ισούται με 2 ∙ (𝜈 − 1) = 2 ∙ 𝜈 −


2. Επομένως, αν 𝜈 ≥ 2, ένα δένδρο με 𝜈 κορυφές έχει κατ’ ανάγκην τουλάχιστον δύο κορυφές βαθμού 1.

4.2 Φρεσκοφυτεμένα δένδρα και δάση


I. Με 1 κορυφή ένα δένδρο, με 2 κορυφές ένα δένδρο, με 3 κορυφές ένα δένδρο, με 4 κορυφές δύο (2) μη
ισόμορφα δένδρα, με 5 κορυφές τρία (3) μη ισόμορφα δένδρα, με 6 κορυφές έξι (6) μη ισόμορφα δένδρα,
με 7 κορυφές υπάρχουν ένδεκα (11) μη ισόμορφα δένδρα. Τα μη ισόμορφα δένδρα που έχουν ως και 6
κορυφές τα βλέπουμε και στο Σχήμα 4.2.

II. Βάλε πλάι-πλάι το δένδρο με 1 κορυφή και το δένδρο με 2 κορυφές κι έχεις ένα δάσος με τρεις κορυφές.
Βάλε πλάι-πλάι το δένδρο με δύο κορυφές και το δάσος με τρεις κορυφές κι έχεις ένα δάσος με 5

94 Χατζηκυριάκου Κώστας
κορυφές. Βάλε πλάι-πλάι ένα από τα δένδρα με τέσσερις κορυφές και το δένδρο με 3 κορυφές κι έχεις ένα
δάσος με επτά κορυφές ή βάλε πλάι-πλάι ένα δένδρο με πέντε κορυφές και το δένδρο με 2 κορυφές και
πάλι έχεις ένα δάσος με επτά κορυφές (όπως και αν βάλεις πλάι-πλάι ένα δένδρο με 6 κορυφές και το
δένδρο με τη μία κορυφή). Βάλε πλάι-πλάι ένα δένδρο με τέσσερις κορυφές κι ένα δένδρο με πέντε
κορυφές ή ένα δένδρο με τέσσερις κορυφές και τα δένδρα με δύο και τρεις κορυφές κι έχεις ένα δάσος με
εννέα κορυφές. Τέλος, βάλε πλάι-πλάι ένα δένδρο με έξι και ένα δένδρο με πέντε κορυφές ή τα δύο
δένδρα με τέσσερις κορυφές και το ένα δένδρο με τρεις κορυφές κι έχεις ένα δάσος με ένδεκα κορυφές.

Σχήμα 4.2

4.3 Οι πυροσβεστικοί κρουνοί


Αν μελετήσουμε προσεκτικά το πρόβλημα, όπως μας συστήνει ο Polya, θα κατανοήσουμε ότι αναζητάμε ένα
συνεκτικό υπογράφημα του δοσμένου γραφήματος, το οποίο να έχει τις ίδιες 𝜈 κορυφές με εκείνο και να μην
έχει κυκλώματα· επομένως, να είναι δένδρο και, επιπλέον, να είναι το ελαφρύτερο τέτοιο, δηλαδή τα βάρη
των ακμών του να έχουν το δυνατόν μικρότερο άθροισμα.

Ψυχαγωγικά Μαθηματικά 95
Αντιλαμβανόμαστε, έτσι, ότι μπορούμε να καταλήξουμε σε μια ενδιαφέρουσα έννοια ως εξής: Έστω
δοσμένο συνεκτικό γράφημα G. Αν δεν έχει κυκλώματα, τότε είναι δένδρο. Αν έχει, επιλέγουμε ένα από αυτά
και αφαιρούμε μία ακμή του. Το γράφημα που απομένει παραμένει συνεκτικό. Αν δεν έχει κυκλώματα, είναι
δένδρο· αν έχει, επιλέγουμε ένα από αυτά και αφαιρούμε μία ακμή του. Συνεχίζουμε έτσι ώσπου να μην
υπάρχει κύκλωμα, οπότε μένουμε με ένα υπογράφημα Τ του G, το οποίο είναι δένδρο με 𝜈 κορυφές. Ένα
τέτοιο δένδρο Τ λέγεται παράγον δένδρο του G.

Με αυτήν την ορολογία, στη σπαζοκεφαλιά μας αναζητούμε ένα παράγον δένδρο του δοσμένου
γραφήματος που είναι το δυνατόν ελαφρύτερο. Μια απλή στρατηγική επίλυσης θα ήταν να καταστρέψουμε τα
κυκλώματα το ένα μετά το άλλο, αφαιρώντας μία ακμή τους (και μάλιστα τη βαρύτερη), προσέχοντας, όμως,
να μην καταστρέψουμε τη συνεκτικότητα.

Π.χ. στο γράφημα του Σχήματος 4.1, στο κύκλωμα ΒΓ, ΓΙ, ΙΗ, ΗΒ μπορούμε να διαγράψουμε την
ακμή ΒΓ(130), στο κύκλωμα ΓΔ, ΔΙ, ΙΓ να διαγράψουμε την ακμή ΓΔ(100), στο κύκλωμα ΙΔ, ΔΕ, ΕΙ να
διαγράψουμε την ακμή ΔΕ(80), στο κύκλωμα ΙΕ, ΕΖ, ΖΘ, ΘΙ να διαγράψουμε την ακμή ΖΘ(110) και στο
κύκλωμα ΗΙΘ να διαγράψουμε την ακμή ΗΙ(110). Μένουμε με το υπογράφημα ({Α, Β, Γ, Δ, Ε, Ζ, Η, Θ, Ι},
{ΑΒ, ΒΗ, ΗΘ, ΘΙ, ΓΙ, ΙΔ, ΙΕ, ΕΖ}, το οποίο είναι δένδρο και τα βάρη στις ακμές του έχουν άθροισμα 520.
Πώς γνωρίζουμε ότι το δένδρο που βρήκαμε είναι το ελαφρύτερο; Πώς γνωρίζουμε (ακόμη και αν είναι το
ελαφρύτερο) αν υπάρχει κι άλλο;

Υπάρχουν αρκετοί αλγόριθμοι με τους οποίους μπορούμε να βρούμε ένα παράγον δένδρο που να είναι
το δυνατόν ελαφρύτερο. Ένας από αυτούς είναι ο αλγόριθμος του Kruskal: Ξεκίνα από μια ακμή 𝛼1 που
έχει το μικρότερο βάρος. Όρισε τις επόμενες ακμές 𝛼2 , 𝛼3 ,. . ., 𝛼𝜈−1 , επιλέγοντας κάθε φορά μια οποιαδήποτε
ακμή που δεν έχει επιλεγεί προηγουμένως και έχει το μικρότερο δυνατό βάρος χωρίς, όμως, να βρίσκεται σε
κάποιο κύκλωμα με τις ήδη επιλεγμένες. Το υπογράφημα που προκύπτει έτσι είναι ένα δένδρο Δ με 𝜈
κορυφές, του οποίου τα βάρη των ακμών έχουν το μικρότερο δυνατό άθροισμα.

Γιατί; Είναι φανερό ότι το υπογράφημα με τις ακμές αυτές δεν περιέχει κυκλώματα, έχει 𝜈 κορυφές και
𝜈 − 1 ακμές, άρα είναι δένδρο. Θα δείξουμε με τη μέθοδο της εις άτοπον απαγωγής ότι τα βάρη των ακμών
του έχουν το μικρότερο δυνατό άθροισμα. Ας υποθέσουμε ότι υπάρχει παράγον δένδρο Τ, του οποίου τα
βάρη των ακμών του έχουν άθροισμα μικρότερο εκείνων του Δ. Έστω 𝛼𝜅 η πρώτη από τις ακμές του Δ που
δεν ανήκει στο Τ. Από τον ορισμό του δένδρου, γνωρίζουμε ότι αν προσθέσουμε την 𝛼𝜅 στο Τ, τότε
παράγεται ένα μοναδικό κύκλωμα Κ που την περιέχει ως ακμή. Προφανώς, στο Κ υπάρχει μια ακμή 𝛼 που
δεν ανήκει στο Δ και αν αντικαταστήσουμε την 𝛼𝜅 με αυτήν στο Τ, το υπογράφημα Τ1 που προκύπτει έτσι
είναι πάλι παράγον δένδρο του G. Αλλά από τον τρόπο κατασκευής, το βάρος της 𝛼𝜅 είναι μικρότερο ή ίσο
του βάρους της 𝛼, το Τ1 είναι ελαφρύτερο ή ισοβαρές του Τ και έχει μια ακμή ακόμη κοινή με το Δ πέρα από
εκείνες που το τελευταίο έχει κοινές με το Τ. Επαναλαμβάνοντας την παραπάνω διαδικασία όσες φορές
γίνεται, εντέλει μετατρέπουμε το Τ σε Δ, δείχνοντας ταυτόχρονα ότι τα βάρη των ακμών του Δ έχουν
άθροισμα μικρότερο ή ίσο του αθροίσματος των βαρών του Τ. Άτοπο.

Ποια λύση θα έδινε στη σπαζοκεφαλιά μας ο αλγόριθμος του Kruskal; Ξεκινώντας από το μικρότερο
βάρος (50), βάζουμε στο δένδρο μας τις ακμές ΒΗ, ΙΕ, ΕΖ, κατόπιν βάζουμε τις ΑΒ, ΙΔ (60), κατόπιν την ΗΘ
(70), δεν μπορούμε να βάλουμε τη ΔΕ (80) (σχηματίζει κύκλωμα με τις ΙΔ και ΙΕ), μπορούμε, όμως, να
βάλουμε τη ΓΙ (80), δεν μπορούμε να βάλουμε την ΑΘ (90) (σχηματίζει κύκλωμα με τις ΑΒ, ΒΗ, ΗΘ) ούτε
τη ΓΔ (100) (σχηματίζει κύκλωμα με τις ΓΙ, ΙΔ), μπορούμε, όμως, να βάλουμε την ΙΘ (100). Επομένως,
έχουμε πάλι το δένδρο ΑΒ, ΒΗ, ΗΘ, ΘΙ, ΙΓ, ΙΔ, ΙΕ, ΕΖ, του οποίου οι ακμές έχουν άθροισμα 520. Τώρα,
όμως, γνωρίζουμε ότι το παράγον δένδρο που βρήκαμε έχει το μικρότερο δυνατό βάρος.

Να ακόμη ένας αποτελεσματικός αλγόριθμος (αντίστροφης διαγραφής) για την επίλυση του
προβλήματος. Βάλε σε φθίνουσα σειρά (ως προς το βάρος) όλες τις ακμές του γραφήματος. Διάγραψε τη
βαρύτερη και συνέχισε με τον ίδιο τρόπο προσέχοντας να μην καταργείται η συνεκτικότητα του γραφήματος.
Σταμάτα όταν απομένουν κατά μία λιγότερες από τις κορυφές.

96 Χατζηκυριάκου Κώστας
Ποια λύση θα πάρουμε στη σπαζοκεφαλιά μας με αυτόν τον τρόπο; Ταξινόμηση των ακμών κατά
φθίνον βάρος: ΒΓ(130), ΘΖ, ΗΙ(110), ΓΔ, ΘΙ(100), ΑΘ(90), ΓΙ, ΔΕ(80), ΗΘ(60), ΑΒ, ΙΔ(50), ΗΒ, ΙΕ,
ΕΖ(50). Επομένως, σβήνουμε ΒΓ, ΘΖ, ΗΙ, δεν μπορούμε να διαγράψουμε την ΙΘ (χάνεται η συνεκτικότητα),
διαγράφουμε τις ΓΔ, ΔΘ, δεν μπορούμε να διαγράψουμε τη ΓΙ (χάνεται η συνεκτικότητα), διαγράφουμε τη
ΔΕ και κρατάμε τις ΗΘ, ΙΔ, ΑΒ, ΒΗ, ΙΕ, ΕΖ. Τώρα, έχουμε το παράγον δένδρο ΑΒ, ΒΗ, ΗΘ, ΘΙ, ΙΓ, ΙΔ, ΙΕ,
ΕΖ. Βρίσκουμε πάλι το ίδιο δένδρο.

Οι δύο τελευταίοι αλγόριθμοι είναι «λαίμαργοι», «άπληστοι» (“greedy”), δηλαδή είναι αλγόριθμοι
που κάνουν τοπικά την καλύτερη δυνατή επιλογή. Προφανώς, χρησιμοποιώντας τον έναν ή τον άλλον και
υποθέτοντας ότι όλες οι ακμές είναι ισοβαρείς, μπορούμε να βρούμε παράγον δένδρο για οποιοδήποτε
συνεκτικό γράφημα.

4.4 Χρωματισμοί (κορυφών)


I. Υπάρχει μόνο ένα δένδρο με 2 κορυφές. Επομένως, υπάρχει μόνο ένα (1) χρωματισμένο δένδρο με 2
χρώματα, η μία κορυφή, ας πούμε, είναι μαύρη και η άλλη κόκκινη. Είναι το δένδρο (Μ, Κ).

II. Υπάρχει μόνο ένα δένδρο με 3 κορυφές. Επομένως, υπάρχουν μόνο τρία (3) διαφορετικά δένδρα με τις
κορυφές τους χρωματισμένες με τρία διαφορετικά χρώματα, μαύρο, κόκκινο, πράσινο. Είναι τα (Μ, Κ,
Π), (Κ, Μ, Π) και (Π, Κ, Μ).

III. Υπάρχουν μόνο δύο δένδρα με 4 κορυφές. Το ένα έχει δύο κορυφές βαθμού 1 και δύο κορυφές βαθμού 2,
το άλλο είναι ο αστερίας (μία κορυφή του έχει βαθμό 3 και οι άλλες έχουν βαθμό 1). Οι διαφορετικοί
χρωματισμοί του πρώτου με τέσσερα χρώματα, προφανώς αντιστοιχούν στις μισές διαφορετικές διατάξεις
του συνόλου {1, 2, 3, 4}, αφού καθεμία από τις δύο κορυφές βαθμού 1 μπορεί να θεωρηθεί «ρίζα» του
δένδρου (το χρωματισμένο δένδρο 1, 2, 3, 4 δεν διαφέρει από το χρωματισμένο δένδρο 4, 2, 3, 1). Έτσι,
4! 4∙3∙2∙1
παίρνουμε δώδεκα (12) διαφορετικά χρωματισμένα δένδρα ( 2 = ). Τα διαφορετικά χρωματισμένα
2
δένδρα του δεύτερου είδους είναι τα τέσσερα (4) στα οποία η μοναδική κορυφή βαθμού 3 έχει ένα από τα
διαφορετικά τέσσερα χρώματα. Σύνολο, δεκαέξι (16) διαφορετικά χρωματισμένα δένδρα με 4 κορυφές.
Στο σημείο αυτό, είναι δύσκολο να μην παρατηρήσουμε ότι 16 = 4 2 = 44-2 και ότι 3 =31 =33-2 και 1 =
2 . Είναι εύλογο, ακολουθώντας και τον Polya, να αναρωτηθούμε μήπως το πλήθος των διαφορετικών
2-2

δένδρων με διαφορετικά χρωματισμένες κορυφές ακολουθεί κάποιο μοτίβο. Υπάρχουν, άραγε, 125 = 53 =
55-2 διαφορετικά δένδρα των οποίων οι πέντε κορυφές είναι χρωματισμένες με διαφορετικά χρώματα;

IV. Υπάρχουν μόνο τρία δένδρα με 5 κορυφές. Το πρώτο έχει δύο κορυφές βαθμού 1 και τρεις κορυφές
βαθμού 2, το δεύτερο έχει τρεις κορυφές βαθμού 1, μία κορυφή βαθμού 2 και μία κορυφή βαθμού 3, ενώ
το τρίτο είναι ο αστερίας (μία κορυφή του έχει βαθμό 4 και οι άλλες έχουν βαθμό 1). Οι διαφορετικοί
χρωματισμοί του πρώτου με πέντε χρώματα, προφανώς αντιστοιχούν στις μισές διαφορετικές διατάξεις
του συνόλου {1, 2, 3, 4, 5}, αφού καθεμία από τις δύο κορυφές βαθμού 1 μπορεί να θεωρηθεί «ρίζα» του
δένδρου (το χρωματισμένο δένδρο 1, 2, 3, 4, 5 δεν διαφέρει από το χρωματισμένο δένδρο 5, 4, 2, 3, 1).
5! 5∙4∙3∙2∙1
Έτσι παίρνουμε δώδεκα (60) διαφορετικά χρωματισμένα δένδρα ( 2 = ). Για να δούμε πόσα είναι
2
τα διαφορετικά χρωματισμένα δένδρα του δεύτερου είδους, μπορούμε να σκεφτούμε ως εξής: Υπάρχουν
τρεις εν σειρά συνδεδεμένες κορυφές και μία διχάλα. Για να χρωματίσουμε τις δύο κορυφές της διχάλας,
πρέπει να επιλέξουμε δύο από τα πέντε χρώματα. Με πόσους διαφορετικούς τρόπους μπορούμε να το
5∙4
κάνουμε αυτό; Με 5Σ2 διαφορετικούς τρόπους, δηλαδή με 2 =10 διαφορετικούς τρόπους. Για καθέναν
από αυτούς, υπάρχουν 3∙2∙1 διαφορετικοί χρωματισμοί των άλλων τριών κορυφών. Σύνολο, εξήντα (60)
διαφορετικά χρωματισμένα δένδρα με 4 κορυφές δεύτερου είδους. Τα διαφορετικά χρωματισμένα
δένδρα του τρίτου είδους είναι τα πέντε (5) στα οποία η μοναδική κορυφή βαθμού 4 έχει ένα από τα
διαφορετικά πέντε χρώματα. Επομένως, υπάρχουν συνολικά εκατόν είκοσι πέντε (125) διαφορετικά

Ψυχαγωγικά Μαθηματικά 97
χρωματισμένα δένδρα με 5 κορυφές.

V. Μπορούμε πια να αποτολμήσουμε μια εικασία: Υπάρχουν 𝝂𝝂−𝟐 διαφορετικά δένδρα με 𝝂 κορυφές οι
οποίες είναι χρωματισμένες με 𝝂 διαφορετικά χρώματα (𝝂 > 1). Αληθεύει άραγε;
Προφανώς θα αληθεύει, αν κατορθώσουμε να δείξουμε ότι υπάρχουν τόσα δένδρα σαν αυτά που
ψάχνουμε όσες είναι οι πεπερασμένες αριθμητικές ακολουθίες 𝛼: {0, 1, 2, . . . 𝜈 − 2} → {0, 1, 2, . . . , 𝜈}, το
πλήθος των οποίων είναι 𝜈 𝜈−2 . Μπορούμε να υποθέσουμε, μάλιστα, ότι 𝜈 > 5.

Θα δείξουμε ότι σε κάθε δένδρο Τ με 𝜈 κορυφές που είναι χρωματισμένες με 𝜈 διαφορετικά χρώματα
αντιστοιχεί ακριβώς μια τέτοια ακολουθία 𝛼Τ = (𝛼1 , 𝛼2 , . . . 𝛼ν−2 ), όπου 1≤ 𝛼𝑖 ≤ 𝜈, με 𝑖 = 1, 2,. . ., 𝜈 − 2.
Έστω ένα χρωματισμένο, με αυτόν τον τρόπο, δένδρο. Πήγαινε στην κορυφή βαθμού 1 που έχει τον
μικρότερο αριθμό· 𝛼1 θα είναι ο αριθμός που έχει η γειτονική κορυφή της (το άλλο άκρο της ακμής στην
οποία ανήκει). Διάγραψε τώρα την ακμή αυτή στο δένδρο, μένεις με ένα δένδρο με μία ακμή λιγότερη.
Πήγαινε στην κορυφή βαθμού 1 αυτού του δένδρου που έχει τον μικρότερο αριθμό· 𝛼2 θα είναι ο αριθμός
που έχει η γειτονική κορυφή της (το άλλο άκρο της ακμής στην οποία ανήκει). Διάγραψε τώρα την ακμή
αυτή στο δένδρο, μένεις με ένα δένδρο με μία ακμή λιγότερη. Εξακολούθησε με αυτόν τον τρόπο ώσπου
να μείνεις με ένα δένδρο 2 κορυφών.

Αντίστροφα, έστω ακολουθία 𝛼 = (𝛼1 , 𝛼2 , . . . 𝛼ν−2 ), όπου 1≤ 𝛼𝑖 ≤ 𝜈, με 𝛼𝑖 = 1, 2,. . ., 𝜈. Θα


ορίσουμε, μέσω αυτής, ένα δένδρο Τα ως εξής: Έστω β1 ο μικρότερος φυσικός που δεν είναι όρος της
ακολουθίας. Μια ακμή του δένδρου μας είναι η β1 α1 . Αγνόησε τώρα τα β1 , α1 και επανάλαβε τη
διαδικασία. Δηλαδή, έστω β2 ≠ β1 ο μικρότερος φυσικός που δεν είναι όρος της ακολουθίας (α2 , . . .
αν−2 ). Μια άλλη ακμή του δένδρου μας είναι η β2 α2 . Αγνόησε τώρα τα β1 , α1 , β2 , α2 και επανάλαβε τη
διαδικασία. Δηλαδή, έστω β3 ≠ β1 , β2 ο μικρότερος φυσικός που δεν είναι όρος της ακολουθίας (α2 , . . .
αν−2 ). Συνέχισε ώσπου να τελειώσουν όλοι οι όροι της ακολουθίας. Είναι φανερό ότι αν εφαρμόσουμε τη
διαδικασία αυτή σε μια ακολουθία 𝛼 που προέκυψε από ένα ταμπελαρισμένο δένδρο Τ μέσω της
προηγούμενης διαδικασίας, τότε το Τα θα είναι το Τ.

Την εικασία που μόλις αποδείξαμε (ακολουθώντας τη μία από τις δύο αποδείξεις της που δίνει ο R. J.
Wilson στο βιβλίο του Introduction to Graph Theory) απέδειξε για πρώτη φορά το 1889 ο Arthur Cayley
(1821-1895).

Παράδειγμα:

Σχήμα 4.3

98 Χατζηκυριάκου Κώστας
Ξεκινάμε από την κορυφή με τον αριθμό 1, άρα 𝛼1 = 6. Διαγράφοντας την ακμή 16 και επαναλαμβάνοντας
την ίδια διαδικασία, πηγαίνουμε στην κορυφή 2, οπότε 𝛼2 =7. Διαγράφουμε την ακμή 27 και
επαναλαμβάνοντας την ίδια διαδικασία πηγαίνουμε στην κορυφή 3, οπότε 𝛼3 = 5. Συνεχίζοντας έτσι
βρίσκουμε 𝛼4 = 5, 𝛼5 = 6. Οπότε, στο παραπάνω ταμπελαρισμένο δένδρο αντιστοιχεί η ακολουθία (6, 7, 5,
5, 6).

Για να δούμε τώρα, ποιο δένδρο με εφτά κορυφές {1, 2, 3, 4, 5, 6, 7} αντιστοιχεί στην ακολουθία (6, 7,
5, 5, 6). Ο μικρότερος αριθμός που δεν εμφανίζεται στην ακολουθία είναι ο 1. Άρα, μια ακμή είναι η 16. Ο
μικρότερος αριθμός που αντιστοιχεί στο δεύτερο βήμα είναι ο 2, άρα μια άλλη ακμή είναι η 27. Οι
μικρότεροι αριθμοί που αντιστοιχούν στα επόμενα βήματα είναι κατά σειρά οι 3 (ακμή 35), 4 (ακμή 45), 5
(ακμή 56). Ενώνουμε τις κορυφές 6 και 7 για να σχηματίσουμε και την τελευταία ακμή 67. Βρήκαμε το
δένδρο από το οποίο ξεκινήσαμε.

Διαδικτυακή δραστηριότητα: Συμβουλέψου έναν έγκυρο ιστότοπο και γράψε ένα σύντομο βιογραφικό
σημείωμα για τον Arthur Cayley.

4.5 Και λίγη… Χημεία: Αλκάνια


I. Πλήθος κορυφών στο γράφημα του CνΗ2ν+2 = 3∙ν+2. Άθροισμα βαθμών = 4∙ν + 2∙ν +2 = 6∙ν +2. Άρα,
πλήθος ακμών 3∙ν +1 (χάρις στο λήμμα των χειραψιών). Το γράφημα είναι συνεκτικό, άρα είναι δένδρο.

II. CH3, C2H6, C3H8. Και στο Σχήμα 4.4 φαίνεται ότι όλα τα γραφήματα είναι δένδρα. Η δομή τους
καθορίζεται από τον τρόπο που συνδέονται τα C σε δένδρο.

III. C4H10, C5H12. Υπάρχουν δύο δένδρα με 4 κορυφές και τρία δένδρα με 5 κορυφές. Επομένως, υπάρχουν
δύο διαφορετικά αλκάνια με τύπο C4H10 και πέντε διαφορετικά αλκάνια με τύπο C5H12.
Στα δύο δένδρα με 4 κορυφές, ονομάζουμε τις κορυφές αυτές με C και προσθέτουμε τα δέκα Η έτσι
ώστε οι C-κορυφές να έχουν βαθμό 4. Παίρνουμε, έτσι, το βουτάνιο και το μεθυλοπροπάνιο.

Στα τρία δένδρα με 5 κορυφές, ονομάζουμε τις κορυφές αυτές με C και προσθέτουμε τα δώδεκα Η έτσι
ώστε οι C-κορυφές να έχουν βαθμό 4. Παίρνουμε, έτσι, το πεντάνιο, το μεθυλοβουτάνιο και το
διμεθυλοπροπάνιο.

Το 1875 ο Arthur Cayley κατόρθωσε να μετρήσει το πλήθος των αλκανίων.

Σχήμα 4.4

Ψυχαγωγικά Μαθηματικά 99
4.6 Γραφήματα
I. Έστω το δένδρο με 2 κορυφές που έχει μία ακμή. Προφανώς, είναι διμερές γράφημα. Έστω ότι κάθε
δένδρο με 𝜈 − 1 ακμές (𝜈 > 1) είναι διμερές γράφημα. Έστω, τώρα, οποιοδήποτε δένδρο Δ με 𝜈 ακμές.
Πήγαινε σε μια οποιαδήποτε κορυφή του 𝜅𝜈+1 με βαθμό 1 και δες την ακμή 𝑎 = 𝜅𝜈 𝜅𝜈+1 . Το γράφημα
χωρίς την ακμή αυτή είναι δένδρο Δ−{𝑎} και έχοντας 𝜈 − 1 ακμές είναι διμερές, άρα το σύνολο των
κορυφών του Κ = Κ0 ∪ Κ1 και κάθε ακμή του έχει το ένα άκρο της στο Κ 0 και το άλλο στο Κ1. Έστω ότι
η κορυφή 𝜅 ανήκει στο σύνολο Κ0. Τότε, Κ0 ∪ (Κ1 ∪ {𝜅𝜈+1 }) είναι η διαμέριση των κορυφών του Δ, για
την οποία κάθε ακμή του Δ έχει το ένα άκρο της στο σύνολο Κ 0 και το άλλο στο σύνολο Κ1 ∪ {𝜅𝜈+1 }.
Για να είναι ένα δένδρο με τουλάχιστον δύο κορυφές πλήρες διμερές, πρέπει οι πληθικοί αριθμοί των
δύο συνόλων της διαμέρισης του συνόλου των κορυφών του 𝑟, 𝑠 να ικανοποιούν τις σχέσεις 𝑟 + 𝑠 =
𝜈 και 𝑟 ∙ 𝑠 = 𝜈 − 1, όπου 𝜈 είναι το πλήθος των κορυφών του δένδρου. Αξίζει να παρατηρήσουμε εδώ,
και πάλι έχοντας στον νου μας τη μέθοδο επίλυσης προβλήματος που μας προτείνει ο Polya, ότι η επίλυση
του προβλήματος έχει αναχθεί στην επίλυση ενός κλασικού μαθηματικού προβλήματος που συναντάμε
ήδη στα Βαβυλωνιακά μαθηματικά: Να βρεθούν οι διαστάσεις ενός αγρού σε σχήμα ορθογωνίου
παραλληλογράμμου, του οποίου γνωρίζουμε την περίμετρο και το εμβαδόν, δηλαδή να βρεθούν οι δύο
αριθμοί των οποίων γνωρίζουμε το άθροισμα και το γινόμενο.

Γνωρίζουμε, λοιπόν, ότι οι αριθμοί 𝑟, 𝑠 είναι οι ρίζες της δευτεροβάθμιας εξίσωσης 𝑥 2 − 𝜈 ∙ 𝑥 +


ν±√𝜈2 −4∙1∙(𝜈−1) ν±(ν −1)
(𝜈 − 1) = 0, οι ρίζες της οποίας δίνονται από τον τύπο 𝑥 = = . Οπότε, η μία ρίζα
2 2
είναι η 1 και η άλλη η 𝜈 − 1. Άρα, για κάθε δένδρο με τουλάχιστον δύο κορυφές, πλήρες διμερές είναι το
δένδρο που είναι γνωστό ως αστερίας.

Ωστόσο, αξίζει και πάλι να αναρωτηθούμε, ακολουθώντας τον Polya, αν μπορούμε να βρούμε αυτό
το αποτέλεσμα και αλλιώς, χωρίς να κάνουμε χρήση της δευτεροβάθμιας εξίσωσης.

Πράγματι, είναι φανερό ότι 𝑟 ∙ 𝑠 = 𝑟 + 𝑠 − 1, οπότε 𝑟 ∙ (𝑠 − 1) = 𝑠 − 1. Οπότε αν 𝑠 = 1, τότε 𝑟 =


𝜈 − 1 και αν 𝑠 ≠ 1, τότε 𝑟 = 1, και επομένως 𝑠 = 𝜈 − 1.

II. Ένα παράγον δένδρο για το K5 στο Σχήμα 4.5 είναι το δένδρο με ακμές ΑΕ, ΕΔ, ΔΓ, ΓΒ.

Σχήμα 4.5

100 Χατζηκυριάκου Κώστας


Ένα παράγον δένδρο για το Κ3,3 στο Σχήμα 4.6 είναι το δένδρο με ακμές ΑΕ, ΕΓ, ΓΖ, ΖΒ, ΒΔ.

Σχήμα 4.6

Ένα παράγον δένδρο για το Π4 στο Σχήμα 4.7 είναι το δένδρο με ακμές τις ΑΕ, ΕΒ, ΕΓ.

Σχήμα 4.8
Σχήμα 4.7

Ένα παράγον δένδρο για το Π6 στο Σχήμα 4.8 είναι το δένδρο με ακμές ΑΒ, ΒΗ, ΗΘ, ΘΓ, ΓΔ, ΔΕ,
ΕΖ.

Σχήμα 4.8

Ψυχαγωγικά Μαθηματικά 101


Ένα παράγον δένδρο για το Π8 στο Σχήμα 4.9 είναι το δένδρο με ακμές ΑΒ, ΒΔ, ΔΕ, ΕΗ, ΗΓ.

Σχήμα 4.9

Ένα παράγον δένδρο για το Π12 στο Σχήμα 4.10 είναι το δένδρο με ακμές ΑΒ, ΒΓ, ΓΔ, ΔΕ, ΕΗ, ΗΛ,
ΛΖ, ΖΟ, ΟΚ, ΚΞ, ΞΙ, ΙΝ, ΝΘ, ΘΜ, ΜΣ, ΣΤ, ΤΥ, ΥΠ, ΠΡ.

Σχήμα 4.10

102 Χατζηκυριάκου Κώστας


Ένα παράγον δένδρο για το Π20 στο Σχήμα 4.11 είναι το δένδρο με ακμές τις ΑΒ, ΒΓ, ΓΔ, ΔΕ, ΕΖ,
ΖΗ, ΗΘ, ΘΙ, ΙΛ, ΛΜ, ΜΚ.

Σχήμα 4.11

Ένα παράγον δένδρο για το γράφημα του Petersen στο Σχήμα 4.12 είναι το δένδρο με ακμές ΑΕ, ΕΔ,
ΔΓ, ΓΒ, ΒΝ, ΝΗ, ΗΞ, ΞΛ, ΛΘ.

Σχήμα 4.12

Ψυχαγωγικά Μαθηματικά 103


Βιβλιογραφία/Αναφορές

Polya, G. (1998). Πώς να το λύσω. Εκδόσεις Καρδαμίτσα.


Wilson, R. J. (1979). Introduction to Graph Theory (2nd edition). Longman.

104 Χατζηκυριάκου Κώστας


Κεφάλαιο 5
Ξεμπλέκουν οι ακμές τους;

Σύνοψη
Τα επίπεδα γραφήματα και οι ιδιότητές τους. Τα πέντε πλατωνικά στερεά.

Προαπαιτούμενη γνώση
Κεφάλαια 2, 3, 4 του παρόντος συγγράμματος.

Ψυχαγωγικά Μαθηματικά 105


5.1 Γείτονες που αντιπαθιούνται
Τρία γειτονικά νοικοκυριά χρησιμοποιούν το ίδιο πηγάδι, την ίδια πετρελαιοπηγή και την ίδια αποθήκη
μελάσας. Δυστυχώς, οι γείτονες αντιπαθούν τόσο πολύ ο ένας τον άλλον ώστε, για να αποφεύγουν να
συναντιούνται, θέλουν να χαράξουν μονοπάτια προς το πηγάδι, την πετρελαιοπηγή και την αποθήκη τα οποία
να μην τέμνονται. Μπορεί να γίνει αυτό; Ναι ή όχι, γιατί;

106 Χατζηκυριάκου Κώστας


5.2 Από τον Πλάτωνα στον Όυλερ και πίσω
I. Και τα πέντε πλατωνικά γραφήματα που είδαμε είναι επίπεδα. Στην επίπεδη αναπαράσταση καθενός από
αυτά διακρίνουμε περιοχές, δηλαδή επίπεδα τμήματα στα οποία τα οποιαδήποτε δύο σημεία τους
μπορούν να συνδεθούν με μια (αδιάσπαστη) γραμμή που δεν τέμνει καμιά ακμή του γραφήματος. Π.χ. το
επίπεδο γράφημα του κανονικού τετραέδρου έχει 4 κορυφές, 6 ακμές και 4 περιοχές (τρεις εσωτερικές
και μία εξωτερική).
Μέτρησε τις ακμές, τις κορυφές και τις περιοχές και των υπόλοιπων πλατωνικών γραφημάτων. Τι
παρατηρείς;

Ψυχαγωγικά Μαθηματικά 107


II. Μέτρησε τις κορυφές, τις ακμές και τις περιοχές των γραφημάτων Κ 1, Κ2, Κ3. Τι παρατηρείς;

108 Χατζηκυριάκου Κώστας


III. Μέτρησε τις κορυφές, τις ακμές και τις περιοχές του γραφήματος που συνδέεται με το πρόβλημα των
εφτά γεφυρών του Κένινγκσμπεργκ.

IV. Μέτρησε τις κορυφές, τις ακμές και τις περιοχές οποιουδήποτε δένδρου.

V. Πιστεύεις ότι οι όποιες παρατηρήσεις σου στα προηγούμενα ερωτήματα Ι έως IV μπορούν να οδηγήσουν
στη διατύπωση κάποιας εικασίας; Αν ναι, σε ποια; Μπορείς να την αποδείξεις;

Ψυχαγωγικά Μαθηματικά 109


VI. Δείξε ότι τα μόνα κανονικά κυρτά πολύεδρα είναι τα πέντε πλατωνικά στερεά.

110 Χατζηκυριάκου Κώστας


VII. Διάβασε τα αποσπάσματα 53c-56e του πλατωνικού Τίμαιου για να δεις τη σχέση των πέντε κανονικών
κυρτών πολυέδρων με τα τέσσερα στοιχεία και το σύμπαν και το πώς υπεισέρχονται στη δημιουργία του
κόσμου. Στη συνέχεια, σε μια εγκυκλοπαίδεια ή έγκυρο ιστότοπο μελέτησε πώς ο Κέπλερ χρησιμοποιεί
τα πέντε πλατωνικά στερεά στο έργο του Mysterium cosmographicum (1596).

Ψυχαγωγικά Μαθηματικά 111


5.3 To K3,3, το Κ5 και όλα τα μη επίπεδα γραφήματα
I. Δείξε ότι σε κάθε συνεκτικό, επίπεδο απλό γράφημα με τουλάχιστον 3 κορυφές ισχύει η ανισότητα:
Α ≤ 3∙Κ – 6.

II. Δείξε ότι το K3,3 δεν είναι επίπεδο γράφημα.

III. Δείξε ότι το K5 δεν είναι επίπεδο γράφημα.

112 Χατζηκυριάκου Κώστας


IV. Δείξε ότι τα μόνα επίπεδα πλήρη γραφήματα είναXι τα Κ1, Κ2, Κ3, Κ4.

V. Δείξε ότι το γράφημα του Petersen δεν είναι επίπεδο.

Ψυχαγωγικά Μαθηματικά 113


Απαντήσεις στα ερωτήματα του 5ου Kεφαλαίου

5.1 Γείτονες που αντιπαθιούνται


Ας αναπαραστήσουμε και πάλι την προβληματική κατάστασή μας με ένα γράφημα. Οι κορυφές του ας είναι
τα τρία νοικοκυριά και οι τρεις εφοδιαστικοί προορισμοί τους. Οι ακμές του τα μονοπάτια από κάθε
νοικοκυριό σε κάθε σταθμό. Ούτε οι τρεις προορισμοί ούτε τα τρία νοικοκυριά συνδέονται με μονοπάτια,
αφού έτσι θα υπήρχε περίπτωση οι γείτονες να συναντηθούν διασχίζοντάς τα. Το γράφημα που προκύπτει,
έτσι, είναι φυσικά το Κ3,3. Και το πρόβλημα που έχουμε να λύσουμε μετατρέπεται στο εξής: Είναι δυνατόν να
αναπαραστήσουμε το Κ3,3 στο επίπεδο έτσι ώστε οι ακμές του να μην τέμνονται; Κάνοντας μία-δύο δοκιμές
(όπως θα μας πρότεινε και ο Polya στο Πώς να το λύσω), είναι δύσκολο να μην αποκτήσουμε την αίσθηση ότι
κάτι τέτοιο δεν μπορεί να γίνει. Μπορούμε να δώσουμε μια «βαθιά», δομική εξήγηση γιατί πρέπει να
συμβαίνει αυτό;

Ας ονομάσουμε επίπεδο ένα γράφημα που μπορεί να σχεδιαστεί στο επίπεδο έτσι ώστε οι ακμές του να
μην τέμνονται παρά μόνο σε κορυφές του γραφήματος.

Το ερώτημά μας έτσι γίνεται: Είναι το Κ 3,3 επίπεδο γράφημα; Υποψιαζόμαστε πως όχι. Αλλά πώς θα το
αποδείξουμε; Τι συμβαίνει, άραγε, με άλλα γνωστά μας γραφήματα; Για παράδειγμα, ποια πλήρη γραφήματα
είναι επίπεδα; Προφανώς, τα Κ1, Κ2, Κ3 είναι επίπεδα. Εύκολα βλέπουμε ότι και το Κ 4 (που συνήθως το
αναπαριστάνουμε με ένα τετράγωνο με τις διαγωνίους του να τέμνονται) είναι επίπεδο: είναι ισόμορφο με το
γράφημα του κανονικού τετραέδρου. Είναι το Κ 5 επίπεδο γράφημα; Και πάλι μετά από λίγες δοκιμές
επανασχεδιασμού του, αποκτούμε την αίσθηση ότι δεν είναι. Πώς θα το αποδείξουμε;

Θα μελετήσουμε το αν υπάρχει αναγκαία και ικανή συνθήκη για να είναι ένα γράφημα επίπεδο σε
κατοπινό ερώτημα αυτού του κεφαλαίου.

5.2 Από τον Πλάτωνα στον Όυλερ και πίσω


I.
Πίνακας 5.1
Κανονικό Κορυφές επίπεδου Ακμές επίπεδου Περιοχές επίπεδου
Κ+Π–Α
Πολύεδρο γραφήματος γραφήματος γραφήματος

Τετράεδρο 4 6 4 2
Εξάεδρο 8 12 6 2

Οκτάεδρο 6 12 8 2
Δωδεκάεδρο 20 30 12 2

Εικοσάεδρο 12 30 20 2

Ψυχαγωγικά Μαθηματικά 115


II.
Πίνακας 5.2
Γράφημα Κορυφές Ακμές Περιοχές Κ+Π–Α

Κ1 1 0 1 2

Κ2 2 1 1 2

Κ3 3 3 2 2

Κ4 4 6 4 2

III. Κορυφές: 4, Ακμές: 7, Περιοχές: 5. Κ + Π – Α = 2.

IV. Κορυφές: 𝜈, Ακμές: 𝜈–1, Περιοχές: 1. Κ + Π – Α = 2.

V. Είναι εύλογο να διατυπώσουμε την ακόλουθη εικασία: Σε κάθε επίπεδο, συνεκτικό γράφημα, το
άθροισμα του αριθμού των κορυφών Κ και του αριθμού των περιοχών Π ισούται με τον αριθμό των
ακμών Α συν δύο (Κ + Π = Α + 2).
Θα δείξουμε ότι αληθεύει με τη μέθοδο της μαθηματικής επαγωγής (πάνω στο πλήθος των ακμών).
Έχουμε ήδη δείξει ότι ο τύπος αληθεύει για το δένδρο Τ1. Έστω ότι ο τύπος αληθεύει για κάθε γράφημα με
𝜈–1 ακμές. Θα δείξουμε ότι αληθεύει και για οποιοδήποτε γράφημα με 𝜈 ακμές. Έστω γράφημα Γ με 𝜈 ακμές,
𝜅 κορυφές και 𝜋 περιοχές. Αν το Γ είναι δένδρο, έχουμε ήδη δείξει ότι ο τύπος αληθεύει. Δεν απαιτείται καν η
χρήση της υπόθεσης της μαθηματικής επαγωγής. Αν δεν είναι δένδρο, έχει τουλάχιστον ένα κύκλωμα C.
Αφαιρώντας μία ακμή του κυκλώματος C, προκύπτει ένα γράφημα με 𝜈–1 ακμές, για το οποίο ισχύει η
υπόθεση της μαθηματικής επαγωγής. Το γράφημα αυτό έχει μία περιοχή λιγότερη από το Γ, αλλά ίδιο πλήθος
κορυφών 𝜅 με αυτό. Επομένως, έχουμε γι’ αυτό: 𝜅 + (𝜋–1) = (𝜈–1) + 2 και επομένως 𝜅 + 𝜋 = 𝜈 + 2, δηλαδή
ο τύπος ισχύει και για το αρχικό γράφημα Γ που έχει 𝜈 ακμές.

VI. Με τη βοήθεια του παραπάνω τύπου του Όυλερ μπορούμε να δείξουμε ότι υπάρχουν μόνο πέντε
κανονικά κυρτά πολύεδρα (πλατωνικά στερεά). Πράγματι, έστω πλατωνικό στερεό με Κ κορυφές, Α
ακμές, Ε έδρες.
Λόγω του τύπου του Euler για τα πλατωνικά γραφήματα, ισχύει Κ + Ε = Α + 2. Εάν ρ είναι ο βαθμός
κάθε κορυφής (δηλαδή ο αριθμός των ακμών του στερεού που την έχουν ένα άκρο) και εάν σ είναι ο αριθμός
των ακμών κάθε έδρας, τότε ρ, σ ≥ 3, και από το λήμμα των χειραψιών έχουμε σ∙Κ = ρ∙Ε = 2∙Α. Επομένως:
2∙Α 2∙Α 1 1 1 1 1 1 1
+ = Α + 2, οπότε + ρ = 2 + Α , οπότε σ + ρ > 2 . Αλλά τότε, (ρ,σ) μπορεί να πάρει τιμές μόνο από το
σ ρ σ
σύνολο {(3,3), (3,4), (4,3), (3,5), (5,3)}. Τότε, οι τιμές που παίρνει το Α αντίστοιχα είναι οι 6, 12, 12, 30, 30
και οι τιμές που παίρνει το Ε αντίστοιχα είναι οι 4, 6, 8, 12, 20 (κανονική τριγωνική πυραμίδα, κύβος,
κανονικό οκτάεδρο, κανονικό δωδεκάεδρο, κανονικό εικοσάεδρο).

Εναλλακτικά, μπορούμε να αποδείξουμε ότι υπάρχουν μόνο πέντε κανονικά κυρτά πολύεδρα
(πλατωνικά στερεά), με πιο γεωμετρικό τρόπο, ακολουθώντας την απόδειξη που δίνει ο Ευκλείδης στο 13 ο
βιβλίο των Στοιχείων του, απόδειξη που οφείλεται στον Θεαίτητο.

Έστω ότι έχουμε ένα κανονικό πολύεδρο που καθεμία από τις ίσες έδρες του είναι κανονικό ν-γωνο. Η
180∙(𝜈−2)
κάθε γωνία ενός κανονικού ν-γώνου είναι ίση με μοίρες (αφού υπάρχουν ν ίσες τέτοιες γωνίες και το
𝜈
ν-γωνο μπορεί να διαμειραστεί σε ν−2 τρίγωνα φέρνοντας τις διαγωνίους από μία κορυφή). Επομένως, εάν
γύρω από μια κορυφή του πολυέδρου μας βρίσκονται κ πολύγωνα, τότε το άθροισμα όλων των γωνιών των
180∙(𝜈−2)
εδρών αυτών που την έχουν κοινή κορυφή θα είναι ίσο με 𝜅 ∙ μοίρες. Αλλά το άθροισμα των μοιρών
𝜈

116 Χατζηκυριάκου Κώστας


αυτών πρέπει να είναι μικρότερο από 360 μοίρες, γιατί είναι φανερό ότι αν ήταν 360 μοίρες, όλες οι γωνίες θα
βρίσκονταν στο ίδιο επίπεδο και το στερεό μας δεν θα υπήρχε.
(𝜈−2)
Επομένως, πρέπει κ ∙ < 2, δηλαδή, κ ∙ (𝜈 − 2) < 2 ∙ 𝜈. Για να ισχύει αυτή η ανισότητα, είναι
𝜈
φανερό ότι πρέπει (𝜅 − 2) ∙ (𝜈 − 2) < 4. Αλλά επειδή τόσο το κ όσο και το ν πρέπει να είναι ακέραιοι
αριθμοί μεγαλύτεροι του δύο (γιατί;), μένουμε μόνο με τις παρακάτω πέντε περιπτώσεις:

Πίνακας 5.3
ν κ ν−2 κ−2 (κ − 2)(ν −2)
3 3 1 1 1
3 4 1 2 2
3 5 1 3 3
4 3 2 1 2
5 3 3 1 3

Π.χ. https://mathshistory.st-andrews.ac.uk/Biographies/Kepler/

5.3 To K3,3, το Κ5 και όλα τα μη επίπεδα γραφήματα


I. Έστω απλό γράφημα Γ, συνεκτικό και επίπεδο. Είναι φανερό ότι σε οποιαδήποτε επίπεδη αναπαράστασή
του κάθε περιοχή συνορεύει με τουλάχιστον τρεις ακμές. Επομένως, ο συνολικός αριθμός ακμών Ν που
θα μετρήσουμε, μετρώντας μία-μία όλες τις ακμές γύρω από όλες τις περιοχές, θα είναι τουλάχιστον 3∙π.
Από την άλλη, αφού κάθε ακμή βρίσκεται στο όριο, το πολύ, δύο περιοχών, ο συνολικός αριθμός των
ακμών που θα βρούμε έτσι δεν μπορεί να ξεπερνά το 2∙Α. Άρα, 3∙Π ≤ Ν ≤ 2∙Α. Αλλά για το γράφημα Γ
2 1
ισχύει ο τύπος του Όυλερ, Κ + Π – Α = 2, επομένως Κ + 3 ∙Α – Α ≥ 2, δηλαδή Κ − 3 ∙Α ≥ 2, δηλαδή Α
≤ 3∙Κ – 6.

II. Αν το K3,3 ήταν επίπεδο γράφημα, σε οποιαδήποτε επίπεδη αναπαράστασή του κάθε περιοχή θα
συνόρευε με 4 τουλάχιστον ακμές. Επομένως, με το ίδιο επιχείρημα που αναπτύξαμε παραπάνω, θα ίσχυε
4∙Π ≤ Ν ≤ 2∙Α. Δηλαδή, 2∙Π ≤ Α και αντίστοιχα Α ≤ 2∙Κ – 4. Αλλά Α = 9 και Κ = 6, οπότε 9 ≤ 2∙6 – 4
= 8, άτοπο.

III. Αν το K5 ήταν επίπεδο γράφημα, θα έπρεπε να ισχύει Α ≤ 3∙Κ – 6. Αλλά Α = 10, Κ = 5, οπότε 10 ≤ 3∙5 –
6 = 9, άτοπο.

IV. Έχουμε ήδη δείξει ότι τα γραφήματα αυτά είναι επίπεδα. Το γράφημα K5 είναι υπογράφημα
οποιουδήποτε πλήρους γραφήματος Kν, 𝜈 ≥ 6. Αν οποιοδήποτε τέτοιο Kν είχε επίπεδη αναπαράσταση,
τότε αυτό θα ίσχυε και για το υπογράφημά του K5, άτοπο.
Στο σημείο αυτό αξίζει να παρατηρήσουμε ότι η προσθήκη μίας κορυφής (βαθμού 2) σε ένα γράφημα
έτσι ώστε μία ακμή να χωριστεί σε δύο ακμές (υποδιαίρεση) ή η απαλοιφή μίας κορυφής βαθμού 2 σε ένα
γράφημα έτσι ώστε οι δύο μη γειτονικές κορυφές να γίνουν γειτονικές (εξομάλυνση) δεν μετατρέπει ένα
επίπεδο γράφημα σε μη επίπεδο ή ένα μη επίπεδο γράφημα σε επίπεδο.

Θα λέμε, λοιπόν, ότι δύο γραφήματα είναι ομοιομορφικά (homeomorphic) ανν είναι ισόμορφα ή
μπορούν να μετασχηματιστούν σε ισόμορφα με επαναλαμβανόμενες υποδιαιρέσεις ή εξομαλύνσεις.

Παραδείγματος χάριν, τα παρακάτω γραφήματα στο Σχήμα 5.4 είναι ομοιομορφικά:

Ψυχαγωγικά Μαθηματικά 117


Είναι εύλογο να αναρωτηθεί κανείς αν υπάρχει ικανή και αναγκαία συνθήκη για να είναι ένα
γράφημα επίπεδο. Η απάντηση είναι ναι. Το 1930 o σπουδαίος Πολωνός μαθηματικός Kuratowski
δημοσίευσε το εξής θεώρημα: Ένα γράφημα είναι επίπεδο ανν δεν έχει κανένα υπογράφημα που να είναι
ομοιομορφικό με το K5 ή το K3,3.

Η απόδειξη είναι μακροσκελής. Μπορείς να βρεις μία στο βιβλίο του F. Harary, Graph Theory ή στο
βιβλίο των J. A. Bondy και U. S. R. Murty, Graph Theory ή ακόμη και να διαβάσεις το άρθρο του ίδιου
του Kuratowski, Sur le problème des courbes gauches en Topologie, αν γνωρίζεις γαλλικά και θέλεις να
τα εξασκήσεις. Γνωρίζουμε ότι την ίδια, πάνω κάτω, εποχή με τον Kuratowski, και άλλοι μαθηματικοί
απέδειξαν το θεώρημα αυτό χωρίς να το δημοσιεύσουν. Σχετικά βλ. το άρθρο των J. W. Kennedy, L. V.

Quintas και Μ. Μ. Syslo (1985), The Theorem on Planar Graphs. HISTORIA MATHEMATICA 12, 356-
368.

V. Θα δείξουμε ότι το γράφημα του Petersen δεν είναι επίπεδο με δύο διαφορετικούς τρόπους:
Σύμφωνα με το θεώρημα του Kuratowski, αρκεί να βρούμε ένα υπογράφημα του γραφήματος του
Petersen ομοιομορφικό με το K5 ή το K3,3. Αλλά αυτό ισχύει, γιατί το γράφημα του Petersen έχει
υπογράφημα ομοιομορφικό με το K3,3. Στο Σχήμα 5.2, το υπογράφημα αυτό είναι το ίδιο το K3,3 = ({Α, Γ,
Θ}∪{Β, Δ, Λ}, {ΑΒ, ΑΔ, ΑΛ, ΓΒ, ΓΔ, ΓΛ, ΘΒ, ΘΔ, ΘΛ}). Καταλήγουμε σε αυτό διαγράφοντας την
κορυφή Η που παίρνει μαζί της τις ακμές ΕΗ, ΗΝ, ΗΞ, εξομαλύνοντας τις κορυφές Ν, Ξ και Ε.

Εναλλακτικά, μπορούμε να επιχειρήσουμε να εφαρμόσουμε τη μέθοδο επίλυσης που


χρησιμοποιήσαμε για να αποδείξουμε ότι τα K 3,3 και Κ5 δεν είναι επίπεδα. Αν το γράφημα του Petersen
ήταν επίπεδο, θα είχε 7 περιοχές, λόγω του τύπου του Euler (αφού οι κορυφές του είναι 10 και οι ακμές
του είναι 15). Παρατήρησε, ωστόσο, ότι κάθε περιοχή του θα έπρεπε να περικλείεται από τουλάχιστον 5
ακμές, επομένως αν το γράφημα αυτό ήταν επίπεδο, 5 ∙ 𝛱 ≤ 2 ∙ 𝛢. Δηλαδή, 35 ≤ 30, άτοπο.

118 Χατζηκυριάκου Κώστας


Σχήμα 5.5

Ψυχαγωγικά Μαθηματικά 119


Βιβλιογραφία/Αναφορές

Bondy, J. A., & Murty, U.S.R. (2008). Graph Theory. (Graduate Texts in mathematics, vol. 244). Springer.
Ηarary, F. (1969). Graph Theory. Addison-Wesley Publishing Company.
Kennedy, J. W., Quintas, L.V., & Sysło, Μ. Μ. (1985). The Theorem on Planar Graphs. HISTORIA
MATHEMATICA 12, 356-368.
Kuratowski, C. (1930). Sur le problème des courbes gauches en Topologie. Fundamenta Mathematicae 15,
271-283.
Polya, G. (1998). Πώς να το λύσω. Εκδόσεις Καρδαμίτσα

120 Χατζηκυριάκου Κώστας


Κεφάλαιο 6
Χρωματισμοί

Σύνοψη
Χρωματισμοί χαρτών και γραφημάτων.

Προαπαιτούμενη γνώση
Κεφάλαια 2, 3, 4, 5 του παρόντος συγγράμματος.

Ψυχαγωγικά Μαθηματικά 121


6.1 Χρωματίζοντας χάρτες
I. Μπορείς να χρωματίσεις τον παρακάτω χάρτη που αναπαριστά τις χώρες μιας περιοχής:
i. με έξι διαφορετικά χρώματα,
ii. με πέντε διαφορετικά χρώματα,
iii. με τέσσερα διαφορετικά χρώματα,
iv. με τρία διαφορετικά χρώματα,
έτσι ώστε χώρες που συνορεύουν να είναι χρωματισμένες με διαφορετικό χρώμα;

Σχήμα 6.1

122 Χατζηκυριάκου Κώστας


II. Μπορείς να χρωματίσεις τον παρακάτω χάρτη με δύο χρώματα;

Σχήμα 6.2
III. Μπορείς να χρωματίσεις τον παρακάτω χάρτη με δύο χρώματα;

Σχήμα 6.3

Ψυχαγωγικά Μαθηματικά 123


6.2 Χρωματίζοντας τα επίπεδα πλατωνικά γραφήματα
I. Χρωμάτισε με όσο το δυνατόν λιγότερα χρώματα τις περιοχές του επίπεδου γραφήματος του κανονικού
τετραέδρου έτσι ώστε περιοχές που συνορεύουν να είναι χρωματισμένες με διαφορετικά χρώματα.
Ποιο είναι το δυϊκό γράφημα του επίπεδου γραφήματος του κανονικού τετραέδρου;

Ποιος είναι ο χρωματικός αριθμός του επίπεδου γραφήματος του κανονικού τετραέδρου;

124 Χατζηκυριάκου Κώστας


II. Χρωμάτισε με όσο το δυνατόν λιγότερα χρώματα τις περιοχές του επίπεδου γραφήματος του κύβου έτσι
ώστε περιοχές που συνορεύουν να είναι χρωματισμένες με διαφορετικά χρώματα.
Δείξε ότι το δυϊκό γράφημα του επίπεδου γραφήματος του κύβου είναι το επίπεδο γράφημα του
κανονικού οκταέδρου.

Ποιος είναι ο χρωματικός αριθμός του επίπεδου γραφήματος του κύβου;

Χρωμάτισε με όσο το δυνατόν λιγότερα χρώματα τις περιοχές του επίπεδου γραφήματος του
κανονικού οκταέδρου έτσι ώστε περιοχές που συνορεύουν να είναι χρωματισμένες με διαφορετικά
χρώματα.

Ποιος είναι ο χρωματικός αριθμός του επίπεδου γραφήματος του κανονικού οκταέδρου;

Ψυχαγωγικά Μαθηματικά 125


III. Χρωμάτισε με όσο το δυνατόν λιγότερα χρώματα τις περιοχές του επίπεδου γραφήματος του κανονικού
δωδεκαέδρου έτσι ώστε περιοχές που συνορεύουν να είναι χρωματισμένες με διαφορετικά χρώματα.
Δείξε ότι το δυϊκό γράφημα του επίπεδου γραφήματος του κύβου είναι το επίπεδο γράφημα του
κανονικού εικοσαέδρου.

Ποιος είναι ο χρωματικός αριθμός του επίπεδου γραφήματος του κανονικού δωδεκαέδρου;

Χρωμάτισε με όσο το δυνατόν λιγότερα χρώματα τις περιοχές του επίπεδου γραφήματος του
κανονικού εικοσαέδρου έτσι ώστε περιοχές που συνορεύουν να είναι χρωματισμένες με διαφορετικά
χρώματα.

Ποιος είναι ο χρωματικός αριθμός του επίπεδου γραφήματος του κανονικού εικοσαέδρου;

126 Χατζηκυριάκου Κώστας


6.3 Χρωματισμοί και χρωματικοί αριθμοί
I. Ποιος είναι ο χρωματικός αριθμός του Κν;

II. Ποιος είναι ο χρωματικός αριθμός οποιουδήποτε δένδρου Τν (ν≥2);

III. Ποιος είναι ο χρωματικός αριθμός του γραφήματος του Petersen;

IV. Ποιος είναι ο χρωματικός αριθμός γραφήματος–κυκλώματος (δηλ. κανονικού που όλες οι κορυφές του
έχουν βαθμό 2) με άρτιο αριθμό κορυφών; Με περιττό αριθμό κορυφών;

Ψυχαγωγικά Μαθηματικά 127


V. Ποιος είναι ο χρωματικός αριθμός τροχού με άρτιο αριθμό κορυφών (Σχήμα 6.4, αριστερά); Μπορείς να
χρωματίσεις τις περιοχές του με τρία χρώματα έτσι ώστε περιοχές που συνορεύουν να είναι
χρωματισμένες διαφορετικά;

VI. Ποιος είναι ο χρωματικός αριθμός τροχού με περιττό αριθμό κορυφών (Σχήμα 6.4, δεξιά); Μπορείς να
χρωματίσεις τις περιοχές του με τρία χρώματα έτσι ώστε περιοχές που συνορεύουν να είναι
χρωματισμένες

Σχήμα 6.4

128 Χατζηκυριάκου Κώστας


6.4 Χρωματίζοντας ακμές
I. Ποιος είναι ο μικρότερος αριθμός διαφορετικών χρωμάτων με τα οποία μπορείς να χρωματίσεις τις ακμές
του Κ3 έτσι ώστε όλες οι γειτονικές ακμές να έχουν διαφορετικό χρώμα;

II. Ποιος είναι ο μικρότερος αριθμός διαφορετικών χρωμάτων με τα οποία μπορείς να χρωματίσεις τις ακμές
του Κ4 έτσι ώστε όλες οι γειτονικές ακμές να έχουν διαφορετικό χρώμα;

Ψυχαγωγικά Μαθηματικά 129


III. Ποιος είναι ο μικρότερος αριθμός διαφορετικών χρωμάτων με τα οποία μπορείς να χρωματίσεις τις ακμές
του Κ5 έτσι ώστε όλες οι γειτονικές ακμές να έχουν διαφορετικό χρώμα;

IV. Ποιος είναι ο μικρότερος αριθμός διαφορετικών χρωμάτων με τα οποία μπορείς να χρωματίσεις τις ακμές
του Κ6 έτσι ώστε όλες οι γειτονικές ακμές να έχουν διαφορετικό χρώμα;

130 Χατζηκυριάκου Κώστας


V. Τι παρατηρείς; Μήπως μπορείς να διατυπώσεις μια αποδείξιμη εικασία (και να την αποδείξεις);

Ψυχαγωγικά Μαθηματικά 131


VI. Ποιος είναι ο χρωματικός δείκτης των πλατωνικών γραφημάτων; Ποιος είναι ο χρωματικός δείκτης του
γραφήματος του Petersen;

132 Χατζηκυριάκου Κώστας


Απαντήσεις στα ερωτήματα του 6ου Kεφαλαίου

6.1 Χρωματίζοντας χάρτες


I.
i. Στο Σχήμα 6.1.α βλέπουμε έναν χρωματισμό του χάρτη του Σχήματος 6.1 με έξι χρώματα.

Σχήμα 6.1.α
Φυσικά, η εξωτερική περιοχή του χάρτη μπορεί να χρωματιστεί με κάποιο από τα χρώματα που δεν
χρωματίζει χώρα του συνόρου, π.χ. με κίτρινο.

Αναστοχαζόμενοι, κατά την πρόταση του Polya στο Πώς να το λύσω, τη λύση μας, φτάνουμε αμέσως
στο ερώτημα: Η δυνατότητα να χρωματίσουμε αυτόν τον χάρτη είναι άραγε ιδιότητα αυτού του χάρτη ή όλοι
οι παρόμοιοι χάρτες μπορούν να χρωματιστούν με έξι διαφορετικά χρώματα με αυτόν τον τρόπο;

Ψυχαγωγικά Μαθηματικά 133


Θυμόμαστε ότι στη σπαζοκεφαλιά των εφτά γεφυρών αναπαραστήσαμε τις όχθες και τα νησάκια ως
κορυφές ενός χάρτη. Αν σκεφτούμε ανάλογα, μπορούμε να αναπαραστήσουμε τις χώρες με τις κορυφές ενός
γραφήματος του οποίου οι ακμές θα συνδέουν κορυφές (χώρες) που μοιράζονται σύνορο. Το γράφημα που
προκύπτει έτσι, στη θεωρία γραφημάτων ονομάζεται δυϊκό γράφημα του χάρτη (βλ. Σχήμα 6.1.β).

Σχήμα 6.1.β
Βλέπουμε ότι, μέσω της μαθηματικής αυτής αναπαράστασης, αυτό που επιτύχαμε ήταν να χρωματίσουμε τις
κορυφές ενός επίπεδου γραφήματος (χωρίς βρόχους) με έξι χρώματα έτσι ώστε δύο οποιεσδήποτε γειτονικές
κορυφές να μην είναι χρωματισμένες με το ίδιο χρώμα. Και επομένως, μπορούμε τώρα να ρωτήσουμε:

Αληθεύει ότι μπορούμε να χρωματίσουμε με ακριβώς έξι χρώματα τις κορυφές οποιουδήποτε επίπεδου
γραφήματος (χωρίς βρόχους) έτσι ώστε γειτονικές κορυφές να έχουν διαφορετικό χρώμα;

Η απάντηση είναι ναι και θα το δείξουμε εφαρμόζοντας τη μέθοδο της μαθηματικής επαγωγής στον
αριθμό των κορυφών. Είναι φανερό πως η εικασία μας αληθεύει αν οι κορυφές είναι λιγότερες από επτά. Ας
υποθέσουμε, λοιπόν, ότι το επίπεδο γράφημά μας έχει 𝜈 (≥ 7) 𝜅ορυφές και ότι όλα τα επίπεδα γραφήματα με
𝜈 − 1 κορυφές μπορούν να χρωματιστούν με αυτόν τον τρόπο (έξι διαφορετικά χρώματα, γειτονικές κορυφές
διαφορετικά χρωματισμένες). Υποθέτοντας, προφανώς, χωρίς περιορισμό της γενικότητας ότι το γράφημα
είναι απλό, το γράφημά μας έχει τουλάχιστον μία κορυφή 𝜅 βαθμού ≤ 5.

134 Χατζηκυριάκου Κώστας


Γιατί; Διότι, αν όλες οι ν κορυφές είχαν βαθμό τουλάχιστον έξι, τότε το άθροισμα των βαθμών των
κορυφών θα ήταν 2 ∙ 𝜇 ≥ 6 ∙ 𝜈, (𝜇 = το πλήθος των ακμών του γραφήματος), δηλαδή 𝜇 ≥ 3 ∙ 𝜈. Αλλά εμείς
γνωρίζουμε ότι σε ένα τέτοιο γράφημα 3 ∙ 𝜈 − 6 ≥ 𝜇. Οπότε, 3 ∙ 𝜈 − 6 ≥ 𝜇 ≥ 3 ∙ 𝜈, άτοπο.

Αν αφαιρέσουμε την κορυφή 𝜅, τότε το γράφημά μας έχει 𝜈 − 1 κορυφές και επομένως μπορεί να
χρωματιστεί με τον τρόπο που θέλουμε. Χρωματίζουμε και την κορυφή 𝜅 με ένα χρώμα διαφορετικό από τα
χρώματα που χρησιμοποιήθηκαν στις, το πολύ πέντε, γειτονικές κορυφές της και έχουμε τον χρωματισμό που
ψάχνουμε.

Στο σημείο αυτό είναι φανερό πως η επινόηση ορολογίας θα μας βοηθούσε στη διατύπωση των
προτάσεών μας και στις προσπάθειές μας να τις αποδείξουμε. Ας ονομάσουμε, λοιπόν, ένα γράφημα χωρίς
βρόχους 𝒌 −χρωματίσιμο ανν μπορούμε να χρωματίσουμε τις κορυφές του με 𝑘 διαφορετικά χρώματα έτσι
ώστε οποιεσδήποτε δύο γειτονικές κορυφές να είναι χρωματισμένες με διαφορετικά χρώματα. Αν για ένα
γράφημα Γ, 𝑘 είναι ο μικρότερος αριθμός διαφορετικών χρωμάτων με τα οποία μπορεί να γίνει ένας τέτοιος
χρωματισμός, τότε λέμε ότι ο χρωματικός αριθμός 𝝌(𝜞) του γραφήματος Γ είναι 𝑘. Επομένως, μόλις
αποδείξαμε ότι κάθε επίπεδο γράφημα (χωρίς βρόχους) είναι 𝟔 −χρωματίσιμο.

Αξίζει εδώ να παρατηρήσουμε ότι αυτό που αποκαλέσαμε χάρτη είναι επίσης ένα γράφημα, συνεκτικό
και επίπεδο, του οποίου οι κορυφές αντιστοιχούν στα σημεία συνάντησης των συνόρων των χωρών του χάρτη
και οι ακμές του αντιστοιχούν στα τμήματα των συνόρων που συνδέουν τέτοια σημεία συνάντησης, και ότι
καμία ακμή του δεν είναι ισθμός (ιδιότητα που κάνει το δυϊκό του γράφημα να μην έχει βρόχους).

Επομένως, κάθε χάρτης (δηλαδή επίπεδο, συνεκτικό γράφημα χωρίς ακμή–ισθμό) μπορεί να
χρωματιστεί με 6 διαφορετικά χρώματα έτσι ώστε οι περιοχές του που συνορεύουν να είναι χρωματισμένες με
διαφορετικά χρώματα. Λέμε και γράφουμε ότι είναι 𝟔 −χρωματίσιμος (π).

Η μέθοδος που χρησιμοποιήσαμε, δηλαδή το πέρασμα στο δυϊκό γράφημα του χάρτη και η μελέτη του
ερωτήματος «είναι ο δυϊκός χάρτης 𝒌 −χρωματίσιμος», κάνει φανερό ότι ισχύει η εξής πρόταση: Ένα
συνεκτικό, επίπεδο γράφημα χωρίς ακμή–ισθμό είναι 𝒌 −χρωματίσιμο (π) ανν το δυϊκό του γράφημα
(που δεν έχει βρόχους) είναι 𝒌 −χρωματίσιμο.

ii. Στο Σχήμα 6.1.γ βλέπουμε έναν χρωματισμό του χάρτη του Σχήματος 6.1 με πέντε χρώματα.

Σχήμα 6.1.γ

Ψυχαγωγικά Μαθηματικά 135


Φυσικά, η εξωτερική περιοχή του χάρτη μπορεί να χρωματιστεί με κάποιο από τα χρώματα που δεν
χρωματίζει χώρα του συνόρου, π.χ. με μωβ.

Αναστοχαζόμενοι, και πάλι κατά την πρόταση του Polya, τη λύση μας, φτάνουμε αμέσως στο ερώτημα:
Η δυνατότητα να χρωματίσουμε αυτόν τον χάρτη είναι άραγε ιδιότητα αυτού του χάρτη ή όλοι οι παρόμοιοι
χάρτες μπορούν να χρωματιστούν με πέντε διαφορετικά χρώματα με αυτόν τον τρόπο;

Περνώντας, όπως και προηγουμένως, στο δυϊκό γράφημα του χάρτη, βλέπουμε ότι μόλις δείξαμε ότι
είναι 5 −χρωματίσιμο. Μπορούμε και πάλι να ρωτήσουμε: Είναι κάθε επίπεδο γράφημα (χωρίς βρόχους)
5 −χρωματίσιμο;

Η απάντηση είναι και πάλι ναι και θα το δείξουμε εφαρμόζοντας ξανά τη μέθοδο της μαθηματικής
επαγωγής στον αριθμό των κορυφών. Είναι φανερό πως η εικασία μας αληθεύει αν οι κορυφές του
γραφήματος είναι λιγότερες από έξι. Ας υποθέσουμε, λοιπόν, ότι το επίπεδο γράφημά μας έχει 𝜈 (≥
6) 𝜅ορυφές και ότι όλα τα επίπεδα γραφήματα με 𝜈 − 1 κορυφές είναι 5 −χρωματίσιμα. Υποθέτοντας και
πάλι, προφανώς χωρίς περιορισμό της γενικότητας, ότι το γράφημα είναι απλό, το γράφημά μας έχει
τουλάχιστον μία κορυφή 𝜅 βαθμού 5.

Αν αφαιρέσουμε την κορυφή 𝜅, τότε το γράφημά μας έχει 𝜈 − 1 κορυφές και επομένως είναι
5 −χρωματίσιμο. Πώς θα χρωματίσουμε την κορυφή 𝜅 με ένα από τα πέντε χρώματα έτσι ώστε το γράφημα Γ
να είναι 5 −χρωματίσιμο;

Αν ο βαθμός της 𝜅 είναι μικρότερος του 5, τότε μπορούμε να τη χρωματίσουμε με ένα από τα χρώματα
που δεν έχουν χρησιμοποιηθεί στον χρωματισμό των γειτονικών κορυφών της και να ολοκληρώσουμε έτσι
τον ζητούμενο χρωματισμό. Τι μπορούμε να κάνουμε αν ο βαθμός της 𝜅 είναι 5; Ας ονοματίσουμε τις
γειτονικές κορυφές της 𝜅1 , 𝜅2 , 𝜅3 , 𝜅4 , 𝜅5 . Αν τύχει και δύο από αυτές είναι χρωματισμένες με το ίδιο
χρώμα, τότε μπορούμε να χρησιμοποιήσουμε αυτό που περισσεύει για να χρωματίσουμε την 𝜅, και έχουμε,
έτσι, τον ζητούμενο χρωματισμό του Γ.

Τι γίνεται, όμως, αν και οι πέντε γειτονικές κορυφές της 𝜅 είναι χρωματισμένες με διαφορετικά
χρώματα 𝜒1 , 𝜒2 , 𝜒3 , 𝜒4 , 𝜒5 ; Ας ονομάσουμε Γij το υπογράφημα του γραφήματος Γ που περιέχει τις κορυφές
που είναι χρωματισμένες είτε με το χρώμα 𝜒𝑖 είτε με το χρώμα 𝜒𝑗 και οι ακμές του είναι μόνο εκείνες οι ακμές
του Γ που το ένα άκρο τους είναι χρωματισμένο με το χρώμα 𝜒𝑖 και το άλλο με το χρώμα 𝜒𝑗 .

Αν οι κορυφές 𝜅1 , 𝜅3 δεν ανήκουν στην ίδια συνιστώσα του Γ13 (που προφανώς μπορεί να μην είναι
συνεκτικό), τότε αλλάζουμε αμοιβαία τον χρωματισμό της 𝜅1 και της 𝜅3 , οπότε τώρα η 𝜅1 είναι
χρωματισμένη με το χρώμα 𝜒3 και έτσι μπορούμε να χρωματίσουμε την κορυφή 𝜅 με το χρώμα 𝜒1 και να
έχουμε τον ζητούμενο χρωματισμό του Γ. Αν, όμως, οι κορυφές ανήκουν στην ίδια συνιστώσα Γ13 , τότε η
συνιστώσα αυτή περιέχει ένα κύκλωμα Κ: 𝜅 → 𝜅1 → . . . → 𝜅3 → 𝜅. Οι κορυφές 𝜅2 , 𝜅4 , όμως, δεν ανήκουν
στο Κ και δεν υπάρχει μονοπάτι από την 𝜅2 στην 𝜅4 που να ανήκει εξ ολοκλήρου στο υπογράφημα Γ24 . Στη
συνιστώσα του Γ24 που περιέχει την 𝜅2 , αλλάζουμε αμοιβαία τον χρωματισμό της 𝜅2 και της 𝜅4 , οπότε
τώρα η 𝜅2 είναι χρωματισμένη με το χρώμα 𝜒4 και έτσι μπορούμε να χρωματίσουμε την κορυφή 𝜅 με το
χρώμα 𝜒2 και να έχουμε τον ζητούμενο χρωματισμό του γραφήματος Γ.

Επομένως, μόλις αποδείξαμε ότι κάθε επίπεδο γράφημα (χωρίς βρόχους) είναι 𝟓 −χρωματίσιμο και
ταυτόχρονα ότι κάθε χάρτης (δηλ. επίπεδο, συνεκτικό γράφημα χωρίς ακμή–ισθμό) μπορεί να χρωματιστεί με
5 χρώματα έτσι ώστε οι περιοχές του που συνορεύουν να είναι χρωματισμένες με διαφορετικά χρώματα, είναι
δηλαδή 5 −χρωματίσιμος (π).

136 Χατζηκυριάκου Κώστας


iii. Στο Σχήμα 6.1.δ βλέπουμε έναν χρωματισμό του χάρτη του Σχήματος 6.1 με τέσσερα χρώματα.

Σχήμα 6.1.δ
Φυσικά, η εξωτερική περιοχή του χάρτη μπορεί να χρωματιστεί με κάποιο από τα χρώματα που δεν
χρωματίζει χώρα του συνόρου, π.χ. με πράσινο.
Αναστοχαζόμενοι, και πάλι κατά την πρόταση του Polya, τη λύση μας, βλέπουμε, πρώτον, ότι ο χάρτης
αυτός δεν είναι 3 −χρωματίσιμος (π) και επομένως υπάρχουν χάρτες που δεν είναι 3 −χρωματίσιμοι (π) και,
δεύτερον, ότι είναι εύλογο να ρωτήσουμε: Η δυνατότητα να χρωματίσουμε αυτόν τον χάρτη είναι άραγε
ιδιότητα αυτού του χάρτη ή όλοι οι παρόμοιοι χάρτες μπορούν να χρωματιστούν με τέσσερα διαφορετικά
χρώματα με αυτόν τον τρόπο;

Περνώντας, όπως και προηγουμένως, στο δυϊκό γράφημα του χάρτη, βλέπουμε ότι μόλις δείξαμε ότι
είναι 4 −χρωματίσιμο. Μπορούμε και πάλι να ρωτήσουμε: Είναι κάθε επίπεδο γράφημα (χωρίς βρόχους)
𝟒 −χρωματίσιμο; Η απάντηση είναι Ναι.

Ωστόσο, η απόδειξη είναι ένα από τα πιο περίφημα μαθηματικά προβλήματα που έμειναν αναπόδεικτα
για πολλά χρόνια. To πρόβλημα λύθηκε τελικά το 1976 από τους Kenneth Appel and Wolfgang Haken με τη
βοήθεια υπολογιστή, με τον οποίον έλεγξαν τα χαρακτηριστικά περίπου δύο χιλιάδων συγκεκριμένων χαρτών
που έπρεπε να ελεγχθούν ώστε να ολοκληρωθεί η στρατηγική επίλυσής τους. Το υπολογιστικό μέρος της
απόδειξης απλοποιήθηκε αργότερα, αλλά ακόμη και σήμερα δεν υπάρχει κλασική μαθηματική απόδειξη του
θεωρήματος, δηλαδή απόδειξη που δεν χρησιμοποιεί υπολογιστή, και όλες οι λεπτομέρειές της μπορούν να
ελεγχθούν από ανθρώπους. Το γεγονός αυτό έχει δώσει τροφή στη φιλοσοφική σκέψη για τα μαθηματικά και
την απόδειξη (Βλ. ενδεικτικά το άρθρο του Thomas Tyzoscko (1986), The Four Colour Problem and its
Philosophical Significance, στο βιβλίο New Directions in the Philosophy of Mathematics, BIRKHÄUSER και
το άρθρο της Izabela Bondecka-Krzykowska (2004), The four-color theorem and its consequences for the
philosophy of mathematics, Annales UMCS Informatica AI 2, 5-14).

Ψυχαγωγικά Μαθηματικά 137


Το πρόβλημα των τεσσάρων χρωμάτων πρωτοεμφανίζεται σε μια επιστολή την οποία έστειλε το 1852
ο August De Morgan στον William Rowan Hamilton, στην οποία του εξηγεί το πρόβλημα και ζητά τη
συμβολή του στην απόδειξη. Το σχετικό ερώτημα, μάλιστα, του το έθεσε ο μαθητής του Frederick Guthrie, o
οποίος αρκετά χρόνια αργότερα αποκάλυψε ότι το ερώτημα του το έθεσε ο αδελφός του Francis Guthrie. Το
1879 ο δικηγόρος και ερασιτέχνης μαθηματικός Alfred Bray Kempe ανήγγειλε στο περιοδικό Nature ότι
απέδειξε το θεώρημα και η απόδειξη που παρουσίασε θεωρούνταν ορθή ως το 1890 όταν ο Percy John
Heawood ανέδειξε το σημείο όπου υπήρχε λάθος και, διαφοροποιώντας τη μέθοδο του Kempe, απέδειξε,
ουσιαστικά με τον τρόπο που είδαμε, ότι κάθε χάρτης μπορεί να χρωματιστεί με πέντε διαφορετικά χρώματα.
Η επιτυχής στρατηγική επίλυσης εγκαταλείπει την αρχή της μαθηματικής επαγωγής και χρησιμοποιεί την
ισοδύναμή της αρχή, του ελάχιστου φυσικού αριθμού (κάθε μη κενό σύνολο φυσικών περιέχει έναν ελάχιστο
φυσικό αριθμό), σε συνδυασμό με την αποδεικτική μέθοδο της εις άτοπον απαγωγής. Αν η εικασία των
τεσσάρων χρωμάτων δεν αληθεύει, τότε ανάμεσα στους χάρτες που για να χρωματιστούν έτσι απαιτούν 5
διαφορετικά χρώματα υπάρχουν κάποιοι «μικρότεροι», υπό την έννοια ότι οποιοσδήποτε «μικρότερός» τους
μπορεί να χρωματιστεί με 4 διαφορετικά χρώματα. Η ιδέα είναι με τη βοήθειά τους να οδηγηθούμε σε
χρωματισμό των αντιπαραδειγμάτων της εικασίας με τέσσερα διαφορετικά χρώματα. Αλλά αυτό είναι
αντιφατικό· επομένως, δεν υπάρχουν τέτοιοι χάρτες, άρα όλοι οι χάρτες μπορούν να χρωματιστούν με
τέσσερα χρώματα. Για σύντομη περιγραφή της ιστορίας και της προϊστορίας της απόδειξης, βλ. Richard
Courant & Herbert Robbins (1996) (revised by Ian Stewart), What is Mathematics, Oxford Paperbacks, (2nd
edition), ενώ για μια λεπτομερή ανασυγκρότηση των προσπαθειών που έγιναν για την απόδειξη της εικασίας
ως το 1934, βλ. Norman L. Bigg, E. Keith Lloyd, Robin J. Wilson (1999), Graph theory 1736-1936,
Clarendon Press.

Παρατηρούμε ότι ο χάρτης που μας δίνεται στο Σχήμα 6.2 μπορεί να θεωρηθεί, όντως, ως ένα
γράφημα με 4 κορυφές και επτά ακμές, συνεκτικό και επίπεδο, χωρίς ακμή–ισθμό. Κατά βάση είναι το μη
οϋλεριανό γράφημα που αναπαριστά τη σπαζοκεφαλιά των Επτά Γεφυρών του Κένινγκσμπεργκ. Αν
ακολουθήσουμε την τεχνική που έχουμε αναπτύξει, το γεωμετρικό δυϊκό γράφημα του χάρτη μας είναι το
ακόλουθο γράφημα στο Σχήμα 6.2α, το οποίο έχουμε αναπαραστήσει μαζί με τον αρχικό χάρτη.

Σχήμα 6.2.α
Αν χρωματίσουμε την κορυφή Ε με το χρώμα 1, την Ζ πρέπει να τη χρωματίσουμε με το διαφορετικό χρώμα
2, την Η με το χρώμα 1, την Κ με το χρώμα 2, τη Θ με το χρώμα 1, άτοπο. Ο χάρτης μας δεν χρωματίζεται με
δύο χρώματα. Χρωματίζεται, βέβαια, με 3 χρώματα, αφού μπορούμε να χρωματίσουμε την κορυφή Ε με το
χρώμα 1, την κορυφή Ζ με το χρώμα 2, την κορυφή Η με το χρώμα 1, την κορυφή Κ με το χρώμα 3 και την
κορυφή Θ με το χρώμα 2.

II. Ο χάρτης στο Σχήμα 6.3 είναι, κατά βάση, το παρακάτω οϋλεριανό γράφημα (Σχήμα 6.3.α), το οποίο
αναπαριστάνουμε μαζί με το δυϊκό του.

138 Χατζηκυριάκου Κώστας


Σχήμα 6.3.α
Παρατηρούμε ότι ο χάρτης μας μπορεί να χρωματιστεί με δύο χρώματα. Οι κορυφές του δυϊκού γραφήματος
(περιοχές του χάρτη) Μ, Ν, Ξ, Ο με το χρώμα 1 και οι κορυφές του δυϊκού γραφήματος (περιοχές του χάρτη)
Π, Ρ, Χ με το χρώμα 2.

Αναλογιζόμενοι τους δύο τελευταίους χρωματισμούς, θα μπορούσαμε να αναρωτηθούμε μήπως η


δυνατότητα του ενός χάρτη να χρωματίζεται με δύο (2) χρώματα και του άλλου χάρτη να μην χρωματίζεται
σχετίζεται με το ότι το 2 −χρωματίσιμο (π) γράφημα είναι οϋλεριανό, ενώ το άλλο, το 3 −χρωματίσιμο (π),
δεν είναι οϋλεριανό. Υποβοηθούμενοι, ίσως, και από άλλα παραδείγματα, θα μπορούσαμε τότε να
διατυπώσουμε την εικασία:

Ένας χάρτης Γ είναι 𝟐 −χρωματίσιμος (π) ανν το Γ είναι οϋλεριανό γράφημα.


Αλλά την πρόταση αυτή μπορούμε να την αποδείξουμε, αφού μπορούμε να αποδείξουμε ότι ένα
επίπεδο συνεκτικό γράφημα είναι οϋλεριανό ανν το δυϊκό γράφημά του είναι διμερές, καθώς είναι φανερό ότι
ένα γράφημα είναι διμερές ανν είναι 2‒χρωματίσιμο. (Στο βιβλίο του R. J. Wilson, Introduction to Graph
Theory, μπορείς να βρεις και μια διαφορετική απόδειξη της εικασίας αυτής).

Έστω Γ, λοιπόν, ένα επίπεδο οϋλεριανό γράφημα. Αφού το Γ είναι οϋλεριανό, όλες οι κορυφές του
έχουν άρτιο βαθμό. Αλλά εξ αιτίας του τρόπου που σχηματίζουμε το δυϊκό γράφημα Γ* ενός γραφήματος,
κάθε περιοχή στο Γ* ορίζεται από άρτιο αριθμό ακμών, οι οποίες σχηματίζουν κυκλώματα με άρτιο πλήθος
ακμών και επομένως το γράφημα δεν έχει κυκλώματα με περιττό πλήθος ακμών. Αυτό, όμως, σημαίνει ότι
είναι διμερές.

Γιατί; Διότι έστω κ0 μια οποιαδήποτε κορυφή του Γ* και έστω Γ1 το σύνολο των κορυφών κ του Γ*
που το συντομότερο μονοπάτι κ . . . κ0 έχει περιττό μήκος και έστω Γ2 το σύνολο των κορυφών κ του Γ* που
το συντομότερο μονοπάτι κ . . . κ0 έχει άρτιο μήκος. Τότε, τα σύνολα Γ1, Γ2 δείχνουν ότι το Γ* είναι διμερές,
αφού είναι φανερό ότι Γ = Γ1∪Γ2 και ότι δεν μπορεί να υπάρχει ακμή ούτε μεταξύ δύο κορυφών που
βρίσκονται και οι δύο στο Γ1 ούτε μεταξύ δύο κορυφών που βρίσκονται και οι δύο στο Γ 2 (και στη μια και
στην άλλη περίπτωση θα είχαμε ένα κύκλωμα με περιττό αριθμό ακμών).

Αντίστροφα, έστω ότι το Γ* είναι διμερές γράφημα, δυϊκό ενός επίπεδου συνεκτικού γραφήματος Γ. Σε
ένα διμερές γράφημα, όμως, είναι φανερό ότι κάθε κύκλωμα έχει άρτιο πλήθος ακμών· επομένως, κάθε
περιοχή του ορίζεται από άρτιο πλήθος ακμών και άρα ο βαθμός κάθε κορυφής του Γ είναι άρτιος, δηλαδή το
Γ είναι οϋλεριανό γράφημα.

Παρατήρησε ότι παράλληλα αποδείξαμε μια ενδιαφέρουσα πρόταση:

Ένα γράφημα είναι διμερές ανν δεν έχει κυκλώματα με περιττό πλήθος ακμών.

Ψυχαγωγικά Μαθηματικά 139


Είναι φανερό ότι το γράφημα του κανονικού τετραέδρου δεν μπορεί να χρωματιστεί με 3 χρώματα.
Είναι εξίσου εύκολο να δει κανείς ότι το γράφημα του κανονικού εξαέδρου χρωματίζεται με τρία χρώματα.
Το γράφημα του κανονικού οκταέδρου είναι οϋλεριανό· επομένως, χρωματίζεται με 2 χρώματα. Το γράφημα
του κανονικού δωδεκαέδρου είναι κυβικό και κάθε περιοχή του ορίζεται από 5 ακμές, οπότε δεν είναι
δυνατόν να χρωματίσουμε τις περιοχές του με 3 χρώματα, άρα χρωματίζονται με 4 χρώματα. Το γράφημα του
κανονικού εικοσαέδρου είναι οϋλεριανό, άρα χρωματίζεται με 2 χρώματα.

6.2 Χρωματίζοντας τα επίπεδα πλατωνικά γραφήματα


Είτε εφαρμόσουμε τη μέθοδο του δυϊκού γραφήματος είτε εργαστούμε με τον χάρτη του Π 4 (βλ. Σχήμα 6.5)
είναι φανερό ότι το Π4 είναι 4–χρωματίσιμο (π), με τα χρώματα α, β, γ, δ. Παρατήρησε, ωστόσο, ότι το δυϊκό
του Π4 είναι πάλι το Π4, οπότε και ο χρωματικός αριθμός του Π4 είναι 4.

Σχήμα 6.5
Είναι φανερό ότι το Π6 είναι 3–χρωματίσιμο (π). Παρατήρησε (Σχήμα 6.6) ότι το δυϊκό του είναι το Π8 και
(επομένως) ο χρωματικός αριθμός του Π8 είναι 3. Είναι φανερό πως το γράφημα αυτό είναι διμερές, άρα ο
χρωματικός αριθμός του είναι 2. Από την άλλη, παρατήρησε ότι το Π 8 είναι κανονικό, με όλες τις κορυφές
του να έχουν άρτιο βαθμό (4). Οπότε, είναι οϋλεριανό και επομένως το Π8 είναι, όπως αναμέναμε άλλωστε,
αφού το δυϊκό γράφημα του είναι το Π6, 2–χρωματίσιμο (π).

Σχήμα 6.6

140 Χατζηκυριάκου Κώστας


Στο παρακάτω σχήμα (Σχήμα 6.7) βλέπουμε πως το δυϊκό γράφημα του Π12 είναι το Π20. Ας δούμε τον
χρωματισμό του Π12. Χρωματίζουμε την πιο εσωτερική περιοχή του σ κόκκινη. Είναι φανερό ότι για να
χρωματίσουμε τις πέντε περιοχές ζ, η, θ, ν, κ που την περικλείουν, χρειαζόμαστε τρία διαφορετικά χρώματα,
και ας τις χρωματίσουμε με πράσινο, ροζ, θαλασσί, ροζ, θαλασσί χρώμα αντίστοιχα. Χρωματίζουμε τις πέντε
περιοχές α, β, γ, δ, ε που τις περικλείουν με κόκκινο, ροζ, πράσινο, κόκκινο, πράσινο χρώμα (είναι φανερό
πως δεν γίνεται με λιγότερα από τρία χρώματα) και την εξωτερική περιοχή του γραφήματος με θαλασσί. Άρα,
το Π12 είναι 4‒χρωματίσιμο (π) και επομένως ο χρωματικός αριθμός του Π20 είναι 4.

Ο χρωματικός αριθμός του Π12 είναι τουλάχιστον 3, αφού το Π12 δεν είναι διμερές. Βλέπουμε ένα
τέτοιο χρωματισμό με τρία χρώματα (πορτοκαλί, καφέ, βιολετί) στο παρακάτω σχήμα. Άρα ο χρωματικός
αριθμός του Π12 είναι 3 και επομένως το Π20 είναι 3‒χρωματίσιμο (π).

Το Π12 είναι κανονικό γράφημα με κάθε κορυφή του να έχει βαθμό 3. Το ότι είναι 3 ‒χρωματίσιμο
προκύπτει και από το σημαντικό θεώρημα του Brooks (1941), σύμφωνα με το οποίο, εάν ένα γράφημα είναι
απλό, συνεκτικό, μη πλήρες, με μεγαλύτερο βαθμό κορυφής β (≥3), τότε το Γ είναι β ‒χρωματίσιμο. Για την
απόδειξή του, που γίνεται με επαγωγή στον αριθμό των κορυφών και είναι παρόμοια με την απόδειξη της
πρότασης: «Κάθε επίπεδο γράφημα χωρίς βρόχους είναι 5‒χρωματίσιμο», συμβουλέψου το R. J. Wilson,
Introduction to Graph Theory.

Σχήμα 6.7
Συνεχίζοντας τον αναστοχασμό των χρωματισμών μας, παρατηρούμε ότι τα Π 4, Π6, Π12 είναι όλα κυβικά
γραφήματα (κανονικά, με όλες τις κορυφές να έχουν βαθμό 3). Από αυτά, όμως, μόνο το Π6 είναι 3 ‒
χρωματίσιμο (π). Σχετίζεται, άραγε αυτό, με το ότι κάθε περιοχή του ορίζεται από άρτιο αριθμό ακμών (κάτι
που δεν συμβαίνει με τα άλλα δύο); Μήπως μπορούμε να αποδείξουμε ότι Ένας κυβικός χάρτης είναι 3 ‒
χρωματίσιμος (π) ανν κάθε περιοχή του ορίζεται από άρτιο αριθμό ακμών;

Ναι. Οι περιοχές που περικλείουν οποιαδήποτε περιοχή π ενός κυβικού χάρτη που είναι 3 ‒
χρωματίσιμος (π) χρωματίζονται εναλλάξ με δύο χρώματα. Άρα, το πλήθος τους είναι άρτιο και επομένως το

Ψυχαγωγικά Μαθηματικά 141


σύνορο της π πρέπει να αποτελείται από άρτιο πλήθος ακμών. Αντίστροφα, ας πάρουμε το δυϊκό γράφημα
ενός κυβικού χάρτη του οποίου κάθε περιοχή ορίζεται από άρτιο πλήθος ακμών. Θα είναι γράφημα χωρίς
βρόχους, κάθε περιοχή του θα είναι τριγωνική και κάθε κορυφή του θα έχει άρτιο βαθμό, οπότε θα είναι
οϋλεριανό. Αλλά τότε, μπορούμε να χρωματίσουμε με δύο μόνο χρώματα όλες τις περιοχές του, ας πούμε με
κόκκινο και πράσινο. Το τέχνασμα για να χρωματίσουμε τις κορυφές του είναι να ξεκινήσουμε από μια από
τις περιοχές του που χρωματίσαμε, ας πούμε, κόκκινη, και να χρωματίσουμε τις κορυφές της με τα τρία
χρώματα 1, 2, 3, κατά τη φορά των δεικτών του ωρολογίου και χρωματίζοντας τις κορυφές κάθε περιοχής από
αυτές που την περικλείουν έτσι ώστε μια περιοχή να είναι χρωματισμένη κόκκινη ανν τα χρώματα 1, 2, 3
εμφανίζονται στη σειρά αυτή κατά τη φορά των δεικτών του ωρολογίου. Με αυτόν τον τρόπο μπορούμε
τελικά να χρωματίσουμε όλες τις κορυφές του δυϊκού γραφήματος.

6.3 Χρωματισμοί και χρωματικοί αριθμοί


I. Είναι φανερό ότι ο χρωματικός αριθμός του Κν είναι ν.

II. Είναι φανερό ότι ο χρωματικός αριθμός οποιουδήποτε δένδρου Τ ν (ν≥2) είναι 2.

III. Να ένας ζητούμενος χρωματισμός του γραφήματος του Petersen (Σχήμα 6.8). Είναι φανερό ότι δεν
μπορούμε να χρησιμοποιήσουμε μόνο δύο χρώματα, αν γειτονικές κορυφές πρέπει να είναι
χρωματισμένες με διαφορετικό χρώμα.

Σχήμα 6.8

IV. Είναι φανερό ότι οποιοδήποτε γράφημα–κύκλωμα με άρτιο αριθμό κορυφών έχει χρωματικό αριθμό 2
(χρωμάτισε τις περιττής τάξης κορυφές με κόκκινο και της άρτιας τάξης με μαύρο χρώμα εναλλάξ), ενώ
οποιοδήποτε γράφημα–κύκλωμα με περιττό αριθμό κορυφών έχει χρωματικό αριθμό 3 (χρωμάτισε τις
διαδοχικές κορυφές με τα χρώματα 1, 2, 0 κ.ο.κ.).

V. Είναι φανερό ότι σε έναν τροχό με άρτιο αριθμό κορυφών (Σχήμα 6.9 αριστερά) το υπογράφημα–
κύκλωμα του τροχού που προκύπτει «λησμονώντας» την κεντρική κορυφή έχει περιττό πλήθος κορυφών

142 Χατζηκυριάκου Κώστας


και άρα το υπογράφημα αυτό χρειάζεται 3 διαφορετικά χρώματα για τον χρωματισμό του και
αναγκαστικά η κεντρική κορυφή του τροχού πρέπει να χρωματιστεί με ένα τέταρτο διαφορετικό χρώμα.
Άρα, ο χρωματικός αριθμός είναι 4. Το δυϊκό γράφημα ενός τροχού είναι ολοφάνερα ο ίδιος τροχός, άρα
οι περιοχές του τροχού αυτού μπορούν να χρωματιστούν το λιγότερο με 4 διαφορετικά χρώματα.

VI. Είναι φανερό ότι σε έναν τροχό με περιττό αριθμό κορυφών (Σχήμα 6.9 δεξιά) το υπογράφημα–
κύκλωμα του τροχού που προκύπτει «λησμονώντας» την κεντρική κορυφή έχει άρτιο πλήθος κορυφών
και άρα το υπογράφημα αυτό χρειάζεται 2 διαφορετικά χρώματα για τον χρωματισμό του και
αναγκαστικά η κεντρική κορυφή του τροχού πρέπει να χρωματιστεί με ένα τρίτο διαφορετικό χρώμα.
Άρα, ο χρωματικός αριθμός είναι 3. Το δυϊκό γράφημα ενός τροχού είναι ολοφάνερα ο ίδιος τροχός, άρα
οι περιοχές του τροχού αυτού μπορούν να χρωματιστούν το λιγότερο με 3 διαφορετικά χρώματα.

Σχήμα 6.9

6.4 Χρωματίζοντας ακμές


I. Ο μικρότερος αριθμός διαφορετικών χρωμάτων με τα οποία μπορείς να χρωματίσεις τις ακμές του Κ 3
έτσι ώστε όλες οι γειτονικές ακμές να έχουν διαφορετικό χρώμα είναι 3.

Σχήμα 6.10

Ψυχαγωγικά Μαθηματικά 143


II. Ο μικρότερος αριθμός διαφορετικών χρωμάτων με τα οποία μπορείς να χρωματίσεις τις ακμές του Κ4
έτσι ώστε όλες οι γειτονικές ακμές να έχουν διαφορετικό χρώμα είναι 3 (Σχήμα 6.11).

Σχήμα 6.11

III. Ο μικρότερος αριθμός διαφορετικών χρωμάτων με τα οποία μπορείς να χρωματίσεις τις ακμές του Κ5
έτσι ώστε όλες οι γειτονικές ακμές να έχουν διαφορετικό χρώμα είναι 5 (Σχήμα 6.12).

Σχήμα 6.12

144 Χατζηκυριάκου Κώστας


IV. Ο μικρότερος αριθμός διαφορετικών χρωμάτων με τα οποία μπορείς να χρωματίσεις τις ακμές του Κ 6
έτσι ώστε όλες οι γειτονικές ακμές να έχουν διαφορετικό χρώμα είναι 5 (Σχήμα 6.13).

Σχήμα 6.13

V. Βλέπουμε ότι αν το γράφημα είναι πλήρες σε άρτιο αριθμό κορυφών ν, τότε τα διαφορετικά χρώματα που
απαιτούνται είναι ν−1, ενώ αν είναι πλήρες σε περιττό αριθμό κορυφών ν, τότε απαιτούνται ν
διαφορετικά χρώματα. Αν ονομάσουμε χρωματικό δείκτη τον ελάχιστο αριθμό χρωμάτων που
απαιτούνται για να χρωματιστούν οι ακμές ενός γραφήματος έτσι ώστε όλες οι γειτονικές ακμές να έχουν
διαφορετικό χρώμα χρωματικό δείκτη του γραφήματος, η εύλογη εικασία είναι η ακόλουθη:
Ο χρωματικός δείκτης του πλήρους γραφήματος Κν είναι ν αν ο ν είναι περιττός αριθμός (>1)
και ν−1 αν ο ν είναι άρτιος.

Αν ν είναι περιττός αριθμός (>1), δες το Κ ν ως κανονικό ν-γωνο με όλες τις διαγωνίους του
σχεδιασμένες, χρωμάτισε με ν διαφορετικά χρώματα τις πλευρές του και τις διαγωνίους του έτσι ώστε
κάθε διαγώνιος να είναι ομόχρωμη της παράλληλης, με αυτήν, πλευράς. Είναι φανερό ότι ο μεγαλύτερος
𝜈−1 𝜈−1
αριθμός ακμών με το ίδιο χρώμα είναι
2
, οπότε το πλήρες γράφημα Κν έχει το πολύ ∙ χ΄(Κν)
2
𝜈−1
ακμές, αλλά γνωρίζουμε ότι οι ακμές του είναι ακριβώς ∙ ν, άρα χ΄(Κν) = ν. Αν ν άρτιος (ν > 3), τότε
2
χρωματίζουμε με ν-1 διαφορετικά χρώματα το πλήρες Κν-1 υπογράφημά του που έχει περιττό πλήθος
κορυφών με τον τρόπο που μόλις είδαμε. Σε κάθε κορυφή, τότε, υπάρχει ένα διαφορετικό χρώμα από
αυτά τα ν-1 αχρησιμοποίητο, οπότε με αυτά χρωματίζουμε τις ακμές που συνδέουν την κορυφή του Κν
που δεν ήταν ανάμεσα στις κορυφές του υπογραφήματος Κ ν-1 με καθεμία από τις κορυφές του Κν-1. Το Κ2
το είδαμε ήδη και προφανώς έχει χρωματικό δείκτη 1.

Ψυχαγωγικά Μαθηματικά 145


VI. Ας υπολογίσουμε τον χρωματικό δείκτη των πλατωνικών γραφημάτων.
Ο χρωματικός δείκτης του Π4 είναι προφανώς 3. Ένας τέτοιος χρωματισμός φαίνεται στο Σχήμα 6.14.

Σχήμα 6.14
Όπως γνωρίζουμε, χρειαζόμαστε τέσσερα διαφορετικά χρώματα για να χρωματίσουμε τις περιοχές του
Π4. Είχαμε χρωματίσει την εξωτερική περιοχή με πράσινο χρώμα (0,1) και τις εσωτερικές με κόκκινο
(1,0), πορτοκαλί (1,1) και μπλε (0,0). Τα αθροίσματα (mod2) γειτονικών περιοχών δίνουν τρία
αποτελέσματα (1,1), (0,1) και (1,0) με τα οποία μπορούμε να χρωματίσουμε τις ακμές έτσι ώστε κάθε
ακμή να χρωματίζεται με το χρώμα που αντιστοιχεί στο «άθροισμα» των χρωμάτων με τα οποία είναι
χρωματισμένες οι περιοχές τις οποίες χωρίζει.

(0,1) + (1,0) = (1,1) (0,1) + (1,1) = (1,0) (0,1) + (0,0) = (0,1)

(1,0) + (1,1) = (0,1) (1,1) + (0,0) = (1,1) (0,0) + (1,0) = (1,0)

Όπως γνωρίζουμε, χρειαζόμαστε τρία διαφορετικά χρώματα για να χρωματίσουμε τις περιοχές του
Π6. Είχαμε χρωματίσει την εξωτερική περιοχή με πράσινο χρώμα (0,1), τις περιφερειακές εσωτερικές με
κόκκινο (1,0) και μπλε (0,0) και την κεντρική εσωτερική με πράσινο χρώμα (0,1). Τα αθροίσματα (mod2)
(1 + 0 = 1, 1 + 1= 0) γειτονικών περιοχών (δίνουν τρία αποτελέσματα (1,1), (0,1) και (1,0) με τα οποία
μπορούμε να χρωματίσουμε τις ακμές έτσι ώστε κάθε ακμή να χρωματίζεται με το χρώμα που αντιστοιχεί
στο «άθροισμα» των χρωμάτων με τα οποία είναι χρωματισμένες οι περιοχές τις οποίες χωρίζει (βλ.
Σχήμα 6.15).

(0,1) + (0,0) = (0,1) (0,1) + (1,0) = (1,1) (0,0) + (1,0) = (1,0)

Επομένως, ο χρωματικός δείκτης του Π6 είναι επίσης 3.

Σχήμα 6.15

146 Χατζηκυριάκου Κώστας


Ο παρακάτω χρωματισμός των ακμών του δωδεκαέδρου (Σχήμα 6.16) προκύπτει με την ίδια μέθοδο, και
βλέπουμε ότι ο χρωματικός δείκτης του είναι 3.

Σχήμα 6.16
Αν αναστοχαστούμε τη μέθοδο με την οποία από τον χρωματισμό των περιοχών των προηγούμενων τριών
πλατωνικών γραφημάτων περάσαμε στον χρωματισμό των ακμών τους, εύκολα θα κατανοήσουμε ότι με τη
βοήθεια της μεθόδου αυτής μπορούμε να αποδείξουμε την εξής πρόταση: Αν κάθε γράφημα είναι 4-
χρωματίσιμο (π), τότε κάθε κυβικό επίπεδο γράφημα έχει χρωματικό δείκτη 3. Αποδεικνύεται και το
αντίστροφο (Βλ. R. J. Wilson, Introduction to Graph Theory). Τη σύνδεση αυτή, του χρωματισμού των
κορυφών (χαρτών) με τον χρωματισμό των ακμών, την πρωτοέκανε ο Peter Guthrie Tait (1831-1901).

Διαδικτυακή δραστηριότητα: Συμβουλέψου έναν έγκυρο ιστότοπο και γράψε ένα σύντομο βιογραφικό
σημείωμα για τον Peter Guthrie Tait.

Είναι φανερό (βλ. Σχήμα 6.17) ότι ο χρωματικός δείκτης του Π8 είναι 4.

Σχήμα 6.17

Ψυχαγωγικά Μαθηματικά 147


Είναι επίσης φανερό (βλ. Σχήμα 6.18) ότι ο χρωματικός δείκτης του Π12 είναι 5.

Σχήμα 6.18

Τέλος, ο χρωματικός δείκτης του γραφήματος του Petersen είναι 4 (βλ. Σχήμα 6.19).

Σχήμα 6.19

148 Χατζηκυριάκου Κώστας


Βιβλιογραφία/Αναφορές

Bigg, N. L., Lloyd, K. E., & Wilson, R. J. (1999). Graph theory 1736-1936. Clarendon Press.
Bondecka-Krzykowska, I. (2004). The four-color theorem and its consequences for the philosophy of
mathematics. Annales UMCS Informatica AI 2, 5-14.
Courant, R., & Robbins, H. (Revised by Ian Stewart). (1996). What is Mathematics (2nd edition). Oxford
Paperbacks.
Polya, G. (1998). Πώς να το λύσω. Εκδόσεις Καρδαμίτσα.
Tymoczko, T. (1986). The Four Colour Problem and its Philosophical Significance. In T. Tymoczko (Ed.)
New Directions in the Philosophy of Mathematics (pp. 243-266). BIRKHÄUSER.
Wilson, R. J. (1979). Introduction to Graph Theory (2nd edition). Longman.

Ψυχαγωγικά Μαθηματικά 149


Κεφάλαιο 7
Γοητευτικά τετράγωνα, Ι

Σύνοψη
Η σπαζοκεφαλιά του μαγικού τετραγώνου.

Προαπαιτούμενη γνώση
Γραφή, ανάγνωση και αριθμητική.

Ψυχαγωγικά Μαθηματικά 151


7.1 Μαγικά τετράγωνα (περιττής τάξης)
I. Παρατήρησε προσεκτικά την παρακάτω εικόνα. Έπειτα, προσπάθησε να την περιγράψεις λεπτομερώς
(να αναφερθείς στα δομικά στοιχεία της και στις δομικές σχέσεις που υπάρχουν ανάμεσά τους).
Αναζήτησε την εικόνα αυτή στο διαδίκτυο και μάθε ό,τι μπορείς γι’ αυτήν.

Εικόνα 7.1
https://commons.wikimedia.org/wiki/File:Magic_square_Lo_Shu.png

152 Χατζηκυριάκου Κώστας


II. Μπορείς να τοποθετήσεις τα εννέα ψηφία 1, 2, 3, 4, 5, 6, 7, 8, 9 στα κελιά του παρακάτω τετραγώνου έτσι
ώστε το άθροισμα των ψηφίων σε κάθε στήλη, σε κάθε γραμμή και κάθε διαγώνιο να είναι το ίδιο;

Ψυχαγωγικά Μαθηματικά 153


III. Μπορείς να τοποθετήσεις εννέα άρτιους θετικούς ακέραιους αριθμούς στα κελιά του παρακάτω
τετραγώνου έτσι ώστε το άθροισμα των ψηφίων σε κάθε στήλη, σε κάθε γραμμή και κάθε διαγώνιο να
είναι το ίδιο;

154 Χατζηκυριάκου Κώστας


IV. Μπορείς να τοποθετήσεις εννέα περιττούς θετικούς ακέραιους αριθμούς στα κελιά του παρακάτω
τετραγώνου έτσι ώστε το άθροισμα των ψηφίων σε κάθε στήλη, σε κάθε γραμμή και κάθε διαγώνιο να
είναι το ίδιο;

Ψυχαγωγικά Μαθηματικά 155


V. Μπορείς να τοποθετήσεις επτά θετικούς ακέραιους αριθμούς στα κελιά των παρακάτω τετραγώνων έτσι
ώστε το άθροισμα των ψηφίων σε κάθε στήλη, σε κάθε γραμμή και κάθε διαγώνιο να είναι το ίδιο;

156 Χατζηκυριάκου Κώστας


VI. Μπορείς να τοποθετήσεις οκτώ θετικούς ακέραιους αριθμούς στα κελιά των παρακάτω τετραγώνων έτσι
ώστε το άθροισμα των ψηφίων σε κάθε στήλη, σε κάθε γραμμή και κάθε διαγώνιο να είναι το ίδιο;

a. 8 b. 15

24

Ψυχαγωγικά Μαθηματικά 157


7.2 Μαγικά τετράγωνα (άρτιας τάξης)
I. Παρατήρησε προσεκτικά το παρακάτω χαρακτικό. Έπειτα, προσπάθησε να περιγράψεις λεπτομερώς τι
βλέπεις. Τι λες ότι αναπαριστάνει; Παρατήρησε το τετράγωνο με τους αριθμούς πάνω δεξιά. Τι ιδιότητες
έχει; Αναζήτησε την εικόνα αυτή στο διαδίκτυο και μάθε ό,τι μπορείς γι’ αυτήν και για τον χαράκτη της.

Εικόνα 7.2
https://commons.wikimedia.org/wiki/File:D%C3%BCrer_Melancholia_I.jpg

158 Χατζηκυριάκου Κώστας


II. Να τοποθετήσεις τέσσερις φυσικούς αριθμούς στα κελιά του παρακάτω τετραγώνου έτσι ώστε το
άθροισμα των ψηφίων σε κάθε στήλη, σε κάθε γραμμή και κάθε διαγώνιο να είναι το ίδιο.

III. Να τοποθετήσεις τους φυσικούς αριθμούς 1, 2, 3, 4, 5, 6, 7, 8, 9, …, 16 στα κελιά του παρακάτω


τετραγώνου έτσι ώστε το άθροισμα των ψηφίων σε κάθε στήλη, σε κάθε γραμμή και κάθε διαγώνιο να
είναι το ίδιο.

Ψυχαγωγικά Μαθηματικά 159


Απαντήσεις στα ερωτήματα του 7ου Κεφαλαίου

7.1 Μαγικά τετράγωνα (περιττής τάξης)


I. Μπορούμε να παρατηρήσουμε ότι στην τετραγωνική διάταξη που βλέπουμε υπάρχει άρτιο πλήθος
μαύρων κουκκίδων σε καθεμία από τις τέσσερις κορυφές (4, 2, 6, 8) και περιττό πλήθος λευκών
κουκκίδων σε κάθε πλευρά (9, 7, 1, 3), καθώς και στο κέντρο (5). Πλάι σε κάθε πλευρά υπάρχει κινεζικό
(;) ιδεόγραμμα. Για να μάθεις για την εικόνα αυτή, μπορείς να ξεκινήσεις από τον διαδικτυακό τόπο
https://en.wikipedia.org/wiki/Lo_Shu_Square

II. Το άθροισμα όλων των ψηφίων 1, 2, 3, 4, 5, 6, 7, 8, 9 ισούται (και σύμφωνα με τον τύπο του Gauss) με 1
9 ∙ 10
+ 2 + 3 + 4 + 5 + 6 + 7 + 8 + 9 = 45 ( = ).
2
Αφού τα ψηφία κάθε γραμμής π.χ. πρέπει να έχουν το ίδιο άθροισμα, άρα το άθροισμα κάθε γραμμής
45
(και επομένως και κάθε στήλης και κάθε διαγωνίου) είναι 3
= 15. Ο αριθμός 15 λέγεται σταθερά του
μαγικού τετραγώνου.

Αν συμβολίσουμε με aij τον αριθμό που βρίσκεται στο κελί της γραμμής υπ’ αριθμ. i και στήλης υπ’
αριθμ. j, τότε το μαγικό τετράγωνο με όλα τα κελιά συμπληρωμένα θα είναι το παρακάτω.

𝑎11 𝑎12 𝑎13

𝑎21 𝑎22 𝑎23

𝑎31 𝑎32 𝑎33

Φυσικά, 1 ≤ 𝑎𝑖𝑗 ≤ 9 (i, j = 1, 2, 3) και ∑3𝑖=1 𝑎𝑖1 = ∑3𝑖=1 𝑎𝑖2 = ∑3𝑖=1 𝑎𝑖3 = ∑3𝑗=1 𝑎1𝑗 = ∑3𝑗=1 𝑎2𝑗 = ∑3𝑗=1 𝑎3𝑗 =
𝑎11+ 𝑎22+ 𝑎33 = 𝑎31+ 𝑎22+ 𝑎13 = 15.

Αλλά τότε, 𝑎22 = 15 – (𝑎11 + 𝑎33) = 15 – (𝑎13 + 𝑎31) = 15 – (𝑎21 + 𝑎23) = 15 – (𝑎12+ 𝑎32), οπότε 4 ∙ 𝑎22 = 4 ∙
15 – (𝑎11 + 𝑎33 + 𝑎13 + 𝑎31+ 𝑎21+ 𝑎23+ 𝑎12+ 𝑎32) = 4 ∙ 15 – (45 – 𝑎22). Οπότε 3 ∙ 𝑎22 = 15 και 𝑎22 = 5.

Αν 𝑎11 = 1, τότε 𝑎33 = 9, οπότε 𝑎13 = 2 ή 4, οπότε 𝑎12 > 9, άτοπο.

Αν 𝑎11 = 2, τότε 𝑎33 = 8, οπότε 𝑎13 = 1 ή 3 ή 4 ή 6 οπότε 𝑎12 = 12 ή 10 ή 9 ή 7 αντίστοιχα, και επομένως η
πρώτη γραμμή είναι 2, 9, 4 ή 2, 7, 6. Αν είναι 2, 9, 4, τότε η δεύτερη γραμμή είναι 7, 5, 3 και η τρίτη γραμμή
είναι 6, 1, 8. Ενώ, αν η πρώτη γραμμή είναι η 2, 7, 6, τότε η δεύτερη γραμμή είναι η 9, 5, 1 και η τρίτη
γραμμή είναι 4, 3, 8.

Αν 𝑎11 = 3, τότε 𝑎33 = 7, και καμία επιλογή για τα υπόλοιπα κελιά δεν στέκει.

Ψυχαγωγικά Μαθηματικά 161


Αν 𝑎11 = 4, τότε 𝑎33 = 6, οπότε 𝑎13 μπορεί να είναι 2 ή 8, οπότε 𝑎12 = 9 ή 3 αντίστοιχα, οπότε η πρώτη
γραμμή είναι 4, 9, 2 ή 4, 3, 8. Αν η πρώτη γραμμή είναι 4, 9, 2, τότε η δεύτερη γραμμή είναι 3, 5, 7 και η
τρίτη 8, 1, 6. Αν η πρώτη γραμμή είναι 4, 3, 8, τότε η δεύτερη είναι 9, 5, 1 και η τρίτη 2, 7, 6.

Αν 𝑎11 = 6, τότε 𝑎33 = 4, και μπορεί να ισχύει μόνο 𝑎13 = 2 ή 8. Τότε, η πρώτη γραμμή είναι 6, 7, 2 ή 6, 1, 8.
Αντίστοιχα, η δεύτερη γραμμή είναι 1, 5, 9 ή 7, 5, 3 και η τρίτη γραμμή είναι αντίστοιχα 8, 3, 4 ή 2, 9, 4.

Αν 𝑎11 = 7, τότε 𝑎33 = 3, και καμία επιλογή για τα υπόλοιπα κελιά δεν στέκει.

Αν 𝑎11 = 8, τότε 𝑎33 = 2, τότε 1 = 𝑎12 ή 1 = 𝑎21. Στην πρώτη περίπτωση, η πρώτη γραμμή είναι 8, 1, 6, η
δεύτερη είναι 3, 5, 7 και η τρίτη 4, 9, 2. Στη δεύτερη περίπτωση, η δεύτερη γραμμή είναι 1, 5, 9, η τρίτη είναι
6, 7, 2 και η πρώτη 8, 3, 4.

Αν 𝑎11 = 9, τότε 𝑎33 = 1 και η τρίτη στήλη είναι φανερό πως δεν μπορεί να συμπληρωθεί.

Είναι φανερό ότι δεν μπορούν να υπάρχουν άλλες λύσεις εκτός από τις παραπάνω οκτώ. Αν τις
παρατηρήσουμε, γεννιέται ένα ενδιαφέρον ερώτημα: Είναι όντως διαφορετικές ή μήπως πρόκειται κατ’
ουσίαν για μία λύση η οποία υπόκειται σε ποικίλους μετασχηματισμούς;

8 1 6 4 3 8 2 9 4 6 7 2
3 5 7 9 5 1 7 5 3 1 5 9
4 9 2 2 7 6 6 1 8 8 3 4

8 3 4 2 7 6 6 1 8 4 9 2
1 5 9 9 5 1 7 5 3 3 5 7
6 7 2 4 3 8 2 9 4 8 1 6

Παρατηρούμε ότι αν βρούμε μία από τις παραπάνω λύσεις, οι άλλες εφτά δεν είναι παρά εκείνες που
προκύπτουν περιστρέφοντας το τετράγωνο κατά τη φορά των δεικτών του ωρολογίου 90, 180, 270 μοίρες ή
σχηματίζοντας τα συμμετρικά ως προς τις δύο διαγωνίους τετράγωνα ή σχηματίζοντας τα συμμετρικά ως
προς τις δύο διαμέσους τετράγωνα. Οι λύσεις αυτές είναι, επομένως, «ισοδύναμες». Η πρώτη λύση στην
πρώτη σειρά είναι το κανονικό μαγικό τετράγωνο 3 × 3.

Το κανονικό μαγικό τετράγωνο 3 × 3 μπορεί να βρεθεί με τη βοήθεια του αλγορίθμου που γενικεύεται
φυσικά σε κάθε μαγικό τετράγωνο ν × ν, ν = περιττός αριθμός, στο οποίο θέλουμε να τοποθετήσουμε τους
διαδοχικούς φυσικούς αριθμούς 1, 2, 3, …, ν2. Τοποθέτησε τον αριθμό 1 στο μεσαίο κελί της πρώτης σειράς.
Τοποθέτησε τον επόμενό του στο κελί της αμέσως, από πάνω, σειράς, στο οποίο δείχνει η διαγώνιος του
κελιού που μόλις γέμισες. Επειδή τέτοιο κελί δεν υπάρχει, φαντάσου ότι η επίπεδη επιφάνεια του μαγικού
τετραγώνου κυρτώνει σε κυλινδρική γύρω από την τρίτη σειρά, οπότε το κελί αυτό ταυτίζεται με το κάτω
δεξιά κελί του μαγικού τετραγώνου. Το αμέσως επάνω «βορειανατολικό» κελί πάλι δεν υπάρχει, οπότε
κυρτώνοντας την επίπεδη επιφάνεια του μαγικού τετραγώνου γύρω από την πρώτη στήλη, τοποθετούμε το 3
στο μεσαίο κελί της πρώτης στήλης. Η ίδια «βορειανατολική» κίνηση, τώρα, οδηγεί σε κατειλημμένο κελί,
οπότε τοποθετούμε το 4 στο ακριβώς κάτω από το κελί όπου βάλαμε το 3. Ο «βορειανατολικός» κανόνας
ισχύει για τους επόμενους δύο αριθμούς: στη γωνία εφαρμόζουμε τον κανόνα «βάλε τον αριθμό στο ακριβώς
από κάτω κελί», το 8 τοποθετείται στο πρώτο κελί της πρώτης σειράς λόγω της κύρτωσης γύρω από την
πρώτη στήλη και το 9 στο μεσαίο κελί της τρίτης σειράς λόγω της κύρτωσης γύρω από την τρίτη σειρά.

Ο αλγόριθμος αυτός ήταν γνωστός στο Σιάμ τον 17ο αιώνα και τον έφερε στη Δύση ο Γάλλος
διπλωμάτης Simon De La Loubère. Την εφαρμογή του τη βλέπουμε αμέσως παρακάτω.

162 Χατζηκυριάκου Κώστας


9 2

8 1 6 8

3 5 7 3

4 9 2

Ακολουθώντας τον αλγόριθμο αυτό, ας βρούμε την κανονική μορφή του μαγικού τετραγώνου 5 × 5.

18 25 2 9

17 24 1 8 15 17

23 5 7 14 16 23

4 6 13 20 22 4

10 12 19 21 3 10

11 18 25 2 9

Ακολουθώντας πάντα την προτροπή του Polya, ας αναστοχαστούμε το κανονικό μαγικό τετράγωνο 3 × 3.
Παρατηρούμε ότι οι εννέα αριθμοί σχετίζονται με τις σημειωμένες, στο παρακάτω τετράγωνο,
προσθαφαιρέσεις.

8=5+3 1=5–(3+1) 6=5+1

3=5–(3–1) 5 7=5+(3–1)

4=5–1 9=5+(3+1) 2=5–3

Γίνεται τότε φανερό ότι μπορούμε να σχηματίζουμε μαγικά τετράγωνα 3 × 3, επιλέγοντας οποιαδήποτε
τριάδα (Κ, α, β) φυσικών αριθμών για την οποία 0 < α < β < Κ – α και β ≠ 2∙α και γεμίζοντας τα κελιά με τον
τρόπο που δείχνει το παρακάτω μαγικό τετράγωνο, όπου Κ = 5, α = 1 και β = 3.

Κ+β Κ–(β+α) Κ+α

Κ–(β–α) Κ Κ+(β–α)

Κ–α Κ+(β+α) Κ–β

Ψυχαγωγικά Μαθηματικά 163


Αν, μάλιστα, εφαρμόσουμε τη μέθοδο αυτή με οποιαδήποτε τριάδα (Κ+α+β, α, β) όπου 0 < α < β και β ≠ 2∙α,
τότε παίρνουμε το παρακάτω μαγικό τετράγωνο.

Κ+α+2∙β Κ Κ+2∙α+β

Κ+2∙α Κ+α+β Κ+2∙β

Κ+β Κ+2∙α+2∙β Κ+α

Είναι δυνατόν τώρα να παρατηρήσουμε ότι το τετράγωνο αυτό μπορεί να προκύψει από την κανονική μορφή
του μαγικού τετραγώνου 3 × 3, αν στη θέση των όρων της αριθμητικής ακολουθίας 1, 2, 3, 4, 5, 6, 7, 8, 9
τοποθετήσουμε αντίστοιχα τους αριθμούς Κ, Κ+α, Κ+2∙α, Κ+β, Κ+α+β, Κ+2∙α+β, Κ+2∙β, Κ+α+2∙β,
Κ+2∙α+2∙β. Διακρίνουμε εδώ αριθμητικές ακολουθίες που εκτείνονται από αριστερά προς τα δεξιά (με
διαφορά όρων α) και από πάνω προς τα κάτω (με διαφορά όρων β).

Κ → Κ+α → Κ+2∙α → … → Κ+(ν–1)∙α

↓ ↓ ↓ ↓

Κ+β → Κ+α+β → Κ+2∙α+β → … → Κ+(ν–1)∙α +β

↓ ↓ ↓
↓ ↓

Κ+2∙β → Κ+α+2∙β → Κ+2∙α+2∙β → … → Κ+(ν–1)∙α+2∙β

↓ ↓ ↓ ↓ ↓

… → … → … → … → …

↓ ↓ ↓ ↓ ↓

Κ+(ν–1)∙β → Κ+α+(ν–1)∙β → Κ+2∙α+(ν–1)∙β → … → Κ+(ν–1)∙α +(ν–1)∙β

Είναι φανερό πως με τον ίδιο τρόπο μπορούμε, ξεκινώντας από το κανονικό μαγικό τετράγωνο ν × ν και
αντικαθιστώντας με τον ίδιο τρόπο τους αριθμούς 1, 2, …, ν 2 με τους αριθμούς Κ, Κ+α, …, Κ+(ν–1)∙α, Κ+β, .
𝜈−1
. ., Κ+(ν–1)∙α+(ν–1)∙β, να σχηματίσουμε νέα μαγικά τετράγωνα ν × ν με σταθερά ν∙[Κ+ ∙(α+β)].
2

Αξίζει, τέλος, να παρατηρήσουμε ότι:

• αν προσθέσουμε τον ίδιο φυσικό μ σε όλους τους αριθμούς ενός μαγικού τετραγώνου ν × ν με
σταθερά Σ, προκύπτει ένα νέο μαγικό τετράγωνο με σταθερά Σ + ν∙μ,
• αν πολλαπλασιάσουμε με τον ίδιο φυσικό μ όλους τους αριθμούς ενός μαγικού τετραγώνου ν × ν με
σταθερά Σ, προκύπτει ένα νέο μαγικό τετράγωνο με σταθερά μ∙Σ.

III. Το παρακάτω μαγικό τετράγωνο είναι μια απάντηση στο ερώτημα.


16 2 12

6 10 14

8 18 4

164 Χατζηκυριάκου Κώστας


IV. Το παρακάτω μαγικό τετράγωνο είναι μια απάντηση στο ερώτημα.
17 3 13

7 11 15

9 19 5

V. Μία λύση είναι το μαγικό τετράγωνο που προκύπτει από το κανονικό όταν αυτό στραφεί 90 μοίρες
κατά τη φορά των δεικτών του ωρολογίου.
4 3 8

9 5 1

2 7 6

Μια άλλη λύση μπορούμε να βρούμε με τη μέθοδο των αριθμητικών ακολουθιών ως εξής:

3 → 6 → 9

↓ ↓ ↓

5 → 8 → 11
↓ ↓ ↓

7 → 10 → 13

Τοποθετούμε τους αριθμούς 3, 6, 9, 5, 8, 11, 7, 10, 13 στη θέση των 1, 2, 3, 4, 5, 6, 7, 8, 9 στο


κανονικό μαγικό τετράγωνο.

10 3 11

9 8 7

5 13 6

VI. Με την ίδια μέθοδο, να ένα μαγικό τετράγωνο με το 8 στην θέση (1,1). Οι όροι των δύο αριθμητικών
ακολουθιών είναι οι 6, 8, 10, 9, 11, 13, 12, 14, 16. Τους τοποθετούμε στο κανονικό μαγικό τετράγωνο στη
θέση των 1, 2, 3, 4, 5, 6, 7, 8, 9 αντίστοιχα και παίρνουμε το μαγικό τετράγωνο που είναι συμμετρικό ως
προς τη μία διαγώνιο. Μια λύση για το (b.) βλέπουμε επίσης παρακάτω.

a. 14 6 13 8 12 13 b. 9 30 15
10 11 12 16 11 6 24 18 12
9 16 8 9 10 14 21 6 27

Ψυχαγωγικά Μαθηματικά 165


7.1 Μαγικά τετράγωνα (άρτιας τάξης)
I. Το τετράγωνο που υπάρχει στο χαρακτικό είναι μαγικό τετράγωνο άρτιας τάξης (4×4). Το χαρακτικό
είναι η Μελαγχολία (1514) του Albrecht Dürer (1471-1518). Για να μάθεις για το περίφημο χαρακτικό,
μπορείς να ξεκινήσεις από τον διαδικτυακό τόπο https://en.wikipedia.org/wiki/Melencolia_I, και για να
μάθεις για τον χαράκτη του από τον διαδικτυακό τόπο
https://en.wikipedia.org/wiki/Albrecht_D%C3%BCrer

II. Μαγικό τετράγωνο 2×2 είναι φανερό πως δεν μπορεί να σχηματιστεί, αφού αν υπήρχε, θα ίσχυε 𝑎11+
𝑎12 = 𝑎11+ 𝑎21 = 𝑎12+ 𝑎22= 𝑎21+ 𝑎22= 𝑎11+ 𝑎22= 𝑎12+ 𝑎21. Αλλά τότε, όλοι αυτοί οι αριθμοί είναι
ίσοι με μηδέν, άτοπο.

𝑎11 𝑎12

𝑎21 𝑎22

III. Να δύο μαγικά τετράγωνα 4×4.

1 15 14 4 16 3 2 13

12 6 7 9 5 10 11 8

8 10 11 5 9 6 7 12

13 3 2 16 4 15 14 1

Το αριστερό το βρήκαμε με την εξής μέθοδο: Αριθμούμε τα κελιά του τετραγώνου από αριστερά στα
δεξιά ως εξής: 1η σειρά: 1, 2, 3, 4. 2η σειρά: 5, 6, 7, 8. 3η σειρά: 9, 10, 11, 12. 4η σειρά: 13, 14, 15, 16.
Σημειώνουμε τον αύξοντα αριθμό της αρίθμησής τους μόνο στα κελιά που βρίσκονται πάνω σε μία από
τις δύο διαγωνίους του τετραγώνου. Στη συνέχεια, αριθμούμε ξανά τα κελιά από δεξιά προς τα αριστερά
ως εξής: 4η σειρά: 1, 2, 3, 4. 3η σειρά: 5, 6, 7, 8. 2η σειρά: 9, 10, 11, 12. 1η σειρά: 13, 14, 15, 16.
Σημειώνουμε τον αύξοντα αριθμό της αρίθμησής τους μόνο στα κελιά που δεν είχαν συμπληρωθεί στο
πρώτο βήμα. Αξίζει να πούμε εδώ ότι με τη μέθοδο αυτή μπορούμε να σχηματίσουμε μαγικά τετράγωνα
4∙ν × 4∙ν, για ν = 1, 2, …

Αν εκμεταλλευτούμε τις γεωμετρικές συμμετρικές ιδιότητες του τετραγώνου (αντικατοπτρισμός ως


προς την κατακόρυφη διάμεσο της 1ης και της 4ης στήλης και κατόπιν αντικατοπτρισμός ως προς την
οριζόντια διάμεσο των πρώτων δύο σειρών και των δύο τελευταίων), βρίσκουμε το δεξί που είναι το
περίφημο μαγικό τετράγωνο που βλέπουμε στο χαρακτικό της πρώτης δραστηριότητας.

166 Χατζηκυριάκου Κώστας


Βιβλιογραφία/Αναφορές

Bolt, B. (1990). Μαθηματικές Σπαζοκεφαλιές (3 τόμοι). Εκδόσεις Κάτοπτρο.


Hunter, J. A. H., & Madachy, J. S. (1975). Mathematical Diversions. Dover Publications Inc.

Ψυχαγωγικά Μαθηματικά 167


Κεφάλαιο 8
Γοητευτικά τετράγωνα, ΙΙ

Σύνοψη
Οι σπαζοκεφαλιές του λατινικού τετραγώνου, του Sudokou και του Kenken.

Προαπαιτούμενη γνώση
Γραφή, ανάγνωση και αριθμητική.

Ψυχαγωγικά Μαθηματικά 169


8.1 Λατινικά τετράγωνα
I. Τετράγωνα με ν × ν κελιά που περιέχουν τα ν σύμβολα (συνήθως στοιχεία του {1, 2, …, ν}), έτσι ώστε
καθένα από αυτά να εμφανίζεται ακριβώς μία φορά σε κάθε σειρά και κάθε στήλη λέγονται λατινικά
τετράγωνα τάξης ν. Είναι φανερό πως τα μαγικά τετράγωνα τάξης ν με στοιχεία από το σύνολο {1, 2,…,
ν} είναι λατινικά τετράγωνα, καθώς επίσης ότι υπάρχουν αριθμητικά λατινικά τετράγωνα που δεν είναι
μαγικά.
Γράψε ένα λατινικό τετράγωνο τάξης δύο, ένα λατινικό τετράγωνο τάξης 3, ένα λατινικό τετράγωνο
τάξης 4, ένα λατινικό τετράγωνο τάξης ν. Να μην είναι κανένα μαγικό!

Μπορείς να γράψεις λατινικά τετράγωνα τάξης 3, 4, 5 που να είναι διαγώνια, δηλαδή και στις
διαγωνίους να μην υπάρχουν επαναλήψεις;

170 Χατζηκυριάκου Κώστας


II. Μπορείς να τοποθετήσεις τις 16 φιγούρες μιας τράπουλας (Βαλές, Ντάμα, Παπάς, Γελωτοποιός) στα 16
κελιά ενός τετραγώνου έτσι ώστε σε κάθε γραμμή και κάθε στήλη να μην υπάρχουν δύο χαρτιά με την
ίδια φιγούρα ή το ίδιο είδος (1 = μπαστούνι, 2 = σπαθί, 3 = καρδιά, 4 = καρό);

Ψυχαγωγικά Μαθηματικά 171


III. Δύο λατινικά τετράγωνα με στοιχεία από το {1, 2, …, ν} λέγονται ορθογώνια ανν μπορούν να
τοποθετηθούν το ένα πάνω στο άλλο έτσι ώστε καθένα από τα ν 2 διατεταγμένα ζευγάρια που προκύπτουν
να εμφανίζεται ακριβώς μία φορά. Ένα τέτοιο ζευγάρι λατινικών τετραγώνων λέγεται ελληνολατινικό
ζευγάρι τάξης ν.
Μια ακολουθία από λ λατινικά τετράγωνα τάξης ν λέγεται αμοιβαία ορθογώνια ανν κάθε ζευγάρι
τετραγώνων της ακολουθίας είναι ελληνολατινικό.

Γράψε ένα ελληνολατινικό ζευγάρι τάξης 3, ένα ελληνολατινικό ζευγάρι τάξης 4, ένα ελληνολατινικό
ζευγάρι τάξης 5, ένα ελληνολατινικό ζευγάρι τάξης 6.

172 Χατζηκυριάκου Κώστας


IV. Από ένα οποιοδήποτε ζευγάρι διαγώνιων ελληνολατινικών τετραγώνων τάξης ν μπορεί να σχηματιστεί
μαγικό τετράγωνο τάξης ν. Μπορείς να βρεις πώς;

Ψυχαγωγικά Μαθηματικά 173


V. Μπορείς να συμπληρώσεις κατάλληλα τα άδεια κελιά των παρακάτω τετραγώνων ώστε να γίνουν
λατινικά;

4 1

2 1

3 4 1

4 1 2 2

174 Χατζηκυριάκου Κώστας


8.2 Οι σπαζοκεφαλιές Sudokou και Kenken
Tις τελευταίες δεκαετίες, δύο δημοφιλείς σπαζοκεφαλιές είναι η Sudokou και η Kenken. Και οι δύο απαιτούν
τη συμπλήρωση ενός, μερικά συμπληρωμένου, λατινικού τετραγώνου για το οποίο ορίζονται πρόσθετες
ιδιότητες.

Ειδικότερα στη σπαζοκεφαλιά Sudokou, το τετράγωνο είναι τάξης 9 (9×9) και χωρισμένο σε εννέα
τομείς που είναι τετραγωνίδια τάξης 3 (3×3). Τα κελιά του τετραγώνου πρέπει να γεμίσουν με τα ψηφία 1, 2,
3, . . . , 9 έτσι ώστε κάθε ψηφίο να εμφανίζεται ακριβώς μία φορά σε κάθε γραμμή και κάθε στήλη, ενώ σε
κάθε τομέα πρέπει να εμφανίζονται και τα εννέα ψηφία.

Ένα ενδιαφέρον μαθηματικό ερώτημα είναι: Ποιος είναι ο μικρότερος αριθμός προσυμπληρωμένων
κελιών στο υπό συμπλήρωση λατινικό τετράγωνο της σπαζοκεφαλιάς Sudokou έτσι ώστε να υπάρχει ένας
μοναδικός τρόπος να συμπληρωθεί το τετράγωνο; Το 2017 αποδείχτηκε ότι ο αριθμός αυτός είναι ο 17,
δηλαδή αποδείχτηκε ότι δεν υπάρχουν Sudokou με μοναδική λύση αν τα προσυμπληρωμένα κελιά είναι
λιγότερα από 17, ενώ υπάρχουν Sudokou με 17 προσυμπληρωμένα κελιά που έχουν μοναδική λύση.

Στη σπαζοκεφαλιά Kenken, το τετράγωνο είναι συνήθως:

a. τάξης 4 (4×4), οπότε οι τομείς είναι τετραγωνίδια τάξης 2 (2×2). Τα κελιά του τετραγώνου πρέπει
να γεμίσουν με τα ψηφία 1, 2, 3, 4 έτσι ώστε κάθε ψηφίο να εμφανίζεται ακριβώς μία φορά σε κάθε
γραμμή και κάθε στήλη. Επιπλέον, υπάρχουν περιοχές μέσα στις οποίες τα ψηφία πρέπει να
συμφωνούν με μια αριθμητική συνθήκη που δίνεται για την περιοχή.
b. τάξης 6 (6×6), οπότε οι τομείς είναι τετραγωνίδια τάξης 3 (3×3). Τα κελιά του τετραγώνου πρέπει
να γεμίσουν με τα ψηφία 1, 2, 3, 4, 5, 6 έτσι ώστε κάθε ψηφίο να εμφανίζεται ακριβώς μία φορά σε
κάθε γραμμή και κάθε στήλη. Επιπλέον, υπάρχουν περιοχές μέσα στις οποίες τα ψηφία πρέπει να
συμφωνούν με μια αριθμητική συνθήκη που δίνεται για την περιοχή.
c. τάξης 9 (9×9) και χωρισμένο σε εννέα τομείς που είναι τετραγωνίδια τάξης 3 (3×3). Τα κελιά του
τετραγώνου πρέπει να γεμίσουν με τα ψηφία 1, 2, 3, . . . , 9 έτσι ώστε κάθε ψηφίο να εμφανίζεται
ακριβώς μία φορά σε κάθε γραμμή και κάθε στήλη. Επιπλέον, υπάρχουν περιοχές μέσα στις οποίες
τα ψηφία πρέπει να συμφωνούν με μια αριθμητική συνθήκη που δίνεται για την περιοχή.

Ψυχαγωγικά Μαθηματικά 175


I. Να συμπληρώσεις τα κελιά του παρακάτω λατινικού τετραγώνου με ψηφία 1, 2, 3, 4, 5, 6, 7, 8, 9 έτσι
ώστε σε κάθε τομέα να περιέχοντα όλα τα ψηφία 1, 2, 3, 4, 5, 6, 7, 8, 9 (Sudokou εύκολο).

2 1 5
7 3 8 6 9
6 1 7 8

6 4 7 3
8 5 7 3 6 9
3 9 8 2

9 8 3 7 5
5 3 1 8

5 3 6

II. Να συμπληρώσεις τα κελιά του παρακάτω λατινικού τετραγώνου με ψηφία 1, 2, 3, 4, 5, 6, 7, 8, 9 έτσι


ώστε σε κάθε τομέα να περιέχοντα όλα τα ψηφία 1, 2, 3, 4, 5, 6, 7, 8, 9 (Sudokou μέτριας δυσκολίας).

1
3 2 9 7
7 8 6 2 5
8 1 7 9

9 6 2
2 4 5

6
4 7 5
9 2 1 8 5

III. Να συμπληρώσεις τα κελιά του παρακάτω λατινικού τετραγώνου με ψηφία 1, 2, 3, 4, 5, 6, 7, 8, 9 έτσι


ώστε σε κάθε τομέα να περιέχοντα όλα τα ψηφία 1, 2, 3, 4, 5, 6, 7, 8, 9 (Sudokou δύσκολο).

7 2 9 6 5
8 7

6 5
4 6 2 5 3

8 9
4 6
2 5 9 1

9 3
9 2 5 4

176 Χατζηκυριάκου Κώστας


IV. Να συμπληρώσεις τα κελιά του παρακάτω λατινικού τετραγώνου με ψηφία 1, 2, 3, 4, 5, 6, 7, 8, 9 έτσι
ώστε σε κάθε τομέα να περιέχοντα όλα τα ψηφία 1, 2, 3, 4, 5, 6, 7, 8, 9 και σε κάθε περιοχή τα ψηφία να
ακολουθούν την αριθμητική σχέση που δίνεται (Kenken εύκολο).
3+ 13+
3

3‒

7+ 6+ 1‒

V. Να συμπληρώσεις τα κελιά του παρακάτω λατινικού τετραγώνου με ψηφία 1, 2, 3, 4, 5, 6, 7, 8, 9 έτσι


ώστε σε κάθε τομέα να περιέχοντα όλα τα ψηφία 1, 2, 3, 4, 5, 6, 7, 8, 9 και σε κάθε περιοχή τα ψηφία να
ακολουθούν την αριθμητική σχέση που δίνεται (Kenken μέτριας δυσκολίας).
24× 2÷

3‒ 9+

5+ 7+

VI. Να συμπληρώσεις τα κελιά του παρακάτω λατινικού τετραγώνου με ψηφία 1, 2, 3, 4, 5, 6, 7, 8, 9 έτσι


ώστε σε κάθε τομέα να περιέχοντα όλα τα ψηφία 1, 2, 3, 4, 5, 6, 7, 8, 9 και σε κάθε περιοχή τα ψηφία να
ακολουθούν την αριθμητική σχέση που δίνεται (Kenken δύσκολο).
1– 5+

2÷ 1–

3– 8× 1–

Ψυχαγωγικά Μαθηματικά 177


Απαντήσεις στα ερωτήματα του 8ου Κεφαλαίου

8.1 Λατινικά τετράγωνα


I. Λατινικό τετράγωνο τάξης 2:

1 2

2 1

Λατινικό τετράγωνο τάξης 3:

1 2 3

2 3 1

3 1 2

Λατινικό τετράγωνο τάξης 4:

1 2 3 4

2 3 4 1

3 4 1 2

4 1 2 3

Ακολουθώντας το ίδιο μοτίβο, να ένα λατινικό τετράγωνο τάξης ν.

1 2 … ν‒1 ν

2 3 … ν 1

3 4 … 1 2

⋮ ⋮ ⋱ ⋮ ⋮

ν 1 … ν‒2 ν‒1

Ψυχαγωγικά Μαθηματικά 179


II. Να ένας τρόπος να το κάνουμε:

Π1 Β3 Ν4 Γ2

Ν2 Γ4 Π3 Β1

Γ3 Ν1 Β2 Π4

Β4 Π2 Γ1 Ν3

Παρατηρούμε ότι το τετράγωνο που σχηματίσαμε μπορεί να προκύψει με την τοποθέτηση του
πρώτου από τα παρακάτω τετράγωνα πάνω στο τετράγωνο που βρίσκεται στα δεξιά του.

Π Β Ν Γ 1 3 4 2

Ν Γ Π Β 2 4 3 1

Γ Ν Β Π 3 1 2 4

Β Π Γ Ν 4 2 1 3

III. Είναι φανερό πως δεν μπορεί να υπάρξει ελληνολατινικό ζευγάρι τετραγώνων τάξης 2, αφού τα μόνα δύο
λατινικά τετράγωνα τάξης 2 είναι τα:

1 2 2 1

2 1 1 2

Να ένα ελληνολατινικό ζευγάρι τετραγώνων τάξης 3.

1 2 3 1 2 3

2 3 1 3 1 2

3 1 2 2 3 1

Πράγματι, αν τοποθετήσουμε το δεύτερο τετράγωνο πάνω στο άλλο, καθένα από τα στοιχεία του
καρτεσιανού γινομένου {1,2, 3}2 εμφανίζεται ακριβώς μία φορά.

(1,1) (2,2) (3,3)

(2,3) (3,1) (1,2)

(3,2) (1,3) (2,1)

Να μια ακολουθία αμοιβαία ορθογώνιων λατινικών τετραγώνων τάξης 4, όπως διαπιστώνουμε


εύκολα τοποθετώντας τα τετράγωνα το ένα πάνω στο άλλο.

180 Χατζηκυριάκου Κώστας


1 2 3 4 1 2 3 4 1 2 3 4
2 1 4 3 3 4 1 2 4 3 2 1
3 4 1 2 4 3 2 1 2 1 4 3
4 3 2 1 2 1 4 3 3 4 1 2

11 22 33 44 11 22 33 44 11 22 33 44
23 14 41 32 24 13 42 31 34 43 12 21
34 43 12 21 32 41 14 23 42 31 24 13
42 31 24 13 43 34 21 12 23 14 41 32

Δεν είναι δύσκολο να δούμε ότι δεν μπορεί να υπάρξει ακολουθία αμοιβαία ορθογώνιων λατινικών
τετραγώνων με 4 όρους, καθώς, χωρίς περιορισμό της γενικότητας, μπορούμε να υποθέσουμε ότι
οποιαδήποτε τέτοια ακολουθία έχει στην πρώτη γραμμή το αρχικό τμήμα των φυσικών 1, 2, 3, 4, οπότε
στη θέση (2,1) δεν μπορεί να έχει το 1.

Παρατήρηση: Γενικεύεται άραγε το αποτέλεσμα αυτό; Δηλαδή, ισχύει ότι για κάθε τάξη ν, η
μεγαλύτερη ακολουθία αμοιβαία ορθογώνιων λατινικών τετραγώνων έχει μήκος ν‒1; Η απάντηση είναι
ναι. Αν σκεφτούμε ανάλογα, μπορούμε να υποθέσουμε, χωρίς περιορισμό της γενικότητας, ότι η οποιαδήποτε
τέτοια ακολουθία μήκους λ έχει στην πρώτη γραμμή το αρχικό τμήμα της φυσικής ακολουθίας 1, 2, 3, …, λ,
οπότε στη θέση (2,1) δεν μπορεί να έχει το 1 και άρα το λ δεν μπορεί να ξεπερνά το ν‒1 (Η υπόθεση που
κάναμε όντως δεν περιορίζει τη γενικότητα, καθώς είναι φανερό πως οποιαδήποτε μετάθεση των αριθμών1,
2, …, ν σε ένα λατινικό πίνακα οδηγεί σε λατινικό πίνακα που παραμένει ορθογώνιος με όλους τους
υπόλοιπους πίνακες της ακολουθίας). Τώρα, στη θέση (2,1) δεν μπορεί να τοποθετηθεί το 1, αλλά κάποιoς
από τους αριθμούς 2, …, λ.

Να μια ακολουθία τεσσάρων αμοιβαία ορθογώνιων λατινικών τετραγώνων τάξης 5.

4 1 3 5 2 5 2 4 1 3

5 2 4 1 3 4 1 3 5 2

1 2 5 2 4 3 5 2 4 1

2 4 1 3 5 2 4 1 3 5

3 5 2 4 1 1 3 5 2 4

3 5 2 4 1 1 3 5 2 4

1 3 5 2 4 3 5 2 4 1

4 1 3 5 2 5 2 4 1 3

2 4 1 3 5 2 4 1 3 5

5 2 4 1 3 4 1 3 5 2

Ψυχαγωγικά Μαθηματικά 181


Αποδεικνύεται ότι για κάθε πρώτο φυσικό αριθμό p, υπάρχουν ακριβώς p‒1 αμοιβαία ορθογώνια λατινικά
τετράγωνα. Μάλιστα, ένας τρόπος που μπορεί να βρει κανείς μια τέτοια ακολουθία είναι ο εξής: Αν α ∈
{1,2,3, …., p‒1}, τότε τα p‒1 αμοιβαία λατινικά τετράγωνα είναι τα {x+y}, {2x+y}, {3x+y}, {4x+y}, όπου
τα x, y ∈ {0, 1, 2, 3, …, p} που αθροίζονται όπως τα υπόλοιπα διαιρέσεων δια 5, π.χ. για p = 5, 2 + 2 = 4, 2 +
3 = 0, 2+ 4 = 1 κ.λπ.

Δεν υπάρχει ζευγάρι ελληνολατινικών τετραγώνων τάξης 6. Η σπαζοκεφαλιά αυτή απασχόλησε τον
Euler, αλλά δεν μπόρεσε να τη λύσει. Γράφει το 1779 σε ένα άρθρο του που τιτλοφορείται «Σχετικά με ένα
νέο είδος μαγικού τετραγώνου»:

Ένα πολύ περίεργο ερώτημα που γύμνασε την εξυπνάδα πολλών ανθρώπων με απασχόλησε στη μελέτη
που ακολουθεί, η οποία φαίνεται να ανοίγει ένα νέο πεδίο ανάλυσης, ιδιαίτερα στη μελέτη της συνδυαστικής.
Το ερώτημα αφορά την τοποθέτηση σε τετραγωνική διάταξη 36 αξιωματικών που προέρχονται από έξι
διαφορετικούς βαθμούς και ταυτόχρονα από έξι διαφορετικά συντάγματα με τρόπο ώστε σε κάθε γραμμή
(τόσο οριζόντια όσο και κατακόρυφη) να βρίσκονται έξι αξιωματικοί διαφορετικών βαθμών και
διαφορετικών συνταγμάτων.

Ο Euler διατύπωσε μάλιστα την εικασία ότι δεν υπάρχουν ελληνολατινικά τετράγωνα τάξης ν = 4λ + 2,
λ = 1, 2, … . Η εικασία του αποδείχθηκε λανθασμένη γύρω στα 1960. Υπάρχουν ελληνολατινικά τετράγωνα
για κάθε μη μηδενικό φυσικό αριθμό ν ≠ 2, 6.

IV. Να μια μέθοδος με την οποία μπορούμε να κατασκευάσουμε μαγικά τετράγωνα από ζευγάρια διαγώνιων
ελληνολατινικών τετραγώνων. Διαγώνια είναι εκείνα τα λατινικά τετράγωνα που και στις διαγωνίους
τους δεν υπάρχουν επαναλήψεις.
Έστω ότι Λ1, Λ2 είναι ζευγάρι διαγώνιων ελληνολατινικών τετραγώνων με αριθμούς από το σύνολο
{1, 2, …, n}. Τοποθέτησε το ένα πάνω στο άλλο. Εάν στο κελί (i, j) βρίσκεται το διατεταγμένο ζευγάρι
(x, y) του Λ1 × Λ2, τότε γράψε τον αριθμό nx + y στο αντίστοιχο (i, j) κελί του τετραγώνου. Το τετράγωνο
που προκύπτει είναι μαγικό.

Είναι φανερό ότι, αφού έχουμε κάθε ζευγάρι (x, y) του Λ1 × Λ2, στο τετράγωνο που σχηματίζουμε θα
υπάρχουν όλοι οι αριθμοί από n+1 ως και n2 + n, συνολικά n2 αριθμοί. Το τετράγωνο επίσης είναι μαγικό,
αφού το άθροισμα των αριθμών κατά γραμμή, στήλη και διαγώνιο είναι ίδιο. Γιατί; Επειδή είτε
κοιτάξουμε γραμμή, είτε στήλη, είτε διαγώνιο, κάθε x ∈ {1, 2, …, n} εμφανίζεται σε αυτήν ακριβώς μία
φορά και επίσης κάθε y ∈ {1, 2, …, n} εμφανίζεται σε αυτήν ακριβώς μία φορά. Άρα, το άθροισμα
οποιασδήποτε γραμμής, στήλης ή διαγωνίου είναι:

𝑛∙(𝑛+1) 𝑛∙(𝑛+1) 𝑛∙(𝑛+1)2


n ∙ (1+2+ . . . + n) + (1+2+ . . . + n) = n ∙ + = .
2 2 2
Παράδειγμα

1 3 5 2 4 1 2 3 4 5

2 4 1 3 5 3 4 5 1 2

3 5 2 4 1 5 1 2 3 4

4 1 3 5 2 2 3 4 5 1

5 2 4 1 3 4 5 1 2 3

182 Χατζηκυριάκου Κώστας


Βήμα 1ο
11 32 53 24 45

23 44 15 31 52

35 51 22 43 14

42 13 34 55 21

54 25 41 12 33

Βήμα 2ο
6 17 28 14 25

13 24 10 16 27

20 26 12 23 9

22 8 19 30 11

29 15 21 7 18

Βήμα 3ο
1 12 23 9 20

8 19 5 11 22

15 21 7 18 4

17 3 14 25 6

24 10 16 2 13

V. Το αριστερό μπορεί να συμπληρωθεί στο λατινικό τετράγωνο που ακολουθεί. Το δεξί, προφανώς δεν
μπορεί να συμπληρωθεί σε λατινικό τετράγωνο.

1 2 3 4

2 3 4 1

3 4 1 2

4 1 2 3

Ψυχαγωγικά Μαθηματικά 183


Αποδεικνύεται ότι αν μας δοθούν συμπληρωμένες οι k πρώτες γραμμές ενός λατινικού τετραγώνου
Μ, τότε μπορούμε να γεμίσουμε τα άδεια κελιά έτσι ώστε όντως να σχηματιστεί ένα λατινικό τετράγωνο.
Αξίζει να δούμε την απόδειξη αυτή, μιας και σε αυτήν χρησιμοποιείται το γαμήλιο θεώρημα του Hall.

Σχηματίζουμε ένα διμερές γράφημα Γ = (Κ= Ν∪Λ, Α) ως εξής:

Ν = {1, 2, …, ν},

Λ = {Λ1, Λ2, …, Λν}, όπου Λj = το σύνολο των στοιχείων του Ν που δεν ανήκουν στη δοσμένη στήλη i,

A = {(i, Λj): i ∈ N και Λj ∈ Λ και i ∈ Λj}.

Λόγω του τρόπου με τον οποίο το σχηματίσαμε, ο βαθμός κάθε κορυφής Λ j ισούται με το πλήθος των
στοιχείων του Ν, που δεν είναι στοιχείο του συνόλου που αντιπροσωπεύει, άρα είναι ν–k. Αντίστοιχα, ο
βαθμός κάθε κορυφής i ισούται με το πλήθος των στηλών στις οποίες δεν εμφανίζεται ακόμη, άρα είναι
πάλι ν–k. Το διμερές γράφημά μας είναι κανονικό και άρα πληροί την υπόθεση του θεωρήματος Hall,
δηλαδή υπάρχει 1-1 αντιστοιχία ανάμεσα στο Ν και στο Λ. Πάρε μια τέτοια αντιστοιχία και συμπλήρωσε
έτσι την επόμενη γραμμή. Επανάλαβε τη διαδικασία για την επόμενη ώσπου να συμπληρωθεί όλο το
μαγικό τετράγωνο.

Να ένα παράδειγμα: Συμπλήρωσε τον παρακάτω λατινικό πίνακα.

1 2 3 4 5

2 4 1 5 3

3 5 2 1 4

Η μέθοδος μας λέει να σχηματίσουμε το γράφημα ({1, 2, 3, 4, 5}∪{Λ1 = {4,5}, Λ2 = {1,3}, Λ3 = {5, 4}, Λ4 =
{2,3}, Λ5 = {2,1}}, {1Λ2, 1Λ5, 2Λ5, 2Λ4, 3Λ2, 3Λ4, 4Λ1, 4Λ3, 5Λ1, 5Λ3}). Οι 1-1 αντιστοιχίες που μπορούμε
να χρησιμοποιήσουμε τότε είναι οι {(1,5), (2,4), (3,2), (4,1), (5,3)} και {(1,2), (2,5), (3,4), (4,3), (5,1)}, που
μας δίνουν αντίστοιχα τα λατινικά τετράγωνα:

1 2 3 4 5 1 2 3 4 5

2 4 1 5 3 2 4 1 5 3

3 5 2 1 4 ή 3 5 2 1 4

4 3 5 2 1 5 1 4 3 2

5 1 4 3 2 4 3 5 2 1

184 Χατζηκυριάκου Κώστας


8.2 Οι σπαζοκεφαλιές Sudokou και Kenken
I. H λύση:

2 1 8 3 6 9 5 4 7
7 5 3 2 4 8 6 9 1
4 9 6 1 7 5 2 3 8

9 6 2 4 5 1 8 7 3
1 8 5 7 3 6 9 2 4

3 7 4 9 8 2 1 6 5
6 4 9 8 1 3 7 5 2
5 3 1 6 2 7 4 8 9

8 2 7 5 9 4 3 1 6

II. Η λύση:

5 6 8 2 7 4 9 1 3
3 4 2 1 5 9 6 8 7

1 9 7 8 3 6 2 5 4
6 8 5 3 2 1 4 7 9
7 3 4 9 8 5 1 6 2

2 1 9 6 4 7 5 3 8
8 5 1 4 9 3 7 2 6

4 7 3 5 6 2 8 9 1
9 2 6 7 1 8 3 4 5

III. Η λύση:

1 7 8 2 9 6 3 4 5
9 4 3 5 8 1 7 2 6
6 5 2 7 4 3 8 9 1
4 6 7 1 2 9 5 3 8

2 3 1 8 6 5 4 7 9
8 9 5 3 7 4 1 6 2
7 2 4 6 5 8 9 1 3

5 1 9 4 3 2 6 8 7
3 8 6 9 1 7 2 5 4

Ψυχαγωγικά Μαθηματικά 185


IV. H λύση:
3+ 13+
1 3 2 4

3‒
2 1 4 3

7+ 6+ 1‒
3 4 1 2

4 2 3 1

Ας δούμε έναν τρόπο με τον οποίο μπορούμε να καταλήξουμε σε αυτήν τη λύση (IV). Για να έχουμε
άθροισμα 13 με 4 προσθετέους τους αριθμούς 1, 2, 3, 4 και με δεδομένους τους περιορισμούς στις γραμμές
και τις στήλες του Kenken, πρέπει να τοποθετήσουμε στην τετράγωνη περιοχή τους αριθμούς 4, 4, 3, 2, με τα
τεσσάρια σε διαφορετικές γραμμές. Αυτό μας αναγκάζει να γράψουμε πάνω το 1 και κάτω το 3 στην περιοχή
του 3‒. Οπότε, είμαστε αναγκασμένοι στην περιοχή του 3+ να γράψουμε πάνω το 1 και κάτω το 2 και στην
περιοχή του 7+ να γράψουμε πάνω το 3 και κάτω το 4. Επομένως, ο αριθμός που βρίσκεται στην τέταρτη
γραμμή και τη δεύτερη στήλη είναι αναγκαστικά ο 2. Αλλά τότε, στην περιοχή του 6+ δεν μπορούμε παρά να
βάλουμε τους αριθμούς 1 και 3 (με τον 1 πάνω και τον 3 κάτω) και στην περιοχή του 1‒ τους αριθμούς 2 και 1
(με τον 2 πάνω και τον 1 κάτω), οπότε οι αριθμοί στην περιοχή του 13+ δεν μπορεί παρά να είναι
τοποθετημένοι με τον τρόπο που βλέπεις στη λύση.

V. Η λύση:
24× 2÷
3 4 2 1

3‒ 9+
2 1 4 3

5+ 7+
4 3 1 2

1 2 3 4

Η τοποθέτηση των 1, 2 στην πρώτη γραμμή είναι αναγκαστική, αφού δεν μπορεί παρά να είναι 9 = 2 + 3 + 4.
Άρα, το 2 στη δεύτερη γραμμή της πρώτης στήλης είναι η μόνη επιλογή. Οπότε, αφού 3 = 4 –1, είναι φανερό
ότι 9 = 3 + 2 + 4 (από πάνω προς τα κάτω). Αλλά τότε, 7 = 3 + 1 + 3, όπως φαίνεται στη λύση, το 1 είναι στα
αριστερά του 4 στην περιοχή του 3‒ και τα υπόλοιπα κελιά συμπληρώνονται αναγκαστικά όπως παραπάνω.

186 Χατζηκυριάκου Κώστας


VI. Η λύση:
1– 5+
2 3 4 1

2÷ 1–
1 2 3 4

3– 8× 1–
4 1 2 3

3 4 1 2

Ψυχαγωγικά Μαθηματικά 187


Βιβλιογραφία/Αναφορές

Hunter, J. A. H., & Madachy, J. S. (1975). Mathematical Diversions. Dover Publications Inc.
Kraitchik, M. (1953). Mathematical Recreations. Dover Publications Inc.
Stein, S. K. (1999). Mathematics, The Man-Made Universe. Dover Publications.

188 Χατζηκυριάκου Κώστας


Κεφάλαιο 9
Συμμετρίες, ομάδες, συγγένειες

Σύνοψη
Η δομή των συμμετριών του τετραγώνου και του ισόπλευρου τριγώνου. Η έννοια της μαθηματικής δομής της
ομάδας. Εθνομαθηματικά: Η συγγένεια και η μαθηματική δομή της ομάδας.

Προαπαιτούμενη γνώση
Το τετράγωνο και το ισόπλευρο τρίγωνο. Η έννοια της συνάρτησης.

Ψυχαγωγικά Μαθηματικά 189


9.1 Οι συμμετρίες του τετραγώνου
I. Να περιστρέψεις, στη φορά των δεικτών του ωρολογίου, κατά 90, 180, 270, 360 μοίρες το παρακάτω
τετράγωνο και για κάθε περιστροφή να καταγράψεις τις θέσεις των αρχικών κορυφών. Να καταγράψεις
τους κατοπτρισμούς του τετραγώνου ως προς τις δύο διαγωνίους του δ13, δ24 και ως προς τις δύο
διαμέσους του, την κατακόρυφη μ1 και την οριζόντια μ2, και για κάθε κατοπτρισμό να καταγράψεις τις
θέσεις των αρχικών κορυφών.

Σχήμα 9.1

190 Χατζηκυριάκου Κώστας


II. Είναι φανερό ότι κάθε περιστροφή σαν τις παραπάνω και κάθε κατοπτρισμός σαν τους παραπάνω ορίζουν
μια μετάθεση των αριθμών 1, 2, 3, 4, δηλαδή είναι μια αμφιμονότιμη συνάρτηση του συνόλου {1,2, 3, 4}
πάνω στον εαυτό του. Ποια (γεωμετρική) ιδιότητα έχουν οι μεταθέσεις αυτές τις οποίες στη γεωμετρία τις
αποκαλούμε συμμετρίες του τετραγώνου;

III. Ας ονομάσουμε ρ τη μετάθεση των κορυφών του τετραγώνου που αντιστοιχεί στην περιστροφή τους, στη
φορά των δεικτών του ωρολογίου, κατά 90 μοίρες. Τότε, ρ[1234] = 4123 και ρ2[1234] = ρρ[1234] =
ρ[4123] = 3412 = η μετάθεση που αντιστοιχεί σε περιστροφή των κορυφών, στη φορά των δεικτών του
ωρολογίου, κατά 180 μοίρες.
Ας ονομάσουμε μ τη μετάθεση που αντιστοιχεί στον κατοπτρισμό ως προς την κατακόρυφη διάμεσο,
τότε μ[1234] = 2143. Προφανώς, μ2[1234] = μ[2143] = 1234 = e[1234] = η ταυτοτική μετάθεση.

Παρατήρησε ότι με τη βοήθεια των ρ και μ μπορούμε να σχηματίσουμε τις εξής οκτώ μεταθέσεις ρ,
ρ2, ρ3, e (= ρ4), μ, ρμ, ρ2μ, ρ3μ. (Παραλείπουμε να γράφουμε το σύμβολο ∙ της σύνθεσης των μεταθέσεων).

Δείξε ότι αυτές είναι οι οκτώ συμμετρίες του τετραγώνου που είδαμε στη δραστηριότητα Ι και
συμπλήρωσε τον παρακάτω πίνακα:

Υπόδειξη: ρμρμ[1234] = ρμρ[2143] = ρμ[3214] = ρ[2341] = 1234 = e[1234].

Αφού ρμρμ = e, άρα μρμ = ρ3 και ρμ = μρ3.

ρ ρ2 ρ3 μ ρμ ρ2μ ρ3μ e
ρ ρμ
ρ2 ρ3 e ρ ρ2μ ρ3μ μ ρμ ρ2
ρ3 ρ3μ
μ ρμ e
ρμ ρ e
ρ2μ ρ2
ρ3μ ρ3
e μ

Ψυχαγωγικά Μαθηματικά 191


9.2 Οι συμμετρίες του ισόπλευρου τριγώνου
I. Να περιστρέψεις, στη φορά των δεικτών του ωρολογίου, κατά 120, 240, 360 μοίρες το παρακάτω
ισόπλευρο τρίγωνο και για κάθε περιστροφή να καταγράψεις τις θέσεις των αρχικών κορυφών. Να
καταγράψεις τους κατοπτρισμούς του τετραγώνου ως προς τις τρεις διάμεσους του μ 1, μ2 και μ3 και για
κάθε

Σχήμα 9.2

192 Χατζηκυριάκου Κώστας


II. Είναι φανερό ότι κάθε περιστροφή σαν τις παραπάνω και κάθε κατοπτρισμός σαν τους παραπάνω ορίζουν
μια μετάθεση των αριθμών 1, 2, 3, δηλαδή είναι μια αμφιμονότιμη συνάρτηση του συνόλου {1,2,3} πάνω
στον εαυτό του. Ποια (γεωμετρική) ιδιότητα έχουν οι μεταθέσεις αυτές τις οποίες στη γεωμετρία τις
αποκαλούμε συμμετρίες του ισόπλευρου τριγώνου;

III. Ας ονομάσουμε σ τη μετάθεση των κορυφών του τριγώνου που αντιστοιχεί στην περιστροφή τους, στη
φορά των δεικτών του ωρολογίου, κατά 120 μοίρες. Τότε, σ[123] = 312 και σ2[123] = σσ[123] = σ[312] =
231 = η μετάθεση που αντιστοιχεί σε περιστροφή των κορυφών, στη φορά των δεικτών του ωρολογίου,
κατά 240 μοίρες.
Ας ονομάσουμε μ τη μετάθεση που αντιστοιχεί στον κατοπτρισμό ως προς τη διάμεσο (μ 2), τότε
μ[123] = 321. Προφανώς, μ2[123] = μ[321] = 123 = e[123] = η ταυτοτική μετάθεση.

Παρατήρησε ότι με τη βοήθεια των ρ και μ μπορούμε να σχηματίσουμε τις εξής έξι μεταθέσεις σ, σ 2,
e (= σ3), μ, σμ, σ2μ.

Δείξε ότι αυτές είναι οι έξι συμμετρίες του ισόπλευρου τριγώνου που είδαμε στη δραστηριότητα Ι και
συμπλήρωσε τον παρακάτω πίνακα:

σ σ2 μ σμ σ 2μ e
σ σμ
σ2 e σ σ2μ μ σμ σ2
μ e
σμ σ e
σ 2μ σ2
e μ

Ψυχαγωγικά Μαθηματικά 193


9.3 Συγγένειες
I. Οι Walpiri ζουν στην Αυστραλία και έχoυν ένα σύστημα συγγένειας που αποτελείται από οκτώ
υποτομείς. Κάθε μέλος της φυλής ανήκει ακριβώς σε έναν υποτομέα, μπορεί να παντρευτεί μόνο με τα
μέλη ορισμένων άλλων υποτομέων και τα παιδιά του ανήκουν σε έναν τρίτο υποτομέα που εξαρτάται από
τον υποτομέα στον οποίον ανήκει η μητέρα. Ας απαριθμήσουμε τους οκτώ υποτομείς 1, 2, ..., 8. Οι
εθνολόγοι που μελέτησαν τη φυλή αυτή πληροφορήθηκαν ότι οι γάμοι επιτρέπονται μεταξύ μελών του
υποτομέα 1 και του υποτομέα 5, του 2 και του 6, του 3 και του 7, του 4 και του 8. Σε ποιον υποτομέα
ανήκουν τα παιδιά των γάμων αυτών φαίνεται από το επόμενο σχήμα. Τα βέλη πηγαίνουν από μητέρα σε
παιδί.

Σχήμα 9.3

194 Χατζηκυριάκου Κώστας


a. Σε ποιον υποτομέα ανήκει η μητέρα ενός μέλους του υποτομέα 1, η μητέρα της μητέρας της, η μητέρα
της μητέρας της μητέρας της, η μητέρα της μητέρας της μητέρας της μητέρας της; Αντίστοιχα για ένα
μέλος οποιουδήποτε άλλου υποτομέα. Σε ποιον υποτομέα ανήκει ο πατέρας ενός μέλους του υποτομέα 1;
Ο πατέρας του πατέρα του; Αντίστοιχα για ένα μέλος οποιουδήποτε άλλου υποτομέα.

b. Μπορείς να δείξεις ότι η δομή συγγένειας των Walpiri είναι η διεδρική ομάδα D4;

Υπόδειξη: Ας είναι m η αντιστοιχία που στέλνει ένα μέλος κάποιου υποτομέα στη μητέρα του και f η
αντιστοιχία που στέλνει ένα μέλος κάποιου υποτομέα στον πατέρα του. Τι μπορείς να πεις για τη σύνθεση
των αντιστοιχιών αυτών;

Ψυχαγωγικά Μαθηματικά 195


II. To σύστημα συγγένειας των κατοίκων της νήσου Μαλεκούλα εξηγήθηκε κάποτε σε έναν ανθρωπολόγο
από έναν ιθαγενή πληροφορητή που σχεδίασε σχήματα στην άμμο. Ο ιθαγενής σχεδίασε τρεις
διανυσματικές ακτίνες ενός κύκλου (που βλέπουν στο κέντρο του) για να αναπαραστήσει τρεις
παντρεμένους άνδρες Ε, Γ, Δ από διαφορετικές φυλές (bwelem). Καθένας είναι παντρεμένος με γυναίκα
από διαφορετική φυλή και έχει δύο παιδιά, ένα αγόρι και ένα κορίτσι που ανήκουν στην ίδια φυλή με
αυτόν, αλλά σε διαφορετική «γραμμή» από τη δική του, γι’ αυτό και τα δύο ευθύγραμμα τμήματά τους
(μικρό για το κορίτσι, μεγάλο για το αγόρι) τοποθετούνται από την πλευρά της ακτίνας που δεν βρίσκεται
ο πατέρας. Από πού προέρχονται οι γυναίκες των ανδρών δηλώνεται με τα διακεκομμένα τόξα που
βλέπεις στη δεξιά εικόνα, στην οποία φαίνεται ολοκληρωμένο το σύστημα συγγένειας, καθώς, όπως ο
πληροφορητής τόνισε εμφατικά, οι γάμοι γίνονται και προς τις δύο κατευθύνσεις (κατά τη φορά των
δεικτών του ωρολογίου, αλλά και αντίστροφα).
Μπορείς να δείξεις ότι η δομή συγγένειας των κατοίκων της νήσου Μαλεκούλα είναι η διεδρική ομάδα D3;
Υπόδειξη: Οι τομείς είναι οι άλφα, βήτα, γάμα, δέλτα, έψιλον και ζήτα. Ας είναι m η αντιστοιχία που
στέλνει ένα μέλος κάποιου τομέα στη μητέρα του και f η αντιστοιχία που στέλνει ένα μέλος κάποιου τομέα
στον πατέρα του. Τι μπορείς να πεις για τη σύνθεση των αντιστοιχιών αυτών;

Σχήμα 9.4

196 Χατζηκυριάκου Κώστας


Απαντήσεις στα ερωτήματα του 9ου Κεφαλαίου

9.1 Οι συμμετρίες του τετραγώνου


I. Στους παρακάτω πίνακες φαίνονται τα αποτελέσματα των στροφών και των κατοπτρισμών:

Αρχική Στροφή Στροφή Στροφή Στροφή


θέση 90º 180º 270º 360º
1 2 4 1 3 4 2 3 1 2
4 3 3 2 2 1 1 4 4 3

Αρχική Κατοπτρισμός Κατοπτρισμός Κατοπτρισμός Κατοπτρισμός


θέση ως προς δ13 ως προς δ24 ως προς μ1 ως προς μ2
1 2 1 4 3 2 2 1 4 3
4 3 2 3 4 1 3 4 1 2

II. Είναι φανερό ότι οι μεταθέσεις αυτές διατηρούν τις αποστάσεις μεταξύ των σημείων του τετραγώνου.
Αποδεικνύεται ότι αυτές είναι όλες οι συμμετρίες του τετραγώνου.

III. Προφανώς οι μεταθέσεις ρ, ρ2, ρ3, e (= ρ4) είναι αντίστοιχα οι στροφές κατά 90º, 180º, 270º, 360º (στη
φορά των δεικτών του ωρολογίου). Αν μ είναι ο κατοπτρισμός ως προς την κατακόρυφη διάμεσο μ 1, η ρμ
είναι ο κατοπτρισμός ως προς τη διαγώνιο δ 24, η ρ2μ είναι ο κατοπτρισμός ως προς την οριζόντια διάμεσο
μ2 και η ρ3μ είναι ο κατοπτρισμός ως προς τη διαγώνιο δ13. Ο συμπληρωμένος πίνακας είναι ο ακόλουθος:

∙ ρ ρ2 ρ3 μ ρμ ρ2μ ρ3μ e
ρ ρ2 ρ3 e ρμ ρ 2μ ρ3μ μ ρ
ρ 2
ρ 3
e ρ ρμ
2
ρμ
3
μ ρμ ρ2
ρ3 e ρ ρ2 ρ3μ μ ρμ ρ2μ ρ3
μ ρ3μ ρ2μ ρμ e ρ3 ρ2 ρ μ
ρμ μ ρμ 3
ρμ
2
ρ e ρ 3
ρ 2
ρμ
ρ2μ ρμ μ ρ3μ ρ2 ρ e ρ3 ρ2μ
ρ3μ ρ2μ ρμ μ ρ3 ρ2 ρ e ρ3μ
e ρ ρ2 ρ3 μ ρμ ρ2μ ρ3μ e

Αναστοχαζόμενοι τη λύση μας, πάντα κατά την προτροπή του Polya, μπορούμε να παρατηρήσουμε
ότι καθεμία από τις οκτώ συμμετρίες εμφανίζεται ακριβώς μία φορά σε κάθε γραμμή και σε κάθε
στήλη, ότι για κάθε συμμετρία υπάρχει συμμετρία που η σύνθεσή τους είναι η ταυτοτική συμμετρία
(μετάθεση, συνάρτηση), ότι η σύνθεση φχψ τριών συμμετριών φ, χ, ψ μπορεί να γίνει είτε ως (φχ)ψ
είτε ως φ(χψ), ενώ ρ2μ = μρ2, ρμ ≠ μρ.

Ψυχαγωγικά Μαθηματικά 197


9.2 Οι συμμετρίες του ισόπλευρου τριγώνου
I. Στους παρακάτω πίνακες φαίνονται τα αποτελέσματα των στροφών και των κατοπτρισμών:
Αρχική Στροφή Στροφή Στροφή
θέση 120º 240º 360º
1 3 2 1
3 2 2 1 1 3 3 2

Αρχική Κατοπτρισμός Κατοπτρισμός Κατοπτρισμός


θέση ως προς μ1 ως προς μ2 ως προς μ3
1 1 3 2
3 2 2 3 1 2 3 1

II. Είναι φανερό ότι οι μεταθέσεις αυτές διατηρούν τις αποστάσεις μεταξύ των σημείων του τριγώνου.
Γεωμετρικά είναι ισομετρίες. Αποδεικνύεται ότι αυτές είναι όλες οι συμμετρίες του ισόπλευρου
τριγώνου.

III. Προφανώς, οι μεταθέσεις σ, σ2, e (=σ3) είναι αντίστοιχα οι στροφές κατά 120º, 240º, 360º (στη φορά των
δεικτών του ωρολογίου). Αν μ είναι ο κατοπτρισμός ως προς τη διάμεσο μ 2, τότε σμ είναι ο κατοπτρισμός
ως προς τη διάμεσο μ1 και σ2μ είναι ο κατοπτρισμός ως προς τη διάμεσο μ3.

∙ σ σ2 μ σμ σ 2μ e
σ σ2 e σμ σ2μ μ σ
σ 2
e σ σμ
2
μ σμ σ2
μ σ 2μ σμ e σ2 σ μ
σμ μ σμ2
σ e σ 2
σμ
σμ
2
σμ μ σ 2
σ e σ 2μ
e σ σ2 μ σμ σ 2μ e

Αναστοχαζόμενοι τη λύση μας, πάντα κατά την προτροπή του Polya, μπορούμε να παρατηρήσουμε ότι:

1. Καθεμία από τις έξι συμμετρίες εμφανίζεται ακριβώς μία φορά σε κάθε γραμμή και σε κάθε
στήλη.
2. Η σύνθεση οποιωνδήποτε τριών από αυτές, φ, χ, ψ, διέπεται από την ιδιότητα φ∙(χ∙ψ) = (φ∙χ)∙ψ.
3. Υπάρχει ανάμεσά τους μία ταυτοτική συμμετρία, δηλαδή μία συμμετρία της οποίας η σύνθεση
(με οποιαδήποτε σειρά) με οποιαδήποτε συμμετρία σ είναι η συμμετρία σ.
4. Για κάθε συμμετρία, υπάρχει συμμετρία που η σύνθεσή τους (με οποιαδήποτε σειρά) είναι η
ταυτοτική συμμετρία.
5. σ∙μ ≠ μ∙σ.

Στο σημείο αυτό μπορούμε να διακρίνουμε την ανάδυση μιας ενδιαφέρουσας μαθηματικής δομής (Δ,
* ) που συνίσταται σε ένα μη κενό σύνολο Δ και μία πράξη * μεταξύ των στοιχείων του, δηλαδή μια διμελή
συνάρτηση Δ×Δ→Δ, η οποία έχει τις εξής ιδιότητες:

1. Είναι προσεταιριστική, δηλαδή για κάθε δ1, δ2, δ3 ∈ Δ: δ1 * (δ2 * δ3) = (δ1 * δ2) * δ3.
2. Έχει ουδέτερο στοιχείο, δηλαδή υπάρχει ε ∈ Δ, τέτοιο που δ * ε = ε * δ.
3. Για κάθε δ ∈ Δ, υπάρχει μοναδικό αντίστροφο στοιχείο δ-1 ∈ Δ, τέτοιο που δ * δ-1 = δ-1 * δ = ε.

Μια τέτοια (συναρτησιακή) δομή την ονομάζουμε ομάδα. Το πλήθος των στοιχείων της λέγεται τάξη
της ομάδας. H έννοια της ομάδας είναι πολύ βασική, μάλιστα τη γνωρίζουμε σε πολύ νεαρή ηλικία, όπως
φαίνεται στα ερωτήματα του επόμενου φύλλου.

198 Χατζηκυριάκου Κώστας


Θυμήσου ότι έχουμε μελετήσει αρκετά αναλυτικά την έννοια του γραφήματος, που είναι επίσης μια
(σχεσιακή) δομή (Γ, σ), όπου Γ σύνολο, μη κενό, και σ σχέση, σ ⊆ Γ. Στη μελέτη μας εκείνη προέκυψαν δύο
βασικές έννοιες, η έννοια του υπογραφήματος και η έννοια των ισόμορφων γραφημάτων. Οι αντίστοιχες
έννοιες εδώ είναι οι έννοιες της υποομάδας και της ισομορφίας ομάδων.

Η ομάδα 𝒢0 = (G0, #) λέγεται υποομάδα της 𝒢 = (G, *) ανν G0 ⊆ G και η πράξη # της 𝒢0 δεν είναι παρά
ο περιορισμός της πράξης * της 𝒢 στο σύνολο G0.

H ομάδα 𝒢1 = (G0, *) λέγεται ισόμορφη με την ομάδα 𝒢1 = (G1, #) ανν υπάρχει 1-1 συνάρτηση f: G0
→G1, η οποία «διατηρεί τη δομή», δηλ. f (g * g΄) = f(g) # f(g΄). Με άλλα λόγια, οι δύο αυτές ομάδες είναι
διαφορετικές περιγραφές της ίδιας δομής.

Παράδειγμα: Η ομάδα των στροφών ({ρ, ρ2, ρ3, e}, ∙), ρ δεξιοστροφή 90º, είναι υποομάδα της ομάδας
των συμμετριών του τετραγώνου και αντίστοιχα η ομάδα ({σ, σ2, e}, ∙), ρ δεξιοστροφή 120º, είναι υποομάδα
των συμμετριών του ισόπλευρου τριγώνου.

9.3 Συγγένειες
I. Η δομιστική ανάλυση της παραπάνω συγγένειας έδειξε ότι αυτή συγκροτεί ομάδα που είναι η διεδρική
ομάδα τάξης 8. Για να το δούμε αυτό, ας θεωρήσουμε τον τομέα 1 ως τον τυπικό τομέα e, και ας
παρατηρήσουμε όλη τη δομή από τη σκοπιά του. Αφού οι μητέρες όλων των μελών του 1 ανήκουν στον
3, δηλαδή m(1) = 3, ας ονομάσουμε m τον τομέα 3. Επομένως, m2 είναι ο τομέας 2, m3 είναι o τομέας 4, f
είναι ο τομέας 7, και mf, m2 f, m3f οι τομείς 5, 8, 6 αντίστοιχα. Γενικά, βρίσκουμε τον πατέρα ενός μέλος
του τομέα εφαρμόζοντας την f και τη μητέρα εφαρμόζοντας την m. Άρα, m4 = e, f2 = e. Επίσης, mfmf = e.

Καταλήγουμε, έτσι, στον παρακάτω πίνακα και επομένως στη διεδρική ομάδα D4. Οι συμμετρίες του
τετραγώνου … συγγενεύουν με τη δομή συγγένειας στους Waripiri!

∙ m m2 m3 f mf m2f m3f e
m m 2
m 3
e mf mμ
2

3
f m
m2 m3 e m m2f m3μ f mf m2
m3 e m m2 m3f f mf m2f m3
f m3f m2f mf e m3 m2 m f
mf f m3f m2f m e m3 m2 mf
2 3 2 3
mf mf f mf m m e m m2f
m3f m2f mf f m3 m2 m e m3f
e m m2 m3 f mf m2f m3f e

Αν Χ και Υ είναι σύζυγοι, τότε mf(X) = Y και mf(Y) =X.

Οι κύκλοι (1→3→2→4) και (5→8→6→7) λέγονται μητρικά ημιφύλια. Παρατηρούμε ότι τα δύο
σύνολα {1, 2, 3, 4} και {5, 6, 7, 8} είναι ξένα μεταξύ τους και η ένωσή τους είναι το σύνολο των τομέων. Οι
συναναστροφές μεταξύ ατόμων που ανήκουν στο ίδιο μητρικό ημιφύλιο είναι σχετικά ελεύθερες, ενώ αυτές
μεταξύ ατόμων από διαφορετικά ημιφύλια διέπονται από πιο περίπλοκους κανόνες.

Αντίστοιχα, τα δύο σύνολα {(17), (28)}, {(36), (45)} λέγονται πατρικά ημιφύλια. Παρατηρούμε ότι τα
τέσσερα σύνολα {1, 7}, {2, 8}, {3, 6}, {4, 5} είναι ξένα μεταξύ τους και η ένωσή τους είναι το σύνολο των
τομέων. Οι τέσσερις κύκλοι καθορίζουν κληρονομικά δικαιώματα, ευθύνες για την έγγεια ιδιοκτησία και τις
θρησκευτικές τελετές. Τα πατρικά ημιφύλια καθορίζουν δραστηριότητες στο πολιτικό-θρησκευτικό πεδίο.

Ψυχαγωγικά Μαθηματικά 199


Τα σύνολα {1, 6, 2, 5} και {8, 4, 7, 3} λέγονται γενεαλογικά ημιφύλια. Παρατηρούμε ότι τα δύο
σύνολα {1, 6, 2, 5} και {8, 4, 7, 3} είναι ξένα μεταξύ τους και η ένωσή τους είναι το σύνολο των τομέων.
Άτομα που ανήκουν στο ίδιο γενεαλογικό ημιφύλιο θεωρούνται συνομήλικα. Τα ημιφύλια αυτά καθορίζουν
τους νόμιμους γάμους και τη δημιουργία συνεργατικών ομάδων για διάφορους σκοπούς. Ειδικότερα, όπως
φαίνεται στο σχήμα που αναπαριστά τη δομή συγγένειας στους Waripiri, οι γάμοι που προκρίνονται είναι
μεταξύ των μελών των τομέων που είναι σε καθένα από τα τέσσερα διμελή σύνολα { 1, 5}, {2, 6} (⊆{1, 6, 2,
5}), {3, 7}, {4, 8}(⊆{8, 4, 7, 3}). Παρατηρούμε πάλι ότι τα τέσσερα σύνολα {1, 7}, {2, 8}, {3, 6}, {4, 5}
είναι ξένα μεταξύ τους και η ένωσή τους είναι το σύνολο των τομέων.

Σχετίζονται με κάποιο μαθηματικό τρόπο τα ημιφύλια με τη δομή συγγένειας;

Το σύνολο <m> = {ε, m, m2, m3} είναι, προφανώς, υποομάδα της D4. Ας ορίσουμε μια σχέση ~ μεταξύ
δύο στοιχείων της ομάδας D4 ως εξής: x~y ανν xy-1 ∈ <m>. Παρατήρησε ότι η σχέση αυτή είναι σχέση
ισοδυναμίας, δηλαδή είναι ανακλαστική (x~x), συμμετρική (x~y ανν y~x) και μεταβατική (αν x~y και y~z, τότε
x~z). Είναι φανερό ότι τα τέσσερα μέλη της <m> είναι ισοδύναμα μεταξύ τους, ενώ τα υπόλοιπα τέσσερα
στοιχεία της D4 είναι επίσης ισοδύναμα μεταξύ τους, αλλά δεν είναι ισοδύναμα με κανένα στοιχείο της <m> . Τα
σύνολα {e, m, m2, m3} και {f, mf, m2 f, m3f} είναι οι κλάσεις ισοδυναμίας της σχέσης ισοδυναμίας που ορίσαμε
και λέγονται πλευρικές ομάδες της <m>. Είναι (αναμενόμενα;) τα δύο μητρικά ημιφύλια.

Είναι φανερό ότι το σύνολο <f> = {e, f} αποτελεί υποομάδα της D4. Ας ορίσουμε μια σχέση ~ μεταξύ
δύο στοιχείων της ομάδας D4 ως εξής: x~y ανν xy-1 ∈ <f>. Οι τέσσερις πλευρικές ομάδες είναι οι {1, 7}, {2,
8}, {3, 6}, {4, 5}.

Ας ορίσουμε μια σχέση ~ μεταξύ δύο στοιχείων της ομάδας D4 ως εξής: x~y ανν xy-1 ∈{e, mf} =
υποομάδα της D4. Είναι φανερό ότι mf~e, m2~m3f, m~mf, m3~m2f. Οι πλευρικές ομάδες της {e, mf}
προσδιορίζουν τους νόμιμους γάμους.

Τα γενεαλογικά ημιφύλια είναι οι πλευρικές ομάδες της {e, mf, m2 f, m3f} = υποομάδας της D4.

Είναι φανερό ότι για οποιαδήποτε ομάδα G και γνήσια υποομάδα της Η (Η ≠ G, <e>) μπορούμε να
ορίσουμε μια σχέση ισοδυναμίας στη G όπως παραπάνω, δηλαδή: x~y ανν xy-1 ∈ Η. Οι κλάσεις ισοδυναμίας,
δηλαδή τα σύνολα ισοδύναμων στοιχείων, αποτελούν πάντα διαμέριση του συνόλου (δηλαδή, η ένωσή τους
είναι το σύνολο, ενώ οι κλάσεις είναι, προφανώς, ανά δύο ξένες μεταξύ τους). Είναι φανερό από τον τρόπο
που ορίστηκε η σχέση ισοδυναμίας ότι το πλήθος των στοιχείων σε κάθε κλάση είναι ίσο με την τάξη της
υποομάδας Η. Άρα, η τάξη οποιασδήποτε υποομάδας Η είναι διαιρέτης της τάξης της ομάδας Γ. Αυτό είναι το
λεγόμενο θεώρημα του Lagrange. Άρα, η τάξη οποιουδήποτε στοιχείου μιας ομάδας διαιρεί την τάξη της
ομάδας, αφού κάθε στοιχείο παράγει μια κυκλική υποομάδα με τάξη ίση με την τάξη του στοιχείου.

ΙΙ. Ας θεωρήσουμε πάλι τον τομέα βήτα ως τον τυπικό τομέα e, και ας δούμε όλη τη δομή από τη σκοπιά του.
Τότε, m3(β) = m3(e) = m2(m(e)) = m2(α) = m(m(α)) = m(γ) = β, άρα m3 = e. Επίσης, mfmf(β) = mfm(Ε) =
mf(ζ) = m(Γ) = β, άρα mfmf = e. Επίσης, f2(β) = ff(β) = f(E) = B, επομένως f2 = e.
Καταλήγουμε στον παρακάτω πίνακα και επομένως στη διεδρική ομάδα D6:

∙ m m2 f mf m2f e
2 2
m m e mf mf f m
m2 e m m2f f mf m2
f m2f mf e m2 m f
mf f m2f m e m2 mf
m2f mf f m2 m e m2f
e m m2 f mf m2f e

200 Χατζηκυριάκου Κώστας


Βιβλιογραφία/Αναφορές

Ascher, M. (1991). Ethnomathematics: A Multicultural View of Mathematical Ideas. Brooks/Cole Publishing


Company.
Baumslag, B., & Chandler, B. (1968). Group Theory. Schaum’s Outline Series in Mathematics.
Polya, G. (1998). Πώς να το λύσω. Εκδόσεις Καρδαμίτσα.

Ψυχαγωγικά Μαθηματικά 201


Κεφάλαιο 10
Το ρολόϊ, η σπαζοκεφαλιά 14-15, Κωδωνοκρουσίες

Σύνοψη
Ομάδες μικρής τάξης, άπειρες ομάδες και η συμμετρική ομάδα Σν.

Προαπαιτούμενη γνώση
H έννοια της ομάδας, το θεώρημα του Lagrange, επίλυση τριωνύμου.

Ψυχαγωγικά Μαθηματικά 203


10.1 Το ρολόι
I. Να συμπληρώσεις όλα τα κελιά του παρακάτω πίνακα ακολουθώντας τον υπονοούμενο κανόνα. Ποιος
είναι αυτός;

0 1 2 3 4 5 6 7 8 9 10 11

0 0 10

1 2 5 0

2 3 4

3 6 7 9

5 10

6 8

7 3

8 0

9 11 4 6 7 8

10 10 9

11 5 10

II. Για κάθε φυσικό αριθμό ν, Ν≠ 0, δείξε ότι υπάρχει τουλάχιστον μία ομάδα τάξης ν.

204 Χατζηκυριάκου Κώστας


10.2 Ομάδες μικρής τάξης
I. Ποιες είναι οι ομάδες τάξης 1;

II. Ποιες είναι οι ομάδες τάξης 2;

III. Ποιες είναι οι ομάδες τάξης 3;

IV. Αν ο p είναι πρώτος φυσικός αριθμός, δείξε ότι υπάρχει μόνο μία ομάδα τάξης p που δεν είναι παρά η
κυκλική ομάδα τάξης p.

Ψυχαγωγικά Μαθηματικά 205


V. Να συμπληρώσεις όλα τα κελιά του παρακάτω πίνακα ώστε η δομή ({e, a, b, ab}, *) να είναι ομάδα.

* e a b ab

ab

VI. Ποιες είναι οι μη ισόμορφες ομάδες τάξης 6;

206 Χατζηκυριάκου Κώστας


VII. Να συμπληρώσεις όλα τα κελιά του παρακάτω πίνακα ακολουθώντας τον υπονοούμενο κανόνα. Ποιος
είναι αυτός; (Κατά τη συνήθη αλγεβρική πρακτική το σύμβολο της πράξης μεταξύ των στοιχείων
παραλείπεται).
* e a b c ab ac bc abc
e e a b c ab ac bc abc
a a e ab ac
b b ab e bc
c c ac bc e
ab ab
ac ac
bc bc
abc abc

VIII. Να συμπληρώσεις όλα τα κελιά του παρακάτω πίνακα, εάν a4 = e, b2 = e και ab = ba και η πράξη ∙ κάνει
το σύνολο {e, a, b, a2, a3, ab, a2b, a3b} ομάδα.
∙ e a b a2 a3 ab a2b a3b
e
a
b
a2
a3
ab
a2b
a3b

Ψυχαγωγικά Μαθηματικά 207


IX. Να συμπληρώσεις όλα τα κελιά του παρακάτω πίνακα, εάν ισχύουν οι ακόλουθες ισότητες μεταξύ των 1,
–1, i, j, k, –i, –j, –k: (–1)2 = 1, i2 = j2 = k2 = ijk = –1.
∙ 1 –1 i j k –i –j –k
1
–1
i
j
k
–i
–j
–k

X. Να συμπληρώσεις όλα τα κελιά του παρακάτω πίνακα, εάν ισχύουν οι ακόλουθες ισότητες μεταξύ των a,
b, e: a4 = e, a2 = b2, aba = b.
e a b a2 a3 ab b2 b3
e
a
b
a2
a3
ab
b2
b3

XI. Να βρεις δύο μη ισόμορφες ομάδες τάξης 9.

XII. Να βρεις δύο μη ισόμορφες ομάδες τάξης 10.

208 Χατζηκυριάκου Κώστας


XIII. Να λύσεις την εξίσωση x3 – 1 = 0 και να μελετήσεις το σύνολο των ριζών της.

Ψυχαγωγικά Μαθηματικά 209


10.3 H σπαζοκεφαλιά 14-15
Δεκαπέντε πλακίδια αριθμημένα από το 1 ως το 15 τοποθετούνται σε ένα ξύλινο πλαίσιο που χωρά 16 τέτοια
πλακίδια (4 ανά γραμμή και 4 ανά στήλη). Η κάτω δεξιά θέση είναι κενή. Νόμιμη κίνηση είναι μόνο το
σύρσιμο ενός πλακιδίου που γειτονεύει με την κενή θέση στην κενή θέση. Μπορείς, μέσω μιας ακολουθίας
νόμιμων κινήσεων και μόνο, να πας το πλακίδιο υπ’ αριθμ. 15 στη θέση του πλακιδίου υπ’ αριθμ. 14, το
πλακίδιο υπ’ αριθμ. 14 στη θέση του πλακιδίου υπ’ αριθμ.15, έτσι ώστε όλα τα υπόλοιπα να είναι στην
αρχική θέση τους;

1 2 3 4

5 6 7 8

9 10 11 12

13 14 15

210 Χατζηκυριάκου Κώστας


10.4 Κωδωνοκρουσίες
I. Σε μια εκκλησία υπάρχουν τρεις καμπάνες, η 1η, η 2η, η 3η. Θέλουμε να τις χτυπήσουμε με κάθε δυνατή
σειρά, ακολουθώντας τους εξής κανόνες:
i. Η πρώτη κωδωνοκρουσία είναι η 123, το ίδιο και η τελευταία.
ii. Καμία σειρά δεν επαναλαμβάνεται (πλην της αρχικής και της τελικής, φυσικά).
iii. Από τη μια κωδωνοκρουσία στην άλλη, κάθε καμπάνα μπορεί να μετακινηθεί το πολύ κατά μία θέση.
Μπορείς να βρεις μία τέτοια σειρά κωδωνοκρουσιών;

Ψυχαγωγικά Μαθηματικά 211


II. Σε μία εκκλησία υπάρχουν τέσσερις καμπάνες, η 1η, η 2η, η 3η και η 4η. Θέλουμε να τις χτυπήσουμε με
κάθε δυνατή σειρά, ακολουθώντας τους εξής κανόνες:
i. Η πρώτη κωδωνοκρουσία είναι η 1234, το ίδιο και η τελευταία.
ii. Καμία σειρά δεν επαναλαμβάνεται (πλην της αρχικής και της τελικής, φυσικά).
iii. Από τη μια κωδωνοκρουσία στην άλλη, κάθε καμπάνα μπορεί να μετακινηθεί το πολύ κατά μία
θέση.
Μπορείς να βρεις μία τέτοια σειρά κωδωνοκρουσιών;

212 Χατζηκυριάκου Κώστας


10.5 Ομάδες με άπειρα στοιχεία
I. Είναι το σύνολο των φυσικών αριθμών ομάδα, με πράξη τον πολλαπλασιασμό; Ναι ή όχι; Γιατί;

II. Είναι το σύνολο των (μη μηδενικών) κλασμάτων ομάδα, με πράξη τον πολλαπλασιασμό; Ναι ή όχι;
Γιατί;

III. Είναι το σύνολο των φυσικών αριθμών ομάδα, με πράξη την πρόσθεση; Ναι ή όχι; Γιατί;

IV. Είναι το σύνολο των ακεραίων αριθμών ομάδα, με πράξη την πρόσθεση; Ναι ή όχι; Γιατί;

Ψυχαγωγικά Μαθηματικά 213


V. Στα μαθηματικά, πίνακα 2 × 2 ονομάζουμε μια ορθογώνια διάταξη αριθμών (μιγαδικών) σαν την
παρακάτω:
𝛼11 𝛼12
𝛢 = (𝛼 )
21 𝛼22

Προσθέτουμε πίνακες με τον ακόλουθο τρόπο:

𝑎11 𝑎12 𝑏11 𝑏12 𝑎 + 𝑏11 𝑎12 + 𝑏12


(𝑎 𝑎22 ) + (𝑏21 ) = ( 11 )
21 𝑏22 𝑎21 + 𝑏21 𝑎22 + 𝑏22

και τους πολλαπλασιάζουμε ως εξής:


𝑎11 𝑎12 𝑏11 𝑏12 𝑎 𝑏 + 𝑎12 𝑏21 𝑎11 𝑏12 + 𝑎12 𝑏22
(𝑎 𝑎22 ) ∙ (𝑏21 ) = ( 11 11 )
21 𝑏22 𝑎21 𝑏11 + 𝑎22 𝑏21 𝑎21 𝑏12 + 𝑎22 𝑏22

Διακρίνουσα του πίνακα Α λέμε τον αριθμό 𝑑 (𝛢) = 𝛼11 𝛼22 − 𝛼12 𝛼21 .
𝛼11 𝛼12
Αν για τον πίνακα 𝛢 = (𝛼 𝛼22 ), 𝑑(𝐴) = 𝛼11 𝛼22 − 𝛼12 𝛼21 ≠ 0, τότε:
21

1 0
Α ∙ Α-1 = Α-1 ∙ Α = Ι = ( ),
0 1
𝛼22 −𝛼12
𝑑(𝐴) 𝑑(𝐴)
όπου Α -1
= (−𝛼21 𝛼11 ).
𝑑(𝐴) 𝑑(𝐴)

i. Δείξε ότι το σύνολο των πινάκων με μη μηδενική διακρίνουσα γίνεται ομάδα, με πράξη τον
πολλαπλασιασμό πινάκων. H ομάδα αυτή λέγεται γενική γραμμική ομάδα GL(2, ℂ).
ii. Δείξε ότι το σύνολο των πινάκων με διακρίνουσα 1 και πράξη τον πολλαπλασιασμό πινάκων
είναι υποομάδα της GL(2, ℂ). H υποομάδα αυτή λέγεται ειδική γραμμική ομάδα SL(2, ℂ).
iii. Δείξε ότι μια υποομάδα της παραπάνω ομάδας είναι ισόμορφη με την ομάδα των τετραδονίων,
δείχνοντας ότι η συνάρτηση που ορίζεται μέσω των αντιστοιχιών:

1 0 –1 0 i 0 i 0
1→( ) –1→ ( ) i→( ) –i → ( )
0 1 0 –1 0 –𝑖 0 –𝑖
0 1 0 –1 0 𝑖 0 –𝑖
j→( ) –j → ( ) k→( ) –k → ( )
–1 0 1 0 –𝑖 0 𝑖 0
είναι ισομορφισμός.

214 Χατζηκυριάκου Κώστας


Απαντήσεις στα ερωτήματα του 10ου Κεφαλαίου

10.1 To ρολόι
I. Προφανώς, ο υπονοούμενος κανόνας είναι η αριθμητική του αναλογικού ρολογιού. Το ουδέτερο στοιχείο
εδώ είναι το 12 (= 0).
Παρατήρησε, επίσης, ότι όλα τα στοιχεία μπορούν να παραχθούν με την επαναληπτική πρόσθεση του 1
(αλλά και οποιουδήποτε άλλου στοιχείου) στον εαυτό του. Μια τέτοια ομάδα ονομάζεται κυκλική και είναι
πάντα αντιμεταθετική, δηλαδή a +12 b = b +12 a, για όλα τα στοιχεία της ομάδας.

+12 1 2 3 4 5 6 7 8 9 10 11 12
1 2 3 4 5 6 7 8 9 10 11 12 1
2 3 4 5 6 7 8 9 10 11 12 1 2
3 4 5 6 7 8 9 10 11 12 1 2 3
4 5 6 7 8 9 10 11 12 1 2 3 4
5 6 7 8 9 10 11 12 1 2 3 4 5
6 7 8 9 10 11 12 1 2 3 4 5 6
7 8 9 10 11 12 1 2 3 4 5 6 7
8 9 10 11 12 1 2 3 4 5 6 7 8
9 10 11 12 1 2 3 4 5 6 7 8 9
10 11 12 1 2 3 4 5 6 7 8 9 10
11 12 1 2 3 4 5 6 7 8 9 10 11
12 1 2 3 4 5 6 7 8 9 10 11 12

II. Είναι φανερό ότι οποιοσδήποτε φυσικός αριθμός Ν = ν∙π + υ, όπου 0 ≤ υ < ν.
Αν, λοιπόν, Ν1 = ν∙π1 + υ1, όπου 0 ≤ υ1 < ν, και Ν1 = ν∙π2 + υ2, όπου 0 ≤ υ2 < ν, τότε για το υπόλοιπο υ της
διαίρεσης Ν:ν έχουμε:
𝜐 +𝜐 , αν 𝜐1 + 𝜐2 < 𝜈
υ = {𝜐 1 + 𝜐 2 − 𝜈,
1 2 αν 𝜐1 + 𝜐2 ≥ 𝜈

Έστω, λοιπόν, το σύνολο των υπολοίπων τέτοιων διαιρέσεων {0, 1, 2, …, ν–1} και έστω η πράξη +ν
μεταξύ των στοιχείων του που ορίζεται με τον παραπάνω τρόπο. Η δομή που προκύπτει είναι φανερά ομάδα,
κυκλική, με γεννήτρια οποιοδήποτε τέτοιο υπόλοιπο. Είναι η ομάδα ℤ𝜈 .

Είναι φανερό ότι σε μια ομάδα τάξης ν, ⏟


𝑎 ∗ 𝑎 ∗ … ∗ 𝑎 = e. O μικρότερος φυσικός για τον οποίο
𝜈 𝜑𝜊𝜌έ𝜍
ισχύει κάτι τέτοιο λέγεται τάξη του στοιχείου a.

Ψυχαγωγικά Μαθηματικά 215


10.2 Ομάδες μικρής τάξης
I. Είναι φανερό ότι ο τετριμμένος πίνακας

* e
e e
ορίζει τη μοναδική ομάδα τάξης 1.

Παρατήρηση: Αν φανταστούμε την πράξη * πολλαπλασιαστικά, αντί για e γράφουμε συνήθως 1, ενώ
αν φανταστούμε την πράξη προσθετικά, αντί για e γράφουμε συνήθως 0. Η ομάδα ({0}, +}, όπου 0 + 0 =
1 και η ομάδα ({1}, ∙), όπου 1∙1 =1 είναι ισόμορφες.

II. Φαντάσου έναν οποιονδήποτε φυσικό αριθμό Ν. Διαίρεσέ τον δια του 2. Ποια είναι τα πιθανά υπόλοιπα
αυτής της διαίρεσης; Το 1 (Ν = περιττός) ή το 0 (Ν άρτιος). Αν με 1 εννοήσουμε, λοιπόν, το σύνολο όλων
των περιττών και με 0 το σύνολο όλων των άρτιων, ο παρακάτω πίνακας εκφράζει την προφανή
διαπίστωση ότι το άθροισμα δύο άρτιων ή δύο περιττών είναι άρτιος, ενώ το άθροισμα ενός άρτιου και
ενός περιττού είναι περιττός.
+ 0 1
0 0 1
1 1 0

Είναι φανερό ότι η δομή ({0,1}, *), όπου η πράξη * ορίζεται μέσω του παραπάνω πίνακα, είναι ομάδα
τάξης 2 (δηλαδή έχει 2 στοιχεία) και ότι δεν μπορεί να υπάρχει άλλη, μη ισόμορφη, τέτοια ομάδα
(γιατί;). Επιπρόσθετα λέμε ότι η τάξη του 1 είναι 2 αφού 1 + 1 = 0. Παρατήρησε ότι το 1 μπορεί να
γεννηθεί γεννήτρια της ομάδας, η οποία, εύλογα, λέγεται κυκλική.

III. Φαντάσου έναν οποιονδήποτε φυσικό αριθμό Ν. Διαίρεσέ τον δια του 3. Ποια είναι τα πιθανά υπόλοιπα
αυτής της διαίρεσης; Το 0 (Ν = πολλ3) ή το 1 (Ν = πολλ3 + 1) ή το 2 (Ν = πολλ3 + 2). Αν με 0
εννοήσουμε, λοιπόν, το σύνολο όλων των πολλ3, με 1 το σύνολο όλων των επόμενων πολλ3 και με 2 το
σύνολο όλων των μεθεπόμενων πολλ3, ο παρακάτω πίνακας εκφράζει το πώς συμπεριφέρεται το
άθροισμα οποιωνδήποτε δύο φυσικών σε σχέση με το αν είναι πολλ3, επόμενος ή μεθεπόμενος πολλ3.
+ 0 1 2
0 0 1 2
1 1 2 0
2 2 0 1

Είναι, πάλι, φανερό ότι η δομή ({0,1, 2}, +), όπου η πράξη * ορίζεται μέσω του παραπάνω πίνακα,
είναι ομάδα τάξης 3 (δηλαδή έχει 3 στοιχεία) και ότι δεν μπορεί να υπάρχει άλλη, μη ισόμορφη,
τέτοια ομάδα (γιατί;). Παρατήρησε ότι η τάξη του 1 αλλά και του 2 είναι 3 και ότι η ομάδα είναι πάλι
κυκλική, με γεννήτρια το 1 (1, 2 = 1 + 1, 0 = 1 + 1+1) ή το 2 (2, 2 + 2 = 1, 2 + 2 + 2 = 0).

Η ομάδα που μόλις είδαμε είναι φανερά ισόμορφη με την ομάδα ({1, ω, ω2}, ∙), όπου ∙ ορίζεται
μέσω του παρακάτω πίνακα, δηλαδή ως «πολλαπλασιασμός» δυνάμεων με την ίδια βάση. Ως
ισομορφία μπορούμε να πάρουμε την 1-1 συνάρτηση 0 ⟼1, 1⟼ω, 2⟼ ω2. Υπάρχει άλλη;

∙ 1 ω ω2
1 e ω ω2
ω ω ω2 e
ω 2
ω2
e ω

216 Χατζηκυριάκου Κώστας


IV. Ας υποθέσουμε ότι η ομάδα μας Ο έχει p στοιχεία, p = πρώτος φυσικός αριθμός. Πάρε ένα στοιχείο a,
εκτός του μοναδιαίου 1, και σχημάτισε τα στοιχεία a, a2, . . ., ap-1. Ισχυρίζομαι ότι όλα είναι διαφορετικά.
Πράγματι, αν δύο από αυτά είναι ίσα, ai = aj, (j < i) ας πούμε, τότε ai-j = 1, 0 ≤ 𝑖 − 𝑗 ≤ 𝑝 − 1. Προφανώς,
ΜΚΔ(𝑖 − 𝑗, 𝑝) = 1, οπότε υπάρχουν ακέραιοι m, n έτσι ώστε m∙(𝑖 − 𝑗) + n∙p = 1, αλλά τότε a1 = 1∙1= 1,
άτοπο. Αλλά τότε, Ο = {1, a, a2, . . ., ap-1} και η ομάδα είναι κυκλική.

V. H δομή ({e, a, b, ab}, *) με την πράξη * να εκτελείται έτσι όπως φαίνεται στον παρακάτω πίνακα είναι
ομάδα τάξης 4 και βέβαια δεν είναι κυκλική. Ωστόσο, είναι αντιμεταθετική (παρατήρησε ότι αν
επιχειρούσαμε να θέσουμε ba = a, τότε b = e, άτοπο). Η ομάδα αυτή είναι φανερά ισόμορφη με την
({(0,0), (0,1), (1,0), (1,1), +).

* e a b ab
e e a b ab
a a e ab b
b b ab e a
ab ab b a e

+ (0,0) (0,1) (1,0) (1,1)

(0,0) (0,0) (0,1) (1,0) (1,1)

(0,1) (0,1) (0,0) (1,1) (1,0)

(1,0) (1,0) (1,1) (0,0) (0,1)

(1,1) (1,1) (1,0) (0,1) (0,0)

Είναι φανερό (γιατί;) ότι δεν μπορούν να υπάρχουν άλλες ομάδες τάξης 4, πλην της κυκλικής και της
παραπάνω που είναι γνωστή ως ℤ2 ⊕ ℤ2(= το ευθύ άθροισμα δύο κυκλικών ομάδων τάξης 2).

VI. Υπάρχουν μόνο δύο μη ισόμορφες ομάδες τάξης 6 (γιατί;): η κυκλική ℤ6 και η ομάδα των συμμετριών
του ισόπλευρου, η διεδρική D3. Δεν υπάρχουν άλλες ομάδες τάξης 6.

VII. H δομή ({e, a, b, c, ab, ac, bc, abc}, *) με την πράξη * να εκτελείται έτσι όπως φαίνεται στον παρακάτω
πίνακα είναι ομάδα τάξης 8 και βέβαια δεν είναι κυκλική. Ωστόσο, είναι αντιμεταθετική και φανερά
ισόμορφη με την ({(0,0,0), (0,1,0), (0,0,1), (0,1,1), (1, 0, 0), (1, 1, 0), (1,0,1), (1,1,1)}, +). Η πράξη
ορίζεται μέσω των κανόνων: (α, β, γ) + (α΄, β΄, γ΄) = (α+α΄, β+β΄, γ+γ΄) και 1+1 = 0. Πρόκειται για τη
δομή που είναι γνωστή ως ℤ2 ⊕ ℤ2 ⊕ ℤ2 (= το ευθύ άθροισμα τριών κυκλικών ομάδων τάξης 2).
Προφανώς, δεν είναι ισόμορφη με τη ℤ8 ούτε με την D4.
* e a b c ab ac bc abc
e e a b c ab ac bc abc
a a e ab ac b c abc bc
b b ab e bc a bc c ac
c c ac bc e abc a b ab
ab ab b a abc e bc ac c
ac ac c abc a bc e ab b
bc bc abc c b ac ab e a
abc abc bc ac ab c b a e

Ψυχαγωγικά Μαθηματικά 217


VIII. Αφού a4 = e, b2 = e και ab = ba και θέλουμε η πράξη ∙ να κάνει το σύνολο {e, a, b, a2, a3, ab, a2b, a3b}
ομάδα, ο πίνακας είναι ο ακόλουθος.
∙ e a b a2 a3 ab a2b a3b
e e a b a2 a3 ab a2b a3b
a a a2 ab a3 e a 2b a3b b
b b ab e a2b a3b a a2 a3
a2 a2 a3 a2b e a a 3b b ab
a3 a3 e a3b a a2 b ab a2b
ab ab a 2b a a3b b a2 a3 e
a2b a 2b a 3b a2 b ab a3 e a
a3b a 3b b a3 ab a2b e a a2

Μπορούμε να διακρίνουμε δύο υποομάδες, μία κυκλική τάξης 4 και μία κυκλική τάξης 2, και
παρατηρούμε ότι κάθε στοιχείο της ομάδας είναι το γινόμενο ενός στοιχείου από τη μία και ενός
στοιχείου από την άλλη. Η ομάδα μας είναι το ευθύ άθροισμα ℤ4 ⊕ ℤ2 .

IX. Αφού ισχύουν οι ισότητες (–1)2 = 1, i2 = j2 = k2 = ijk = –1, ο πίνακας της πράξης είναι ο παρακάτω και
είναι φανερό ότι έχουμε ομάδα. Η μη αντιμεταθετική αυτή ομάδα λέγεται ομάδα των τετραδονίων
(quaternions) Q8 και φυσικά δεν είναι ισόμορφη ούτε με τη ℤ8 , ούτε με την D4, ούτε με τη ℤ2 ⊕ ℤ2 ⊕ ℤ2
(γιατί;).
∙ 1 –1 i j k –i –j –k

1 1 –1 i j k –i –j –k
–1 –1 1 –i –j –k i j k
i i –i –1 k –j 1 –k j
j j –j –k –1 i k 1 –i
k k –k j –i –1 –j i 1
–i –i i 1 –k j –1 k –j
–j –j j –k 1 –i k –1 i
–k –k k –j i 1 j –i –1

X. Αφού ισχύουν οι ισότητες a4 = e, a2 = b2, aba = b, ο πίνακας της πράξης είναι ο παρακάτω και είναι
φανερό ότι έχουμε ομάδα. H ομάδα αυτή είναι ισόμορφη με την Q8.
∙ e a a2 a3 b ab a2b a3b
e e a a2 a3 b ab a2b a3b
a a a2 a3 e ab a 2b a3b b
a2 a2 a3 e a a2b a 3b b ab
a3 a3 e a a2 a3b b ab a2b
b b a 3b a2b ab a2 a e a3
ab ab b a3b a2b a3 e a e
a2b a2b ab b a3b e a3 a2 a
a3b a 3b a 2b ab b a e a3 a2

XI. Μία είναι η κυκλική ℤ9 , μία άλλη είναι το ευθύ άθροισμα ℤ3 ⊕ ℤ3 .

XII. Μία είναι η κυκλική ℤ10 (= το ευθύ άθροισμα ℤ5 ⊕ ℤ2 ) και μία άλλη η ομάδα των συμμετριών του
κανονικού πενταγώνου. Ποιες είναι αυτές;

218 Χατζηκυριάκου Κώστας


XIII. Θα λύσουμε την εξίσωση παραγοντοποιώντας: x3−1 = 0 ανν (x−1) ∙ (x2 + x + 1) = 0. Επομένως, μία
λύση είναι η αναμενόμενη, x = 1. Το τριώνυμο x2 + x + 1 = 0 έχει αρνητική διακρίνουσα (−3), συνεπώς
για κάθε πραγματική τιμή του x παίρνει θετική τιμή, δεν μπορεί να είναι μηδέν. Αν φανταστούμε, όμως,
−1+𝑖 √3 −1−𝑖 √3
την ύπαρξη ενός «αριθμού» 𝑖, για τον οποίον 𝑖2 = −1, τότε οι «αριθμοί» x = και x =
2 2
−1+𝑖 √3
είναι ρίζες της (x + x + 1) = 0. Αν θέσουμε ω =
2
, τότε προφανώς ω =1 και ω = −1 − ω =
3 2
2
−1−𝑖 √3
. Με άλλα λόγια, οι τρεις αυτές «μιγαδικές» κυβικές ρίζες της μονάδας αποτελούν κυκλική
2
πολλαπλασιαστική ομάδα, με γεννήτρια οποιαδήποτε από τις δύο «μιγαδικές» ρίζες.

Όπως γράφουν οι R. Courant και H. Robbins στο What is Mathematics?, οι μαθηματικοί του 16ου αιώνα
που σκέφτηκαν να συμβολίσουν την τετραγωνική ρίζα του −1 με i, κατόρθωσαν να λύσουν όλες τις
εξισώσεις δεύτερου, τρίτου και τέταρτου βαθμού, αλλά δεν ήταν σε θέση να νοηματοδοτήσουν «μιγαδικές»
εκφράσεις του τύπου a + ib, a, b ∈ ℝ. Χάρις στη γεωμετρική ερμηνεία, όμως, που προτάθηκε τον 19ο αιώνα,
οι εκφράσεις αυτές απέκτησαν νόημα και νέες μαθηματικές έννοιες και δομές αναδείχθηκαν.

Ποια είναι η γεωμετρική αυτή ερμηνεία; Απλά η έκφραση a + ib, a, b ∈ ℝ δηλώνει το σημείο (a,b) στο
καρτεσιανό επίπεδο. Τότε, όμως, a + ib = ρ(cosφ + isinφ).

Πώς προσθέτουμε δύο τέτοιους αριθμούς; Με τον λεγόμενο κανόνα του παραλληλογράμμου:

Αν z1 = a1 + ib1 = ρ1 (cosφ1 + isinφ1) και z2 = a2 + ib2 = ρ2 (cosφ2 + isinφ2), τότε

z1 + z2 = (a1 + a2) + i (b1 + b2).


Πώς πολλαπλασιάζουμε δύο τέτοιους αριθμούς; Διατηρώντας την επιμεριστική ιδιότητα του
πολλαπλασιασμού ως προς την πρόσθεση:

z1 ∙ z2 = (a1 + ib1) ∙ (a2 + ib2) = (a1 ∙ a2 – b1 ∙ b2) + i (a1 ∙ b2+ b1 ∙ a2) =


ρ1 ρ2 (cos (φ1 + φ2) + i(sin φ1 + φ2)).
𝜋 𝜋 2𝜋 2𝜋
Παρατηρούμε, τώρα, ότι ω = cos + i sin , ω2 = cos + i sin και ω3 = 1 είναι οι τρεις κορυφές ενός
3 3 3 3
ισόπλευρου τριγώνου εγγεγραμμένου στον μοναδιαίο κύκλο (στο μιγαδικό επίπεδο).

10.3 H σπαζοκεφαλιά 14-15


Αν το προσπάθησες, ίσως και να άρχισες να υποψιάζεσαι ότι δεν γίνεται. Πράγματι, δεν γίνεται. Γιατί όμως;
Η πιο διαφωτιστική απάντηση απαιτεί να γνωρίσουμε από κοντά τη συμμετρική ομάδα Σν και μια υποομάδα
της, την εναλλάσσουσα ομάδα Αν.

Έστω το σύνολο των φυσικών αριθμών {1, 2, . . . , ν}. Όπως είναι εύκολο να δούμε, υπάρχουν ν!
διαφορετικές διατάξεις τους, δηλαδή ν! διαφορετικοί τρόποι να τους βάλουμε στη σειρά. (Έχουμε ν επιλογές
για την πρώτη θέση της διάταξης, ν–1 επιλογές για τη δεύτερη κ.ο.κ, οπότε συνολικά ν! επιλογές/διατάξεις).
Ο μετασχηματισμός σ του αρχικού τμήματος 1< 2 < 3 < …< ν σε μια άλλη σειρά σ(1), σ(2), σ(3), . . ., σ(ν)
λέγεται μετάθεση και είναι φανερό ότι υπάρχουν ν! τέτοιες μεταθέσεις.

Άρτιες λέγονται οι μεταθέσεις που είναι το γινόμενο (σύνθεση) άρτιου αριθμού αντιμεταθέσεων,
περιττές αυτές που είναι το γινόμενο (σύνθεση) περιττού αριθμού αντιμεταθέσεων.

1234
Π.χ. η μετάθεση ( ) = (3 2)(2 1)(1 4). Είναι, επομένως, περιττή.
4123
1234
Η μετάθεση (31)(24) = ( ) είναι άρτια.
3 4 12

Ψυχαγωγικά Μαθηματικά 219


μν τ(μ)−τ(ν)
Για κάθε αντιμετάθεση α = (μ,ν) = (ν μ ) (μ > ν), το κλάσμα signα = = -1, οπότε αν signσ =
μ−ν
το γινόμενο των προσήμων των αντιμεταθέσεων από τις οποίες συντίθεται, οι άρτιες μεταθέσεις έχουν
πρόσημο 1 και οι περιττές -1.

Μια μετάθεση είναι ή άρτια ή περιττή (δεν μπορεί να είναι άρτια και περιττή). Το σύνολο των άρτιων
𝜈!
μεταθέσεων είναι υποομάδα της Σν , έχει στοιχεία και λέγεται εναλλάσουσα υποομάδα.
2

Λεπτομέρειες για τις άρτιες και περιττές μεταθέσεις, καθώς και για την Αν, μπορείς να βρεις στο βιβλίο
των B. Baumslag & B. Chandler (1968), Group Theory, Schaum’s Outline Series in Mathematics.

Η Αν (ν >2) μπορεί να παραχθεί από τους 3-κύκλους (a, b, c). Γιατί; Διότι, εξ ορισμού, κάθε στοιχείο
της Αν μπορεί να γραφτεί ως γινόμενο άρτιου πλήθους αντιμεταθέσεων. Πράγματι, αν τα a, b, c, d είναι
διαφορετικά, τότε (a,b)(c,d) = (a, b, c)(b, c, d), αν έχουν κάποιο στοιχείο κοινό, π.χ. a = c, τότε (a, b)(a, d) =
(a, d, b), b = c, τότε (a, b)(b, d) = (a, b, d) και βέβαια (a, b)(a, b) = e.

Ειδικότερα, η Αν (ν >2) μπορεί να παραχθεί από τους κύκλους (123), (234), . . . (ν−2, 𝜈 − 1, 𝜈). Η
απόδειξη γίνεται με επαγωγή.

Είναι φανερό ότι η πρόταση αυτή αληθεύει για ν = 3. Έστω ότι αληθεύει για την Α ν (ν > 2), θα
δείξουμε ότι αληθεύει για την Αν+1. Ταυτίζουμε τα στοιχεία της Αν+1 που δεν μετακινούν το ν+1 με τα
στοιχεία της Αν. Πάρε μια μετάθεση σ της Αν+1. Αν σ(ν+1) = ν+1. Τότε, η σ ανήκει στην Αν και επομένως από
την υπόθεσή μας παράγεται από διαδοχικούς 3-κύκλους που δεν περιέχουν το ν+1.

Αν σ(ν+1) = δ < ν+1, τότε, φυσικά, υπάρχει τ (της Α ν), τέτοιο που τ(δ) = ν. Τότε, ρ(ν+1) = (ν-1, ν,
ν+1)τσ(ν+1) = ν+1, οπότε ρ είναι μετάθεση της Αν. Αλλά τότε, η σ = τ-1(ν-1, ν, ν+1)2ρ μπορεί να παραχθεί από
διαδοχικούς κύκλους και τελειώσαμε.

Αριθμούμε τα 16 κελιά με τον τρόπο που φαίνεται παρακάτω. Είναι φανερό ότι νόμιμη κίνηση είναι η
μετακίνηση του κενού κελιού, δηλαδή η αντιμετάθεσή του με ένα από τα γειτονικά, κατακορύφως ή
οριζοντίως, πλακίδια. Κάθε τοποθέτηση των 15 πλακιδίων στα 16 κελιά είναι μια 1-1 συνάρτηση μεταξύ του
συνόλου των {1, 2, …, 15, ∅} και του συνόλου {1, 2, …, 16}.

Δοθείσης μιας αρχικής τοποθέτησης, θέλουμε να προσδιορίσουμε ποιες τοποθετήσεις μπορούν να


επιτευχθούν μέσω μιας ακολουθίας νόμιμων κινήσεων.

220 Χατζηκυριάκου Κώστας


Παρατήρησε ότι μετακινώντας την κενή θέση κατά μήκος του ελικοειδούς μονοπατιού, μπορούμε να τη
φέρουμε σε οποιοδήποτε κελί χωρίς να αλλάξει η σειρά των άλλων πλακιδίων πάνω στο μονοπάτι. Αυτό μας
οδηγεί στην ιδέα να ορίσουμε δύο τοποθετήσεις ως ισοδύναμες, εφόσον η μία προκύπτει από την άλλη μέσω
της μετακίνησης της κενής θέσης κατά μήκος του μονοπατιού αυτού. Θα λέμε τις ισοδύναμες κλάσεις
σχηματισμούς και είναι φανερό ότι κάθε τέτοιος σχηματισμός περιέχει 16 διαφορετικές τοποθετήσεις που
αντιστοιχούν στις 16 διαφορετικές θέσεις στις οποίες μπορεί να μην υπάρχει πλακίδιο. Εάν το πλακίδιο i
βρίσκεται στο κελί j και η κενή θέση είναι σε κελί υψηλότερης αρίθμησης, τότε λέμε ότι το πλακίδιο i
βρίσκεται σε υποδοχή j, αλλιώς σε υποδοχή j-1. Όλες οι τοποθετήσεις σε έναν σχηματισμό έχουν τα 15
πλακίδια στις ίδιες υποδοχές, επομένως μπορούμε να συμβολίσουμε με [α1, α2, . . ., α15] τον σχηματισμό, όπου
αi είναι η υποδοχή στην οποία βρίσκεται το πλακίδιο i. Εάν εφαρμόσουμε τη μετάθεση σ στον σχηματισμό
[α1, α2, . . ., α15], παίρνουμε τον σχηματισμό [σ(α1), σ(α2), . . ., σ(α15)]. Από κάθε μετακίνηση της κενής θέσης
προκύπτει μια μετάθεση των υποδοχών στα οποία βρίσκονται τα πλακίδια. Παραδείγματος χάριν, η
μετακίνηση της κενής θέσης από το κελί 10 στο κελί 15 αντιστοιχεί στη μετάθεση (10, 11, 12, 13, 14), αφού
το πλακίδιο που ήταν στο κελί 15 (υποδοχή 14) πηγαίνει στο κελί 10 (υποδοχή τώρα 0), ενώ τα πλακίδια στα
κελιά 11, 12, 13, 14 αποκτούν υποδοχή που είναι κατά μία μονάδα μεγαλύτερη.

Παράδειγμα: Η τοποθέτηση Β στον παρακάτω πίνακα αντιστοιχεί, προφανώς, στον σχηματισμό [1, 2,
3, 4, 8, 7, 6, 5, 14, 12, 13, 10, 15, 11, 9]. Από την άλλη, η αρχική τοποθέτηση Α αντιστοιχεί στον σχηματισμό
[1, 2, 3, 4, 8, 7, 6, 5, 9, 10, 11, 12, 15, 14, 13]. Αν στην αρχική τοποθέτηση εφαρμοστεί η μετάθεση (10,12)(9,
14, 11, 13), θα έχουμε (10,12)(9, 14, 11, 13)(Α) = (10, 12) ([1, 2, 3, 4, 8, 7, 6, 5, 14, 10, 13, 12, 15, 11, 9]) =
[1, 2, 3, 4, 8, 7, 6, 5, 14, 12, 13, 10, 15, 11, 9]. Συνεπώς, ο σχηματισμός Β μπορεί να προκύψει από τον Α,
αφού η μετάθεση σ είναι άρτια (όπως θα δείξουμε παρακάτω).

Αν με σi,j δηλώσουμε τη μετάθεση που αντιστοιχεί στη μετακίνηση της κενής θέσης από το κελί i στο
κελί j, τότε σi,i+1 είναι η ταυτοτική μετάθεση και σi,j = σ−1
i,j .

1 2 3 4

5 6 7 8

15 12 14

13 9 11 10

Συνεπώς, μένουμε με τις εννέα μεταθέσεις, σ1,8 = (1, 2, 3, 4, 5, 6, 7), σ2,7 = (2, 3 ,4, 5, 6), σ3,6 = (3, 4, 5), σ5,12
= (5, 6, 7, 8, 9, 10, 11), σ6,11 = (6, 7, 8, 9, 10), σ7,10 = (7, 8, 9), σ9,16 = (9, 10, 11, 12, 13, 14, 15), σ10,15 = (10, 11,
12, 13, 14), σ11, 14 = (11, 12, 13) (βλ. την ελικοειδή γραμμή στην εικόνα), οι οποίες, μαζί με τις αντίστροφές
τους, μπορούν να εφαρμοστούν υπό οποιαδήποτε σειρά και έτσι το πρόβλημά μας ανάγεται στο να βρούμε
ποια είναι η υποομάδα της Σ15 (μεταθέσεις των 15 υποδοχών) που παράγεται από αυτές τις μεταθέσεις. Η
ομάδα αυτή είναι η Α15 (η ομάδα των άρτιων μεταθέσεων). Γιατί;

Διότι οι παραπάνω εννέα κύκλοι είναι άρτιες μεταθέσεις και επομένως παράγουν μια υποομάδα
υποσύνολο της Α15. Παράγουν, όμως, και όλους τους διαδοχικούς 3-κύκλους της Σ15 και επομένως παράγουν
την ίδια Α15. Πράγματι, όταν το ν παίρνει τις τιμές -2, -1, 0, 1, 2, έχουμε:

(1, 2, …, 7)-ν(3, 4, 5)(1, 2, …, 7)ν = (1, 2, 3), . . ., (5, 6, 7)

Ψυχαγωγικά Μαθηματικά 221


(5, 6, ..., 11)-ν(7, 8, 9)(5, 6, …, 11)ν = (5, 6, 7), . . ., (9, 10, 11) και
(9, 10, ..., 15) -ν(11, 12, 13)(9, 10, …, 15)ν = (9, 10, 11), . . . (13, 14, 15).
Καταλήγουμε, επομένως, ότι αν έχουμε δύο τοποθετήσεις Τ1 και Τ2, οι οποίες ανήκουν στους
σχηματισμούς Σχ1 και Σχ2 αντίστοιχα, τότε ο σχηματισμός Σχ 2 προκύπτει από τον Σχ1 ανν ο Σχ2 είναι άρτια
μετάθεση του Σχ1. Το συμπέρασμα αυτό μπορεί να επαναδιατυπωθεί και ως εξής: αν οι τοποθετήσεις Τ 1 και
Τ2 έχουν την κενή θέση στο ίδιο κελί, τότε η τοποθέτηση Τ 2 προκύπτει από την τοποθέτηση Τ1 αν η Τ2 είναι
άρτια μετάθεση των 15 αριθμημένων πλακιδίων της Τ1.

Αλλά στην σπαζοκεφαλιά 14-15, η ζητούμενη τοποθέτηση είναι περιττή μετάθεση της αρχικής,
επομένως η σπαζοκεφαλιά δεν λύνεται!

10.4 Κωδωνοκρουσίες
I. Παρακάτω μπορείς να δεις έναν απλό τρόπο να το κάνεις αυτό. Η κωδωνοκρουσία αυτή είναι γνωστή ως
singles.

1 2 3

↓ ↘ ↙

1 3 2

↙ ↘ ↓

3 1 2

↓ ↘ ↙

3 2 1

↘ ↙ ↓

2 3 1

↓ ↘ ↙

2 1 3

↙ ↘ ↓

1 2 3

222 Χατζηκυριάκου Κώστας


Να ένας τρόπος να χτυπήσουν τέσσερις καμπάνες, σύμφωνα με τους κανόνες που τέθηκαν:

Δ 1 2 3 4 Ρ 2 3 1 4 Ν 3 1 2 4

↓ ↓ ↙ ↘ ↓ ↓ ↘ ↙ ↓ ↓ ↘ ↙

Γ 1 2 4 3 Υ 2 3 4 1 Ο 3 1 4 2

↓ ↘ ↙ ↓ ↓ ↘ ↙ ↓ ↓ ↘ ↙ ↓

Β 1 4 2 3 Ξ 2 4 3 1 Φ 3 4 1 2

↘ ↙ ↓ ↓ ↘ ↙ ↓ ↓ ↘ ↙ ↓ ↓

Θ 4 1 2 3 Μ 4 2 3 1 Ι 4 3 1 2

↓ ↘ ↙ ↓ ↓ ↘ ↙ ↓ ↓ ↘ ↙ ↓

Κ 4 2 1 3 Λ 4 3 2 1 Η 4 1 3 2

↘ ↙ ↓ ↓ ↙ ↘ ↓ ↓ ↘ ↙ ↓ ↓

Ψ 2 4 1 3 Χ 3 4 2 1 Α 1 4 3 2

↓ ↘ ↙ ↓ ↓ ↘ ↙ ↓ ↓ ↘ ↙ ↓

Σ 2 1 4 3 Ω 3 2 4 1 Ζ 1 3 4 2

↓ ↓ ↘ ↙ ↓ ↓ ↙ ↘ ↓ ↓ ↙ ↘

Π 2 1 3 4 Τ 3 2 1 4 Ε 1 3 2 4

↓ ↘ ↙ ↓ ↓ ↘ ↙ ↓ ↓ ↘ ↙ ↓

(1 2 3 4)

Ψυχαγωγικά Μαθηματικά 223


Μια κωδωνοκρουσία σαν την παραπάνω λέγεται Plain Bob Minimus. Πρόσεξε ότι η σειρά με την οποία
κρούεται η κάθε καμπάνα είναι σχετικά εύκολα απομνημονεύσιμη. Π.χ. ο χειριστής της καμπάνας 4 τη
χτυπάει 4ος, 3ος, 2ος, 1ος, ξανά 1ος, 2ος, 3ος, 4ος, ξανά 4ος, 3ος, 2ος, 1ος, ξανά 1ος, 2ος, 3ος, 4ος, ξανά 4ος, 3ος, 2ος 1ος και
ξανά 1ος, 2ος, 3ος, 4ος. Αντίστοιχα μνημονικά μοτίβα βοηθούν και τους άλλους χειριστές, σε κάθε περίπτωση
όμως, απαιτείται η απομνημόνευση μεγάλων αριθμητικών μοτίβων.

Αν αναστοχαστούμε τη λύση αυτή, όπως μας προτείνει ο Polya να κάνουμε πάντα, μπορούμε να δούμε
ότι ο τρόπος Plain Bob Minimus προκύπτει από τον απλό τρόπο ως εξής: Γράφουμε κάθε στοιχείο του απλού
τρόπου 4 φορές και τοποθετούμε το 4 ανάμεσα στα ψηφία του στοιχείου με τον τρόπο που υπονοείται. Είναι
φανερό ότι ο τρόπος αυτός γενικεύεται, δηλαδή αν έχουμε έναν τρόπο τέτοιων κωδωνοκρουσιών για ν
καμπάνες, μπορούμε έτσι να παραγάγουμε έναν τρόπο τέτοιων κωδωνοκρουσιών για ν +1 καμπάνες.

Μπορούμε, επίσης, να παρατηρήσουμε ότι οι τρεις στήλες αποτελούν μια διαμέριση της ομάδας S4.
Μήπως πρόκειται για τις πλευρικές ομάδες μιας σχέσης ισοδυναμίας σαν και αυτήν που είδαμε στο
Κεφάλαιο 8; Πράγματι, πρόκειται για τις πλευρικές ομάδες που προκύπτουν από τη σχέση ισοδυναμίας σ~τ
αν στ-1 ∈ D4, τη διεδρική ομάδα τάξης 8 που συγκροτούν οι συμμετρίες του τετραγώνου.

Η επόμενη στρατηγική κωδωνοκρουσίας είναι γνωστή ως Plain Bob. Τα μετασχηματιστικά σχήματα


σύμφωνα με τα οποία αλλάζει η σειρά στην κωδωνοκρουσία δεν είναι παρά δύο αξονικές συμμετρίες του
τετραγώνου (η συμμετρία ως προς την κατακόρυφη διάμεσο και η συμμετρία ως προς τη διαγώνιο από
αριστερά στα δεξιά, οι οποίες επαναλαμβάνονται εναλλάξ στο τετράγωνο του οποίου οι διαδοχικές κορυφές,
στη φορά των δεικτών του ωρολογίου, είναι οι 1243).

1 2 4 3 1 3 4 2 1 4 2 3

↙ ↘ ↙ ↘ ↙ ↘ ↙ ↘ ↙ ↘ ↙ ↘

2 1 4 3 3 1 2 4 4 1 3 2

↓ ↘ ↙ ↓ ↓ ↘ ↙ ↓ ↓ ↘ ↙ ↓

2 4 1 3 3 2 1 4 4 3 1 2

↘ ↙ ↘ ↙ ↘ ↙ ↘ ↙ ↘ ↙ ↘ ↙

4 2 3 1 2 3 4 1 3 4 2 1

↓ ↘ ↙ ↓ ↓ ↘ ↙ ↓ ↓ ↘ ↙ ↓

4 3 2 1 2 4 3 1 3 2 4 1

↘ ↙ ↘ ↙ ↘ ↙ ↘ ↙ ↘ ↙ ↘ ↙

3 4 1 2 4 2 1 3 2 3 1 4

↓ ↘ ↙ ↓ ↓ ↘ ↙ ↓ ↓ ↘ ↙ ↓

3 1 4 2 4 1 2 3 2 1 3 4

↘ ↙ ↘ ↙ ↘ ↙ ↘ ↙ ↘ ↙ ↘ ↙

1 3 2 4 1 4 3 2 1 2 4 3

↓ ↓ ↘ ↙ ↓ ↓ ↘ ↙ ↓ ↓ ↘ ↙

(1 2 3 4)

224 Χατζηκυριάκου Κώστας


Έως τώρα, πίσω από τη σπαζοκεφαλιά των κωδωνοκρουσιών ανακαλύψαμε τη θεωρία των ομάδων. Είναι
ενδιαφέρον να παρατηρήσουμε ότι πίσω από τη σπαζοκεφαλιά αυτή μπορούμε να ανακαλύψουμε και τη
θεωρία γραφημάτων. Αν κοιτάξουμε προσεκτικά τον πίνακα του τρόπου Plain Bob Minimus, μπορούμε να
δούμε στη διαδοχή των σειρών με τις οποίες χτυπιούνται οι καμπάνες ένα χαμιλτονιανό κύκλωμα στο
γράφημα με 24 κορυφές (όλες τις διαφορετικές μεταθέσεις των 1, 2, 3, 4) και 48 ακμές (δυο κορυφές
συνδέονται με ακμή ανν η μία μετάθεση προκύπτει από την άλλη με τη μετακίνηση ενός μόνο ψηφίου κατά
μία μόνο θέση). Το γράφημα αυτό είναι χαμιλτονιανό και παράγει 10.792 διαφορετικούς τρόπους Plain Bob
Minimus κωδωνοκρουσιών (Βλ. το γράφημα του Σχήματος 10.1).

Σχήμα 10.1

10.5 Ομάδες με άπειρα στοιχεία


I. Το σύνολο των φυσικών αριθμών δεν αποτελεί ομάδα ως προς τον πολλαπλασιασμό, αφού το γινόμενο
δύο φυσικών αριθμών (διαφορετικών από το 1) δεν μπορεί ποτέ να ισούται με 1.

II. Το σύνολο των μη μηδενικών κλασμάτων αποτελεί ομάδα ως προς τον πολλαπλασιασμό, αφού κάθε μη
μηδενικό κλάσμα έχει αντίστροφο και η προσεταιριστική ιδιότητα, φυσικά, ισχύει.

III. Το σύνολο των φυσικών αριθμών δεν αποτελεί ομάδα ως προς την πρόσθεση, αφού το άθροισμα δύο
φυσικών αριθμών (μη μηδενικών) δεν μπορεί ποτέ να ισούται με 0.

IV. Το σύνολο των ακέραιων αριθμών αποτελεί ομάδα ως προς την πρόσθεση, αφού κάθε ακέραιος έχει
(μοναδικό) αντίθετο.

Ψυχαγωγικά Μαθηματικά 225


V. Ομάδες πινάκων:
iv. Δείξε ότι το σύνολο των πινάκων με μη μηδενική διακρίνουσα γίνεται ομάδα, με πράξη τον
πολλαπλασιασμό πινάκων. H ομάδα αυτή λέγεται γενική γραμμική ομάδα GL(2, ℂ). Προφανές.
v. Δείξε ότι το σύνολο των πινάκων με διακρίνουσα 1 και πράξη τον πολλαπλασιασμό πινάκων
είναι υποομάδα της GL(2, ℂ). H υποομάδα αυτή λέγεται ειδική γραμμική ομάδα SL(2, ℂ).
Προφανές.
vi. Προφανώς, η υποομάδα της SL(2, ℂ) που έχει ως στοιχεία τους οκτώ πίνακες για τους οποίους
ισχύει:

–1 0 –1 0 (– 1) ∙ (– 1) + (– 1) ∙ 0 (– 1) ∙ 0 + 0 ∙ (– 1) 1 0
( )∙( )=( )=( ),
0 –1 0 –1 0 ∙ (– 1) + (– 1) ∙ 0 0 ∙ 0 + (– 1) ∙ (– 1) 0 1
i 0 i 0 –1 0 0 1 0 1 –1 0 0 i 0 i –1 0
( )∙( )= ( ), ( )∙( )=( ), ( )∙( )=( ),
0 –i 0 –i 0 –1 –1 0 –1 0 0 –1 i 0 i 0 0 –1
i 0 0 1 0 i 0 i 0 i –1 0
( )∙( )∙( )= ( )∙( )=( ).
0 –i –1 0 i 0 i 0 i 0 0 –1
είναι η ζητούμενη υποομάδα και η παρακάτω αντιστοιχία είναι ο ζητούμενος ισομορφισμός:

1 0 –1 0 i 0 i 0
1→( ) –1→ ( ) i→( ) –i → ( )
0 1 0 –1 0 –i 0 –i
0 1 0 –1 0 i 0 –i
j→( ) –j → ( ) k→( ) –k → ( )
–1 0 1 0 i 0 –i 0

226 Χατζηκυριάκου Κώστας


Βιβλιογραφία/Αναφορές

Archer, A. F. (1999). A Modern Treatment of the 15 Puzzle. The American Mathematical Monthly, 106(9),
793-799.
Averbach, B., & Chein O. (2000). Problem Solving Through Recreational Mathematics. Dover Publications
Inc.
Baumslag B., & Chandler, B. (1968). Group Theory. Schaum’s Outline Series in Mathematics.
McCoy, N. H., & Janusz, G. J. (1992). Introduction to Modern Algebra (5th edition). WCB.
Polster, B., & Marty R. (2009). Ringing the Changes. Plus Magazine, 1-9.
https://plus.maths.org/issue53/features/polsteross/2pdf/index.html/op.pdf
Polya, G. (1998). Πώς να το λύσω. Εκδόσεις Καρδαμίτσα.
Stein, S. K. (1999). Mathematics, The Man-Made Universe. Dover Publications.

Ψυχαγωγικά Μαθηματικά 227


Κεφάλαιο 11
Νούμερα

Σύνοψη
Αριθμητικές σπαζοκεφαλιές, κρυπτάριθμοι.

Προαπαιτούμενη γνώση
Γραφή, ανάγνωση και αριθμητική.

Ψυχαγωγικά Μαθηματικά 229


11.1 Ζυγίσματα
I. Έχεις 27 οπτικά όμοια μπαλάκια του τένις από τα οποία ένα είναι ελλιποβαρές. Χρησιμοποιώντας μια
κοινή ζυγαριά με δύο δίσκους και χωρίς σταθμά, να βρεις το ελλιποβαρές κάνοντας μόνο τρεις ζυγίσεις.

230 Χατζηκυριάκου Κώστας


II. Έχεις 13 οπτικά όμοια μπαλάκια του τένις από τα οποία ένα είναι ελλιποβαρές. Χρησιμοποιώντας μια
κοινή ζυγαριά με δύο δίσκους και χωρίς σταθμά, να βρεις το ελλιποβαρές κάνοντας μόνο τρεις ζυγίσεις.

Ψυχαγωγικά Μαθηματικά 231


III. Ένας χονδρέμπορος έχει στο μαγαζί του μία ζυγαριά και επτά είδη σταθμών, με τα οποία μπορεί να
ζυγίζει σώματα οποιουδήποτε ακέραιου βάρους από 1 ως 127 κιλά.Τι είδους σταθμά έχει και πώς
κατορθώνει να ζυγίζει τα προϊόντα του; Δεν επιτρέπεται αντίβαρο.

232 Χατζηκυριάκου Κώστας


IV. Ένας χονδρέμπορος έχει στο μαγαζί του μία ζυγαριά και τρία είδη σταθμών, με τα οποία μπορεί να
ζυγίζει σώματα οποιουδήποτε ακέραιου βάρους από 1 ως και 13 κιλά. Τι είδους σταθμά έχει και πώς
κατορθώνει να ζυγίζει τα προϊόντα του; Επιτρέπεται αντίβαρο.
Μελέτησε το ίδιο πρόβλημα για το ζύγισμα οποιουδήποτε ακέραιου βάρους από 1 ως και 40 κιλά και
οποιουδήποτε ακέραιου βάρους από 1 ως και 121 κιλά.

Ψυχαγωγικά Μαθηματικά 233


11.2 Μεταγγίσεις
I. Δύο φίλοι αγόρασαν μια νταμιτζάνα με 8 κιλά κρασί και τώρα θέλουν να το μοιραστούν. Για τη μοιρασιά
διαθέτουν μόνο δύο δοχεία που το ένα χωρά 5 κιλά και το άλλο 3 κιλά. Θα μπορέσουν να κάνουν τη
μοιρασιά, να πάρει, δηλαδή, ο καθένας από 4 κιλά κρασί;

234 Χατζηκυριάκου Κώστας


II. Η κυρία Παναγιώτα, χρησιμοποιώντας μόνο δύο δοχεία, ένα που χωρά 4 κιλά κρασί και ένα που χωρά 7
κιλά, μπορεί τελικά να αδειάσει από το μεγάλο κρασοβάρελό της οποιαδήποτε ποσότητα κρασιού τής
ζητηθεί από τους πελάτες. Πώς θα πουλήσει 2 κιλά κρασί; (Στην προσπάθειά της να μετρήσει τη
ζητούμενη ποσότητα, μπορεί να ξαναρίξει κάποια ποσότητα κρασιού στο βαρέλι).

Ψυχαγωγικά Μαθηματικά 235


11.3 Μάντεψε!
Κάνε το παρακάτω ταχυδακτυλουργικό κόλπο σε μια φίλη σου. Παρουσίασέ της τις παρακάτω τέσσερις
κάρτες:

Α Β

1 3 5 7 2 3 6 7
9 11 13 15 10 11 14 15

Γ Δ

4 5 6 7 8 9 10 11
12 13 14 15 12 13 14 15

Ζήτα από τη φίλη σου να βάλει στον νου της, χωρίς να σου αποκαλύψει, έναν αριθμό από το 1 ως το 15 και
να σου πει σε ποιες κάρτες ο αριθμός αυτός εμφανίζεται. Εσύ δεν βλέπεις τις κάρτες από τη μεριά που είναι
γραμμένοι οι αριθμοί, αλλά από την πίσω μεριά όπου είναι σημειωμένο μόνο το γράμμα που αντιστοιχεί στην
κάρτα.

Όταν η φίλη σού πει σε ποιες κάρτες είναι ο αριθμός που έβαλε στον νου της, εσύ προσθέτεις γρήγορα
και νοερά τους χαρακτηριστικούς αριθμούς των καρτών αυτών που είναι 1 για την Α, 2 για την Β, 4 για την Γ
και 8 για την Δ. Το άθροισμα είναι ο αριθμός που έβαλε στον νου της η φίλη σου.

Πείσου ότι το κόλπο πετυχαίνει πάντα. Εξήγησε γιατί πετυχαίνει.

Μπορεί να γίνει το ίδιο κόλπο με περισσότερους αριθμούς; Αν ναι, με ποιους; Πόσες κάρτες
χρειάζονται τότε;

236 Χατζηκυριάκου Κώστας


11.4 Κρυπτάριθμοι
Να αντικαταστήσεις τα γράμματα με ψηφία (κάθε γράμμα αντικαθιστά ένα μοναδικό ψηφίο και διαφορετικά
γράμματα αντικαθιστούν διαφορετικά ψηφία) ώστε να ισχύουν οι παρακάτω πράξεις:

ΟΚΤΩ ΔΥΟ ΛΙΓΟ

+ ΔΥΟ  ΔΥΟ +ΝΕΡΟ

ΔΕΚΑ ΠΕΝΤΕ ΝΕΦΟΣ

Η σπαζοκεφαλιά αυτή λέγεται κρυπτάριθμος. Νονός της ήταν ο Simon Vatriquant (ψευδώνυμο
Minos) στο περιοδικό Sphinx (Μάιος 1931), αν και τέτοιου είδους σπαζοκεφαλιές προϋπήρχαν. Σημειώνει ο
Minos, «οι κρυπτογράφοι αντικαθιστούν τα γράμματα με αριθμούς. Σε αντίποινα, εμείς αντικαταστήσαμε
τους αριθμούς με γράμματα».

Ψυχαγωγικά Μαθηματικά 237


Απαντήσεις στα ερωτήματα του 11ου Κεφαλαίου

11.1 Ζυγίσματα
I. Χωρίζουμε τα 27 μπαλάκια σε 3 ομάδες Α, Β, Γ των 9. Συγκρίνουμε ζυγίζοντας τις ομάδες Α και Β. Αν
είναι ισοβαρείς, τότε το ελλιποβαρές βρίσκεται στην ομάδα Γ. Αν μία από τις Α, Β είναι ελλιποβαρής,
τότε το ελλιποβαρές μπαλάκι βρίσκεται στην Α ή τη Β αντίστοιχα. Ούτως ή άλλως, έχουμε τώρα 9
μπαλάκια να ελέγξουμε. Τα χωρίζουμε πάλι σε τρεις ομάδες Α 1, Β1, Γ1 των 3. Όπως και πριν, βρίσκουμε
την ελλιποβαρή ομάδα με ένα ακόμη ζύγισμα. Τώρα έχουμε να ελέγξουμε μόνο τρία μπαλάκια α, β, γ. Με
ένα ακόμη ζύγισμα βρίσκουμε το ελλιποβαρές μπαλάκι.

II. Αν τα μπαλάκια είναι 13, χώρισε σε 3 ομάδες Α, Β, Γ των 6, 6, 1. Συγκρίνουμε ζυγίζοντας τις ομάδες Α
και Β. Αν είναι ισοβαρείς, τότε το ελλιποβαρές είναι το μπαλάκι που είναι μόνο του. Αν μία από τις Α, Β
είναι ελλιποβαρής, τότε το ελλιποβαρές μπαλάκι βρίσκεται στην Α ή τη Β αντίστοιχα. Τα 6 μπαλάκια
της Α ή της Β ομάδας τα χωρίζουμε σε 3 ομάδες Α 1, Β1, Γ1 των 2. Βρίσκουμε και πάλι με ένα ζύγισμα
την ελλιποβαρή ομάδα. Τώρα, έχουμε να ελέγξουμε 2 μπαλάκια α, β. Με ένα ακόμη ζύγισμα βρίσκουμε
το ελλιποβαρές μπαλάκι.

III. Παρατηρώ ότι 127 = 27 –1. Οποιοσδήποτε φυσικός αριθμός (οπότε και ακέραιο βάρος) από 1 έως και
127 μπορεί να γραφτεί στο δυαδικό σύστημα στη μορφή 𝛂 ∙ 𝟐𝟔 + 𝛃 ∙ 𝟐𝟓 + 𝛄 ∙ 𝟐𝟒 + 𝛅 ∙ 𝟐𝟑 + 𝛆 ∙ 𝟐𝟐 +
𝛇 ∙ 𝟐 + 𝛈, με α, β, γ, δ, ε, ζ, η = 0 ή 1. Επομένως, για να ζυγίσω τα βάρη από 1 έως και 127, χρειάζομαι τα
σταθμά για 1, 2, 4, 8, 16, 32 και 64 κιλά. Είναι φανερό ότι για να ζυγίσουμε όλα τα κιλά ως το ν,
χρειαζόμαστε τα σταθμά 1, 2, …, 2k, όπου 2k ≤ ν < 2k+1. Είναι, όμως, τα k+1 αυτά σταθμά τα λιγότερα
δυνατά; Η απάντηση είναι ναι και να γιατί. Θα δείξουμε ότι αν αρκούν k σταθμά για να ζυγίσουμε όλα
τα βάρη ως το ν, τότε ν < 2k. Δηλαδή, αν 2k ≤ ν < 2k+1, τότε χρειαζόμαστε τουλάχιστον k+1 σταθμά και
επειδή τα 1, 2, . . ., 2k κάνουν τη δουλειά αυτή, άρα k+1 είναι ο μικρότερος δυνατός αριθμός. Προφανώς,
για να ζυγίσουμε βάρος 1 κιλού, χρειαζόμαστε το βάρος β1 = 1. Για να ζυγίσουμε βάρος 2 κιλών,
χρειαζόμαστε ακόμη ένα βάρος β2 που δεν ξεπερνά τα 2 κιλά (δηλαδή. β2 = 1 ή 2). Τώρα, β1 + β2 ≤ 3.
Άρα, για να ζυγίσουμε βάρος 4 κιλών, χρειαζόμαστε και άλλο βάρος β3 που δεν ξεπερνά τα 4 κιλά. Αλλά
τώρα, β1 + β2 + β3 ≤ 7, οπότε για να ζυγίσουμε βάρος 8 κιλών, χρειαζόμαστε κι άλλο βάρος β4 που δεν
ξεπερνά τα 8 κιλά. Με άλλα λόγια, για κάθε βάρος ρ πρέπει να έχουμε κάποιο βάρος βρ που δεν ξεπερνά
τα 2ρ-1 κιλά. Οπότε, αφού έχουμε k βάρη, β1 + β2 + … + βκ ≤ 1 + 2 + … + 2k-1 = 2 k –1. Άρα, ν ≤ 2k –1<
2k.

IV. Τα σταθμά για το ζύγισμα βάρους έως και 13 (= 1 + 3 + 9) κιλών είναι 1 κιλού, 3 κιλών, 9 κιλών, αφού
κάθε αριθμός από 1 ως και 13 μπορεί να γραφτεί στη μορφή 𝛂 ∙ 𝟑𝟐 + 𝛃 ∙ 𝟑 + 𝛄, με α, β, γ = 0, 1 ή 2.
Αλλά 2∙ 𝟑𝒌 = 𝟑𝒌+𝟏 – 𝟑𝒌 , επομένως οι συντελεστές των δυνάμεων του 3 στην κανονική αυτή μορφή
μπορούν να είναι οι 0, 1, -1 και φυσικά ο συντελεστής -1 να δηλώνει αντίβαρο. Έτσι, 13 = 9 + 3 +1,
αλλά 11 = 9 + 2 = 9 + 3 – 1 και 7 = 2∙ 𝟑 + 1 = 9 – 3 + 1. Αν σκεφτούμε ανάλογα για το ζύγισμα όλων
των ακέραιων βαρών έως και 40 (= 1 + 3 + 9 + 27) κιλά, χρειαζόμαστε τα σταθμά 1, 3, 9, 27, και για το
ζύγισμα όλων των ακέραιων βαρών έως και 121 (= 1 + 3 + 9 + 27 + 81) κιλά, χρειαζόμαστε τα σταθμά 1,
3, 9, 27, 81.

Ψυχαγωγικά Μαθηματικά 239


11.2 Μεταγγίσεις
I.
Α(8) Β(5) Γ(3)
8
3 5
3 2 3
6 2
6 2
1 5 2
1 4 3
4 4

II. Α Β(7) Γ(4)


7
3 4
3
3
7 3
6 4
5
2 4

11.3 Μάντεψε!
Γράφουμε τους αριθμούς 1 έως και 15 στο δυαδικό σύστημα.

1 2 3 4 5 6 7 8 9 10
1 10 11 100 101 110 111 1000 1001 1010
11 12 13 14 15
1011 1100 1001 1110 1111

Στην καρτέλα Α βάζουμε αυτούς που έχουν ψηφίο 1 στη θέση των μονάδων, στην καρτέλα Β βάζουμε αυτούς
που έχουν ψηφίο 1 στη θέση των δυάδων, στην καρτέλα Γ βάζουμε αυτούς που έχουν ψηφίο 1 στη θέση των
τετράδων και στην καρτέλα Δ αυτούς που έχουν ψηφίο 1 στη θέση των οκτάδων. Έτσι, για παράδειγμα, αν η
φίλη σού πει ότι ο αριθμός που σκέφτηκε είναι στη Β και τη Γ καρτέλα, εσύ γνωρίζεις αμέσως ότι έχει μια
δυάδα και μια τετράδα ακριβώς, δηλαδή είναι ο 6, και παρόμοια με τους άλλους αριθμούς. Το 15 είναι 42 –1,
και επομένως για να γράψουμε όλους τους αριθμούς ως εκεί, χρειάζεται να χρησιμοποιήσουμε μονάδες,
δυάδες, τετράδες και οκτάδες (το 8 είναι η μεγαλύτερη δύναμη του 2, μικρότερη από το 16). Για αριθμούς ως
και το 31, θα χρειαζόμασταν αντίστοιχα πέντε καρτέλες (η επόμενη δύναμη του 2 είναι το 32 = 25) και για
αριθμούς ως και τον 63, θα χρειαζόμασταν έξι καρτέλες, αφού η επόμενη δύναμη του 2 είναι το 64 = 26 κ.ο.κ.

11.4 Κρυπτάριθμοι
Προφανώς, Δ = 0 + 1 και Ο  8. Ας δοκιμάσουμε Ο = 8. Με λίγες δοκιμές βλέπουμε ότι μια λύση είναι η:

8 7 2 3

+ 9 4 8

9 6 7 1

240 Χατζηκυριάκου Κώστας


Θέλουμε να αντικαταστήσουμε τα γράμματα με κατάλληλα ψηφία έτσι ώστε ΔΥΟ επί ΔΥΟ να ισούται με
ΠΕΝΤΕ. Προφανώς (γιατί;), Δ  3 και εύκολα η περίπτωση Δ = 3 απορρίπτεται. Άρα, Δ = 1 ή 2.
Δοκιμάζουμε για Δ = 1. Τότε:

(102 + Υ·10 + Ο)·(102 + Υ·10 + Ο) =


104 + 2·Υ·103 + (Υ2 + 2·Ο)·102 + 2·Υ·Ο·10 +Ο2 =
Π·104 + Ε·103 + Ν·102 + Τ·10 +Ε.
Προφανώς, Υ  5 (γιατί;). Δοκιμάζουμε Υ = 5. Τότε, αρκεί Ο = 8. Άρα, ο αριθμός ΔΥΟ μπορεί να
είναι ο 158 και όντως:

1 5 8

× 1 5 8

1 2 6 4

7 9 0
+
1 5 8

2 4 9 6 4

Τέλος, θέλουμε να αντικαταστήσουμε τα γράμματα με αριθμούς έτσι ώστε ΛΙΓΟ + ΝΕΡΟ = ΝΕΦΟΣ.
Προφανώς (γιατί;), Ν = 1. Με δοκιμές βρίσκουμε ότι η πράξη μπορεί να είναι η:

9 4 7 6

+ 1 0 8 6

1 0 5 6 2

Ψυχαγωγικά Μαθηματικά 241


Βιβλιογραφία/Αναφορές

Averbach, B., & Chein O. (2000). Problem Solving Through Recreational Mathematics. Dover Publications
Inc.
Bolt, B. (1990). Μαθηματικές Σπαζοκεφαλιές (3 τόμοι). Εκδόσεις Κάτοπτρο.
Brooke, M. (1963). 150 Puzzles in Crypt-Arithmetic. Dover Publications Inc.
http://cryptarithms.awardspace.us/150-puzzles-in-crypt-arithmetic.pdf
Hunter, J. A. H., & Madachy, J. S. (1975). Mathematical Diversions. Dover Publications Inc.
Kraitchik, M. (1953). Mathematical Recreations. Dover Publications Inc.
Χατζηκυριάκου, K. (2017). Μαθηματικά για τη Δασκάλα και τον Δάσκαλο, Αριθμοί, Σύνολα, Σχήματα (2η
έκδοση). Εκδόσεις Σοφία.

242 Χατζηκυριάκου Κώστας


Κεφάλαιο 12
Παιχνίδια τύχης

Σύνοψη
Σύντομη εισαγωγή στις βασικές έννοιες της θεωρίας των πιθανοτήτων με τη μελέτη προβληματικών
καταστάσεων σε παιχνίδια τύχης.

Προαπαιτούμενη γνώση
Γραφή, ανάγνωση, αριθμητική, απλή συνδυαστική, απλοϊκή θεωρία συνόλων.

Ψυχαγωγικά Μαθηματικά 243


12.1 Ζαριές
I. Ρίχνω δύο κανονικά ζάρια. Με πόσους διαφορετικούς τρόπους μπορεί:
i. το άθροισμα των αριθμών που φέρνω να είναι 13;
ii. το άθροισμα των αριθμών που φέρνω να είναι αριθμός μεγαλύτερος του 1 και μικρότερος του
13;
iii. το άθροισμα των αριθμών που φέρνω να είναι μεγαλύτερο ή ίσο του 5 και μικρότερο ή ίσο του 9;

244 Χατζηκυριάκου Κώστας


II. Ποια είναι η πιθανότητα:
i. ο αριθμός στο ένα μόνο ζάρι να είναι περιττός;
ii. τουλάχιστον ένας από τους δύο αριθμούς που φέρνω να είναι περιττός;
iii. και οι δύο αριθμοί να είναι περιττοί;
iv. να φέρω εξάρες;
v. να με κερδίσει ο συμπαίκτης μου «στα ζάρια», όταν το άθροισμα των δύο αριθμών που έφερα
είναι δέκα;
vi. το άθροισμά των δύο αριθμών που έφερα να είναι άρτιο;
vii. οι αριθμοί που φέρνω να είναι περιττοί, αν το άθροισμά τους είναι άρτιο;

Ψυχαγωγικά Μαθηματικά 245


III. Ρίχνω τρία κανονικά ζάρια. Ποια είναι η πιθανότητα:
i. οι αριθμοί που έφερα να είναι διαδοχικοί;
ii. κανείς από τους αριθμούς να μην είναι «άσος»;

IV. Ένας παίκτης συμμετέχει σε ένα παιχνίδι «ζάρια». Οι κανόνες που συμφωνήθηκαν είναι οι εξής: Ο κάθε
παίκτης ρίχνει δύο ζάρια. Αν η ζαριά του έχει άθροισμα εφτά (7) ή ένδεκα (11), κερδίζει. Αν η ζαριά του
έχει άθροισμα 2 ή 3 ή 12, χάνει. Αν η ζαριά του είναι οποιοδήποτε άλλο άθροισμα, το άθροισμα αυτό
γίνεται ο τυχερός του αριθμός, και συνεχίζει να ρίχνει τα ζάρια έως ότου φέρει εφτά (7), οπότε χάνει, ή
φέρει τον τυχερό του αριθμό, οπότε κερδίζει.
i. Ποια είναι η πιθανότητα να κερδίσει ένας παίκτης;
ii. Πριν αρχίσει το παιχνίδι, κάθε παίκτης βάζει ένα στοίχημα. Αν χάσει, χάνει το ποσό που
στοιχημάτισε. Αν κερδίσει, κερδίζει χρήματα ανάλογα με τον τρόπο που κέρδισε ως εξής:
• Αν κερδίσει φέρνοντας 7 ή 11, κερδίζει το ποσό που στοιχημάτισε.

• Αν κερδίσει φέρνοντας τον τυχερό του αριθμό που είναι 6 ή 8, πάλι κερδίζει το ποσό που
στοιχημάτισε.

• Αν κερδίσει φέρνοντας τον τυχερό του αριθμό που είναι 4 ή 10, κερδίζει το διπλάσιο του
ποσού που στοιχημάτισε.

• Αν κερδίσει φέρνοντας τον τυχερό του αριθμό που είναι 5 ή 9, κερδίζει τα τρία δεύτερα του
ποσού που στοιχημάτισε.
Πόσα θα κερδίσει ή θα χάσει ο παίκτης κατά μέσο όρο ανά παιχνίδι, αν παίξει πολλά παιχνίδια
και το ποσό που στοιχηματίζει ανά παιχνίδι είναι 1 ευρώ;

246 Χατζηκυριάκου Κώστας


V. Ποια είναι η πιθανότητα να φέρω τουλάχιστον μία φορά εξάρες σε ν διαδοχικές ριξιές δύο ζαριών;

Ψυχαγωγικά Μαθηματικά 247


12.2 Χαρτοπαίγνια
I. Από μία τράπουλα (52 χαρτιά) τραβώ τυχαία 10 χαρτιά.
i. Ποια είναι η πιθανότητα να μην υπάρχει άσος ανάμεσά τους;
ii. Ποια είναι η πιθανότητα να υπάρχει τουλάχιστον ένας άσος;
iii. Ποια είναι η πιθανότητα να υπάρχει μόνο ένας άσος;
iv. Ποια είναι η πιθανότητα να υπάρχουν τουλάχιστον δύο άσοι;

248 Χατζηκυριάκου Κώστας


II. Στο χαρτοπαίγνιο πόκερ, λέγοντας χέρι εννοούμε οποιοδήποτε σύνολο 5 χαρτιών επιλεγμένων από την
καλά ανακατωμένη τράπουλα των 52 χαρτιών. [Η τράπουλα έχει δύο χρώματα και τέσσερα διαφορετικά
είδη: κούπα, καρό, σπαθί και μπαστούνι και τρεις φιγούρες: ρήγας, ντάμα, βαλές. Η σειρά των χαρτιών
είναι η συνηθισμένη αριθμητική διάταξη από το ένα (άσος) ως το δέκα, βαλές, ντάμα, ρήγας. Επιπλέον,
θεωρείται σειρά και η ακολουθία: δέκα, βαλές, ντάμα, ρήγας, άσος].
Πόσα διαφορετικά χέρια υπάρχουν;

Ποια είναι η πιθανότητα το χέρι που σου έρχεται στο μοίρασμα να αποτελείται από:

i. άσο, ρήγα, ντάμα, βαλέ και δέκα του ίδιου είδους (βασιλικό φλας);
ii. πέντε ομοειδή χαρτιά στη σειρά, που, όμως, δεν αποτελούν βασιλικό φλας (στρέιτ φλας);
iii. ακριβώς τέσσερις ίδιες φιγούρες ή αριθμούς (και κάποιο διαφορετικό χαρτί);
iv. ακριβώς τρεις ίδιες φιγούρες ή αριθμούς και ένα ζευγάρι ίδιων αριθμών;
v. πέντε ομοειδή χαρτιά, αλλά όχι στρέιτ φλας;
vi. πέντε χαρτιά στη σειρά, αλλά όχι όλα ομοειδή;
vii. ακριβώς τρεις ίδιες φιγούρες ή αριθμούς και δύο άλλα χαρτιά;
viii. δύο διαφορετικά ζευγάρια και ένα διαφορετικό χαρτί;
ix. ένα ακριβώς ζευγάρι με φιγούρες ή αριθμούς;
x. Να μην είναι τίποτε από τα παραπάνω;

Ψυχαγωγικά Μαθηματικά 249


12.3 Γενέθλια
Ποια είναι η πιθανότητα σε μία ομάδα 10 (αντίστοιχα 20, 23, 30, 57, 366) φοιτητριών, τουλάχιστον δύο να
έχουν γενέθλια την ίδια ημέρα; (Υποθέτουμε ότι κάθε χρόνος έχει 365 ημέρες και ότι όλες οι ημέρες είναι
εξίσου πιθανές).

250 Χατζηκυριάκου Κώστας


12.4 Από την ψυχαγωγία στην κριτική μαθηματική αγωγή
I. Στη γειτονιά Φ μιας παλιάς βιομηχανικής πόλης, όπου υπήρχε παλαιότερα εργοστάσιο χημικών,
κατοικούν το 4% των παιδιών της πόλης. Από αυτά, 16% αντιδρούν θετικά σε ένα τεστ που ανιχνεύει
τοξικά μέταλλα στο σώμα τους. Το αντίστοιχο ποσοστό για τα παιδιά που κατοικούν σε άλλες γειτονιές
είναι 2%.
i. Ποια είναι η πιθανότητα ότι τυχαία επιλεγμένο παιδί της πόλης ζει στη γειτονιά Φ και αντιδρά
αρνητικά στο τεστ;
ii. Ποια είναι η πιθανότητα ότι τυχαία επιλεγμένο παιδί της πόλης αντιδρά αρνητικά στο τεστ;
iii. Ποια είναι η πιθανότητα ότι τυχαία επιλεγμένο παιδί της πόλης που αντιδρά αρνητικά στο τεστ
κατοικεί στη γειτονιά Φ;

Ψυχαγωγικά Μαθηματικά 251


II. Το διαγνωστικό τεστ για την ασθένεια Α έχει τα εξής χαρακτηριστικά: Αν κάποιος άνθρωπος έχει την
ασθένεια Α, τότε η πιθανότητα να είναι θετικό το τεστ είναι 0,9. Αν δεν την έχει, η πιθανότητα να βγει
θετικό είναι 0,01. Γνωρίζουμε ότι η πιθανότητα να έχει κάποιος την ασθένεια είναι 0,01. Ο Γ. έκανε το
τεστ και βγήκε θετικό. Ποια είναι η πιθανότητα να έχει όντως την ασθένεια;

252 Χατζηκυριάκου Κώστας


Απαντήσεις στα ερωτήματα του 12ου Κεφαλαίου

12.1 Ζαριές
I.
i. 6 + 6 = 12  13. Άρα, με κανέναν τρόπο δεν μπορεί το άθροισμα των αριθμών που φέρνω να είναι
13, είναι μηδέν.
ii. Αφού το άθροισμα των αριθμών που φέρνω μπορεί να είναι το λιγότερο 2 = 1 + 1 και το
περισσότερο 12 = 6 + 6 , άρα το ζητούμενο άθροισμα μπορεί να επιτευχθεί και με τους 36
διαφορετικούς τρόπους που βλέπουμε στον παρακάτω πίνακα:

1 2 3 4 5 6

1 (1,1) (1,2) (1,3) (1,4) (1,5) (1,6)

2 (2,1) (2,2) (2,3) (2,4) (2,5) (2,6)

3 (3,1) (3,2) (3,3) (3,4) (3,5) (3,6)

4 (4,1) (4,2) (4,3) (4,4) (4,5) (4,6)

5 (5,1) (5,2) (5,3) (5,4) (5,5) (5,6)

6 (6,1) (6,2) (6,3) (6,4) (6,5) (6,6)

iii. Τα κοκκινισμένα ζευγάρια είναι τα επιθυμητά αποτελέσματα και είναι 24. Μπορούμε να πούμε
2
ότι τα επιθυμητά αποτελέσματα είναι τα του συνόλου των δυνατών αποτελεσμάτων.
3
Ας αναστοχαστούμε, κατά τη συμβουλή του Polya, τα ερωτήματα και την προβληματική κατάσταση
που είχαμε να αντιμετωπίσουμε. Είναι φανερό ότι το ρίξιμο των δύο ζαριών είναι ένα πείραμα τύχης.

Στο πείραμα αυτό, τα δυνατά αποτελέσματα συγκροτούνε το σύνολο που η συνήθης ονομασία του
είναι ο δειγματικός χώρος Ω του πειράματος. Τα αποτελέσματα που μας ενδιαφέρει να μετρήσουμε ανάμεσά
τους, ας τα πούμε τα για οποιονδήποτε λόγο ευνοϊκά αποτελέσματα, αποτελούν ένα υποσύνολο Ε του Ω.
Οποιοδήποτε υποσύνολο Ε του Ω ονομάζεται συμβάν ή γεγονός.

|Ε|
Είναι φανερό, λοιπόν, ότι για οποιοδήποτε συμβάν, το κλάσμα είναι κάποιος αριθμός στο
|Ω|
διάστημα [0,1] και ονομάζεται η πιθανότητα του συμβάντος p(E).

Στο ερώτημα i. E = ∅, οπότε p(E) = 0,


στο ερώτημα ii. E = Ω, οπότε p(E) = 1,
24 2
ενώ στο ερώτημα iii. P(E) = = .
36 3

Γνωρίζουμε ότι για δύο οποιαδήποτε σύνολα Α, Β ισχύει: |A ∪ B| = |A| + |B| − |A ∩ B|, άρα P(A ∪
B) = P(A) + P(B) − P(A ∩ B). Προφανώς, αν τα συμβάντα είναι ξένα μεταξύ τους, δηλαδή A ∩ B = ∅, τότε
P(A ∪ B) = P(A) + P(B).

Ψυχαγωγικά Μαθηματικά 253


Επίσης, αν Α ⊆ Ω, τότε ΑC = συμπλήρωμα του Α = Ω – Α = {x: x ∈ Ω και x ∉ A} και επομένως P(ΑC)
= 1−P(A).

II.
18 1
i. P (ένα μόνο ζάρι περιττό) = 36 = 2 .
27 3
ii. P (ένα τουλάχιστον ζάρι περιττό) = = 4 .
36
9 1
iii. P (και τα δύο ζάρια περιττά) = 36 = 4 .
1
iv. P ({(6,6)}) = 36
v. O αντίπαλος μπορεί να κερδίσει με τρεις διαφορετικούς τρόπους: Φέρνοντας (5,6), (6,5) ή (6,6),
3 1
οπότε η πιθανότητα να με κερδίσει είναι = 12.
36
18 1
vi. P (άθροισμα ζαριάς άρτιο) = 36 = 2.

Το ερώτημα αυτό παρουσιάζει ιδιαίτερο ενδιαφέρον. Στο ερώτημα iii, έχουμε υπολογίσει την
9 1
πιθανότητα να είναι και τα δύο ζάρια περιττά (ισούται με = ). Ωστόσο, τώρα γνωρίζουμε κάτι
36 4
παραπάνω, ότι το άθροισμα των δύο αριθμών που φέραμε είναι άρτιος αριθμός (αυτό μπορεί να γίνει με
1
πιθανότητα ίση με ). Τα υπόλοιπα αποτελέσματα, προφανώς, δεν μας ενδιαφέρουν. Με άλλα λόγια, ο
2
δειγματικός χώρος έχει αλλάξει, είναι ένα υποσύνολο του αρχικού και είναι το συμβάν που αποτελείται από
τις ζαριές με άθροισμα άρτιο). Είναι φανερό ότι η πιθανότητα που αναζητάμε δίνεται τώρα από το κλάσμα
9 1
=2.
18

Αν αναστοχαστούμε, πάντα κατά τη συμβουλή του Polya, την κατάσταση που αντιμετωπίσαμε,
μπορούμε να αντιληφθούμε ότι έχουμε φτάσει σε μια νέα έννοια, στην πιθανότητα υπό συνθήκη. Πράγματι,
αναζητούμε την πιθανότητα του συμβάντος Α (εδώ: «και τα δύο ζάρια περιττά») υπό τη συνθήκη Β (εδώ:
«άθροισμα ζαριών άρτιο»).

|A∩B|
Είναι εύλογο να ορίσουμε την πιθανότητα να συμβεί το Α εάν έχει συμβεί το Β ως P(A|B) =
|B|

Σχήμα 12.1

254 Χατζηκυριάκου Κώστας


P(A∩B) |A∩B|
= , με P(B) ≠ 0 (όντως, το Β έχει συμβεί για να έχει νόημα η συνθήκη). Αντίστοιχα, P(Β|Α) =
P(B) |Α|
P(A∩B)
= , με P(Α) ≠ 0 (όντως, το Α έχει συμβεί για να έχει νόημα η συνθήκη).
P(Α)

Παρατηρούμε ότι αν P(Α) ≠ 0 και P(B) ≠ 0, τότε P(A ∩ B) = P(B)∙P(A|B) = P(A)∙P(B|A). Επομένως,
P(A∩B) P(A)∙P(B|A) P(A∩B) P(B)∙P(A|B)
μπορούμε να γράψουμε P(A|B) = = και P(Β|Α) = = .
P(B) P(B) P(Α) P(Α)

Εάν συμβεί P(A|B) = P(Α), τότε λέμε ότι το συμβάν Α είναι ανεξάρτητο από το συμβάν Β (το ότι
γνωρίζουμε ότι συνέβη το Β δεν αλλάζει την πιθανότητα να συμβεί το Α). Ωστόσο, τότε ισχύει και P(B)∙P(A)
= P(A)∙P(B|A) και επομένως ισχύει και P(B) = P(B|A), οπότε και το συμβάν Β είναι ανεξάρτητο από το
συμβάν Α. Άρα, μπορούμε να πούμε ότι αρκεί να ισχύει μία από τις παραπάνω σχέσεις για να είναι τα
συμβάντα Α, Β ανεξάρτητα μεταξύ τους. Παρατήρησε ότι συμβάντα (μη κενά) ξένα μεταξύ τους είναι κατ’
ανάγκη ανεξάρτητα μεταξύ τους. Είναι φανερό ότι το αντίστροφο δεν ισχύει (Γιατί;).

Τέλος, οι παραπάνω σχέσεις μας οδηγούν στο θεώρημα του Bayes που μας βοηθά να υπολογίσουμε
τις πιθανότητες P(Ai |B), όπου i = 1, 2 και A1 ∪ A2 = Ω και A1 ∩ A2 = ∅ (δηλαδή A1, A2 είναι μια διαμέριση
του δειγματικού χώρου Ω) και Β ⊆ Ω.
P(Αi ∩B) P(Αi )∙P(B|Αi ) P(Αi )∙P(B|Αi ) P(Αi )∙P(B|Αi )
P (Ai |B) = = = = , i = 1,2.
P(B) P(B) P(Α1 ∩B)+ P(Α2 ∩B) P(Α1 )∙P(B|Α1 )+ P(Α2 )∙P(B|Α2 )

A1 A2

Σχήμα 12.2

III.
i. Οι τρεις διαδοχικοί αριθμοί 1, 2, 3 μπορούν να «έρθουν» με έξι διαφορετικούς τρόπους και το
ίδιο ισχύει για τους τρεις διαδοχικούς 2, 3, 4, τους τρεις διαδοχικούς 3, 4, 5 και τους τρεις
διαδοχικούς 4, 5, 6. Άρα, οι ευνοϊκές περιπτώσεις είναι 24. Ο δειγματικός χώρος, φυσικά, έχει
24 1
πληθικό αριθμό 6∙6∙6 = 216. Άρα, η ζητούμενη πιθανότητα είναι =9.
216
ii. Αφού δεν θέλουμε κανείς από τους αριθμούς να είναι άσος, οι επιλογές που έχουμε για κάθε
ζάρι δεν είναι έξι αλλά πέντε, οι αριθμοί 2, 3, 4, 5, 6. Επομένως, οι εν συνόλω ευνοϊκές
περιπτώσεις είναι 5∙5∙5, ενώ ο δειγματικός χώρος έχει πληθικό αριθμό 6∙6∙6. Επομένως, η
5∙5∙5 125
πιθανότητα να μη φέρουμε κανέναν άσο ρίχνοντας τρία ζάρια είναι 6∙6∙6 = 216.
Αξίζει να υπολογίσουμε την πιθανότητα να φέρουμε τουλάχιστον έναν άσο, με δύο
διαφορετικούς τρόπους. Προφανώς, η πιθανότητα να φέρουμε τουλάχιστον έναν άσο είναι 1–
125 91
= 216 , αφού τα συμβάντα «φέρνω τουλάχιστον έναν άσο» και «δεν φέρνω κανένα άσο»
216
είναι συμπληρωματικά. Μπορούμε, ωστόσο, την πιθανότητα του να φέρουμε τουλάχιστον έναν

Ψυχαγωγικά Μαθηματικά 255


25
άσο να την υπολογίσουμε κατευθείαν ως εξής: η πιθανότητα να φέρουμε έναν άσο είναι 3∙216 (ο
άσος μπορεί να είναι στο πρώτο ή στο δεύτερο ή στο τρίτο ζάρι), η πιθανότητα να φέρουμε δύο
5
άσους είναι 3∙216 (οι άσοι στο 1ο και το 2ο ζάρι ή στο 2ο και το 3ο ή στο 1ο και στο 3ο) και η
1
πιθανότητα να φέρουμε τρεις άσους είναι . Τα συμβάντα αυτά είναι ξένα μεταξύ τους και η
216
ένωσή τους είναι το συμβάν του οποίου την πιθανότητα ζητάμε. Άρα, η πιθανότητα αυτή είναι
25 5 1 91
3∙216 + 3∙216 + 216 = 216 .

IV.
i. Ποια είναι τα νικηφόρα συμβάντα για έναν παίκτη;
Σ1 = «άθροισμα ζαριάς 7 ή 11»,

Σ2 = «άθροισμα ζαριάς 4 και νέο άθροισμα ζαριάς 4 χωρίς να έχει ρίξει ζαριά με άθροισμα 7»,

Σ3 = «άθροισμα ζαριάς 5 και νέο άθροισμα ζαριάς 5 χωρίς να έχει ρίξει ζαριά με άθροισμα 7»,

Σ4 = «άθροισμα ζαριάς 6 και νέο άθροισμα ζαριάς 6 χωρίς να έχει ρίξει ζαριά με άθροισμα 7»,

Σ5 = «άθροισμα ζαριάς 8 και νέο άθροισμα ζαριάς 8 χωρίς να έχει ρίξει ζαριά με άθροισμα 7»,

Σ6 = «άθροισμα ζαριάς 9 και νέο άθροισμα ζαριάς 9 χωρίς να έχει ρίξει ζαριά με άθροισμα 7»,

Σ7 = «άθροισμα ζαριάς 10 και νέο άθροισμα ζαριάς 10 χωρίς να έχει ρίξει ζαριά με άθροισμα
7».

Είναι φανερό ότι τα συμβάντα αυτά είναι ξένα μεταξύ τους και επομένως η πιθανότητα νίκης του
παίκτη P(N) ισούται με ∑71 P(Σi ). Ας υπολογίσουμε τις πιθανότητες P(Σi ), i = 1, 2,…,7.
6 2 8 2
Προφανώς, P(Σ1 ) = P(άθροισμα ζαριάς 7) + P(άθροισμα ζαριάς 11) = + = = .
36 36 36 9

1 1 1
Είναι εξάλλου φανερό πως P(Σ2) = P(Σ(4)∙P(Σ(4)|Σ(4 ή7)) = 12 ∙ 3 = , αφού το συμβάν Σ2 είναι η
36
τομή δύο προφανώς ανεξάρτητων συμβάντων, του συμβάντος «το άθροισμα της ζαριάς είναι 4» και του
συμβάντος «το άθροισμα της ζαριάς είναι 4 προτού το άθροισμα της ζαριάς είναι 7».
3 1 3 1
Αλλά P(Σ(4)) = = 12 και P(Σ(4)|Σ(4 ή7)) = = , αφού Σ(4 ή 7) = Σ(4) ∪ Σ(7) = {(1,3),
36 9 3
(3,1), (2,2)} ∪ {(1,6), (6,1), (2,5), (5,2), (3,4), (4,3)}.

4 4 2
Με ανάλογο τρόπο, P(Σ3) = P(Σ(5)∙P(Σ(5)|Σ(5 ή7)) = ∙ = .
36 10 45

5 5 25
P(Σ4) = P(Σ(6)∙P(Σ(6)|Σ(6 ή7)) = 36 ∙ 11 = .
396

5 5 25
P(Σ5) = P(Σ(8)∙P(Σ(8)|Σ(8 ή7)) = 36 ∙ 11 = .
396

4 4 2
P(Σ6) = P(Σ(9)∙P(Σ(9)|Σ(9 ή7)) = 36 ∙ 10 = .
45

256 Χατζηκυριάκου Κώστας


1 1 1
P(Σ7) = P(Σ(10)∙P(Σ(10)|Σ(10 ή7)) = 12 ∙ 3 = .
36
2 2 50 4 244
Επομένως, P(N) = ∑71 P(Σi ) = + + + = .
9 36 396 45 495

ii. Μόλις είδαμε ότι:


2
η πιθανότητα να κερδίσει 1 ευρώ με άθροισμα ζαριάς 7 ή 11 είναι ,
9
2
η πιθανότητα να κερδίσει 2 ευρώ με άθροισμα ζαριάς 4 ή 10 είναι ,
36
3 4
η πιθανότητα να κερδίσει ευρώ με άθροισμα ζαριάς 5 ή 9 είναι ,
2 45
50
η πιθανότητα να κερδίσει 1 ευρώ με άθροισμα ζαριάς 6 ή 8 είναι ,
396

244 251
η πιθανότητα να χάσει 1 ευρώ είναι 1 − 495 = 495 .

Επομένως, κατά μέσο όρο, μακροπρόθεσμα, κερδίζει (άθροισμα θετικό) ανά παιχνίδι
2 2 50 3 4 251 85
1∙ + 2 ∙ 36 + 1∙ 396
+2∙ – 1 ∙ 495 = 990 , δηλαδή περίπου 8,6 σέντσια για κάθε ευρώ που
9 45
στοιχηματίζει.

Εν γένει, έστω ένα παιχνίδι τύχης του οποίου ο δειγματικός χώρος διαμερίζεται στα (ξένα μεταξύ τους)
συμβάντα Σi , i = 1, 2, …,ν, η πιθανότητα των οποίων να συμβούν είναι (αντίστοιχα) pi , i = 1, 2, …, ν, (∑ν1 pi
= 1), και οι αποδόσεις τους (αντίστοιχα) mi , i = 1, 2, …, ν (δηλαδή αν το Σi συμβαίνει, ο παίκτης κερδίζει mi
αν mi είναι θετικός αριθμός, χάνει mi αν ο mi είναι αρνητικός αριθμός, ούτε χάνει ούτε κερδίζει αν ο mi είναι
μηδέν). Tότε, η προσδοκώμενη τιμή του παιχνιδιού είναι ο αριθμός π = ∑ν1 mi ∙ pi .

V. H επικρατούσα άποψη για την ανάδυση του μαθηματικού κλάδου της θεωρίας των πιθανοτήτων είναι ότι
αυτή ξεκινά με την αλληλογραφία του Pascal (1623-1622) με τον Fermat (1601-1655) και αφορά
ερωτήματα γύρω από παιχνίδια τύχης που έθεσε o Antoine Gombaud, ιππότης de Meré, στον Pascal. Ένα
από αυτά είναι το ερώτημα που έχουμε να απαντήσουμε εδώ:
35
Κάθε φορά που ρίχνω τα ζάρια, η πιθανότητα να μη φέρω εξάρες είναι φυσικά (αφού η πιθανότητα
36
1
να ρίξω εξάρες είναι όπως είδαμε . Αν ρίξουμε τα ζάρια ν φορές, ο πληθικός αριθμός του δειγματικού
36
χώρου είναι προφανώς 36ν, ενώ οι μη ευνοϊκές περιπτώσεις 35ν, οτιδήποτε εκτός από εξάρες δηλαδή,
35
επομένως η πιθανότητα να φέρω τουλάχιστον μία φορά εξάρες είναι 1 − ( )𝜈 . Αν θέλω ο αριθμός αυτός να
36
log 2
είναι 0,5, τότε παίρνοντας λογαρίθμους βρίσκω: ν = = 24, 605… . Με άλλα λόγια, η θεωρία
log 36−log 35
των πιθανοτήτων δεν … θα με συμβούλευε να στοιχηματίσω ότι θα φέρω εξάρες σε λιγότερες από 24 ριξιές!

Ψυχαγωγικά Μαθηματικά 257


12.2 Χαρτοπαίγνια

I. O πληθικός αριθμός του δειγματικού χώρου ισούται με 52Σ10.


i. Οι άσοι είναι 4, άρα πρέπει η δεκάδα μου να προέρχεται από τα υπόλοιπα 48 χαρτιά, άρα ο
πληθικός αριθμός των ευνοϊκών περιπτώσεων είναι 48Σ10. Επομένως, η πιθανότητα να μην
48∙…∙39 241
υπάρχει κανένας άσος στη δεκάδα είναι = ≅ 0,41.
52∙…∙43 595
241 354
ii. Η πιθανότητα να υπάρχει τουλάχιστον ένας άσος ισούται με 1– = ≅ 0,59.
595 595
4Σ1 ∙ 48Σ9 6708426560
iii. Η πιθανότητα να υπάρχει μόνο ένα άσος ισούται με = 15820024220 ≅ 0,42.
52Σ10

iv. Η πιθανότητα να υπάρχουν τουλάχιστον δύο άσοι ισούται, προφανώς, περίπου με 1– 0,42 – 0,42
= 0,17 (Παρατήρησε ότι, όπως είναι αναμενόμενο, 0,42 + 0,17 = 0,59).

II. Προφανώς, υπάρχουν 52Σ5 = 2589960 διαφορετικά χέρια.


Υπάρχουν:

4Σ1 = 4 χέρια που αποτελούνται από άσο, ρήγα, ντάμα, βαλέ και δέκα του ιδίου είδους (βασιλικό φλας),
36 = 4∙9 χέρια που αποτελούνται από πέντε ομοειδή χαρτιά στη σειρά που, όμως, δεν αποτελούν
βασιλικό φλας (στρέιτ φλας),

624 = 13Σ1∙ 12Σ1∙ 4Σ1 χέρια που αποτελούνται από ακριβώς τέσσερις ίδιες φιγούρες ή ίδιους αριθμούς
και κάποιο διαφορετικό, από αυτά, χαρτί (τέσσερα ίδια),

3744 = 13Σ1∙ 4Σ3∙ 12Σ1∙ 4Σ2 χέρια που αποτελούνται από ακριβώς τρεις ίδιες φιγούρες ή ίδιους αριθμούς
και από δύο ίδιες φιγούρες ή αριθμούς (φουλ χάουζ),

5108 = 13Σ5∙ 4Σ1– 4∙10 χέρια που αποτελούνται από πέντε ομοειδή χαρτιά, αλλά όχι βασιλικό ούτε
στρέιτ φλας,

10200 = 10Σ1∙ 45 – 4∙10 χέρια που αποτελούνται από πέντε χαρτιά στη σειρά, όχι όλα ομοειδή, αλλά
ούτε βασιλικό ούτε στρέιτ φλας,

54912 = 13Σ1∙ 4Σ3∙ 12Σ2∙ 42 χέρια που αποτελούνται από ακριβώς τρεις ίδιες φιγούρες ή τρεις ίδιους
αριθμούς και δύο άλλα χαρτιά (τρία ίδια),

123552 = 13Σ2∙ 4Σ2∙ 4Σ2∙ 11Σ1∙ 4 χέρια που αποτελούνται από δύο διαφορετικά ζευγάρια και ένα
διαφορετικό χαρτί (δύο ζευγάρια),

1098240 = 13Σ1∙ 4Σ2∙ 12Σ3∙ 4 χέρια που αποτελούνται από ακριβώς ένα ζευγάρι με φιγούρες ή αριθμούς,

1302540 = (13Σ5 – 10) ∙ (45– 4) χέρια που δεν είναι τίποτα από τα παραπάνω.

Βρίσκουμε την πιθανότητα να μας μοιραστεί ένα είδος χεριού από τα παραπάνω, διαιρώντας το πλήθος
τέτοιων χεριών με τον αριθμό όλων των χεριών.

258 Χατζηκυριάκου Κώστας


`12.3 Γενέθλια
Είναι φανερό ότι αν οι φοιτήτριες είναι 366, τότε η πιθανότητα δύο να έχουν γενέθλια την ίδια μέρα είναι 1.
Σε όλες τις άλλες περιπτώσεις μπορούμε να υπολογίσουμε τη ζητούμενη πιθανότητα, υπολογίζοντας πρώτα
την πιθανότητα όλες οι φοιτήτριες να έχουν γενέθλια σε διαφορετικές μέρες. Οπότε, αν ν είναι ν, η
365!
πιθανότητα να έχουν δύο τουλάχιστον από αυτές γενέθλια την ίδια μέρα είναι 1–365𝜈∙(365−𝜈)! .

Για ν = 10, 20, 23, 30, 57, οι αντίστοιχες πιθανότητες είναι περίπου 0,117, 0,411, 0,507, 0,706, 0,990.
Παρατηρούμε ότι αν οι φοιτήτριες είναι πάνω από 22, τότε η πιθανότητα τουλάχιστον δύο από αυτές να έχουν
γενέθλια την ίδια μέρα είναι πάνω από 0,5.

12.4 Από την ψυχαγωγία στην κριτική μαθηματική αγωγή


Η θεωρία των πιθανοτήτων πρωτοαναπτύχθηκε γύρω από τα παιχνίδια τύχης. Στη συνέχεια, εφαρμόστηκε
στη φυσική. Ωστόσο, εδώ και αρκετές δεκαετίες, πολλές από τις πληροφορίες στη βάση των οποίων οι
άνθρωποι παίρνουν αποφάσεις για προσωπικά και κοινωνικά θέματα είναι πιθανοκρατούμενες. Έτσι, η χρήση
και η κατανόηση εννοιών και μεθόδων της θεωρίας πιθανοτήτων έχει γίνει ζητούμενο βασικό εφόδιο ενός
κριτικά σκεπτόμενου πολίτη. Τα δύο ερωτήματά μας αφορούν παραδειγματικές καταστάσεις όπου η γνώση
της θεωρίας των πιθανοτήτων είναι απαραίτητη στην κατανόηση της κατάστασης και τη λήψη αποφάσεων για
σωστή δράση.

Δραστηριότητα: Γράψε σύντομα βιογραφικά για τον Blaise Pascal και τον Pierre Fermat,
ανατρέχοντας σε έγκυρο ιστότοπο, π.χ.: https://mathshistory.st-andrews.ac.uk/

I. Έστω Φ το γεγονός «παιδί ζει στη γειτονιά Φ». Τότε, P(Φ) = 0,04 και P(Φ) = 0,96. Γνωρίζουμε ότι P(+|Φ)
= 0,16, επομένως P(–|Φ) = 0,84, ενώ P(+|ΦC) = 0,02 και επομένως P(–|ΦC) = 0,98.
Η πρώτη ερώτηση μας ζητά να υπολογίσουμε την πιθανότητα P(Φ ∩ –), ότι τυχαία επιλεγμένο παιδί
της πόλης να ζει στη γειτονιά Φ και να αντιδρά αρνητικά στο τεστ. Γνωρίζουμε ότι P(Φ ∩ –) = P(–|Φ)∙P(Φ).
Επομένως, P(Φ ∩ –) = 0,84∙0,04 = 0,0336. Στη δεύτερη ερώτηση θα χρειαστούμε και την P(ΦC ∩ –) =
0,98∙0,96 = 0,9408.
Η δεύτερη ερώτηση μας ζητά να υπολογίσουμε την πιθανότητα P(–). Προφανώς, P(–) = P(Φ ∩ –) +
P(ΦC ∩ –) = 0,0336 + 0,9408 = 0,9744.
P(Φ ∩ –) 0,0336
Η τρίτη ερώτηση μας ζητά να υπολογίσουμε την πιθανότητα P(Φ|–) = = ≅ 0,0344.
P(–) 0,9744
Σημείωση. Υπάρχει αρκετά εκτεταμένη συζήτηση για το εάν ο μη ειδικός πολίτης χρειάζεται να
χρησιμοποιήσει σχετικά λεπτές μεθόδους της θεωρίας των πιθανοτήτων ή να είναι ικανός απλώς να
μεταφράζει την πιθανοκρατούμενη πληροφορία σε πληροφορία για φυσικές συχνότητες, οπότε η όποια
ζητούμενη λύση απαιτεί μόνο την κατανόηση των κλασματικών αριθμών. Έτσι στο ερώτημα αυτό, θα
μπορούσαμε να είχαμε εργαστεί ως εξής: Ας είναι ο παιδικός πληθυσμός της πόλης 10.000. Τότε, στη
γειτονιά Φ ζουν 400 παιδιά και έξω από αυτήν 9.600. Από τα 400 παιδιά της γειτονιάς Φ, θετικά αντιδρούν τα
64. Αρνητικά, τα υπόλοιπα 336. Από τα 9.600 παιδιά που ζουν έξω από τη γειτονιά Φ, θετικά αντιδρούν 192
και αρνητικά τα υπόλοιπα 9.408.

10.000

9.600
400


+ – +

9.408
64 336 192

Είναι φανερό ότι μπορούμε να διαβάσουμε όλες τις απαντήσεις στον παραπάνω πίνακα.

Ψυχαγωγικά Μαθηματικά 259


ΙΙ. Ας εφαρμόσουμε τα δεδομένα σε έναν πληθυσμό 10.000 ατόμων. Στον πληθυσμό αυτό, γνωρίζουμε ότι οι
ασθενείς είναι 100. Οι 90 από αυτούς, αν κάνουν το τεστ, θα βγουν θετικοί, ενώ 10, αν και ασθενείς, θα έχουν
αρνητική διάγνωση. Από την άλλη, 9.900 άτομα δεν ασθενούν. Ωστόσο, 99 από αυτούς, αν κάνουν το τεστ,
θα έχουν θετική διάγνωση και 9.801 θα έχουν αρνητική. Ας καταρτίσουμε τον σχετικό πίνακα.
Επομένως, συνολικά έχουν θετική διάγνωση 90 + 99 = 189 άτομα, από τα οποία όντως ασθενούν 90.
90 10
Άρα, η πιθανότητα ο Γ. να έχει την ασθένεια είναι = , δηλαδή λίγο λιγότερο από 50%.
189 21

10.000

100 9.900

+ – + –

90 10 99 9.801

260 Χατζηκυριάκου Κώστας


Βιβλιογραφία/Αναφορές

Averbach, B., & Chein O. (2000). Problem Solving Through Recreational Mathematics. Dover Publications
Inc.
Gigerenzer, G. (2002). Reckoning With Risk, Learning to Live With Uncertainty. Penguin Books.
Goodman, A. W., & Ratti, J. S. (1975). Finite Mathematics with Applications (2nd edition). Macmillan
Publishing Co., Inc.
Hunter, J. A. H., & Madachy, J. S. (1975). Mathematical Diversions. Dover Publications Inc.
Kraitchik, M. (1953). Mathematical Recreations. Dover Publications Inc.
Polya, G. (1998). Πώς να το λύσω. Εκδόσεις Καρδαμίτσα.
Χατζηκυριάκου, K. (2017). Μαθηματικά για τη Δασκάλα και τον Δάσκαλο, Αριθμοί, Σύνολα, Σχήματα (2η
έκδοση). Εκδόσεις Σοφία.

Ψυχαγωγικά Μαθηματικά 261


Κύριος διδακτικός στόχος του συγγράμματος είναι να δείξει πώς ορισμένες σπαζοκεφαλιές ή ορισμένα
προβλήματα παιγνιώδους χαρακτήρα που έχουν απλή διατύπωση όντως οδήγησαν ή μπορούν να οδηγήσουν
σε όμορφες, ενδιαφέρουσες, βαθιές και χρήσιμες μαθηματικές θεωρίες, όπως είναι, παραδείγματος χάριν, η
θεωρία γραφημάτων, η θεωρία των πεπερασμένων ομάδων, η συνδυαστική, η θεωρία των πιθανοτήτων και η
θεωρία των αριθμών.
Ειδικότερα, στα τρία πρώτα κεφάλαια παρουσιάζονται βασικά είδη γραφημάτων και μελετώνται οι ιδιότητές
τους. Στο τέταρτο κεφάλαιο μελετώνται οι ιδιότητες των δένδρων και η χρήση τους σε πεδία πέραν των
Μαθηματικών, όπως λόγου χάριν η Χημεία. Στο πέμπτο κεφάλαιο μελετώνται τα επίπεδα γραφήματα και ο
τύπος του Euler γι’ αυτά και για τα κυρτά πολύεδρα. Τέλος, στο έκτο κεφάλαιο, ολοκληρώνεται η θεωρία των
γραφημάτων με τη μελέτη του κλασικού προβλήματος του χρωματισμού του χάρτη. Στο έβδομο κεφάλαιο,
μελετώνται τα μαγικά τετράγωνα περιττής και άρτιας τάξης, ενώ στο όγδοο κεφάλαιο μελετώνται τα λατινικά
τετράγωνα και η σχέση τους με τα μαγικά τετράγωνα. Στο ένατο κεφάλαιο, μελετάται η δομή ομάδας των
συμμετριών του τετραγώνου και του ισόπλευρου τριγώνου και στη συνέχεια η σχέση αυτών των δύο δομών
με τις δομές συγγένειας δύο φυλών που έχουν μελετήσει ανθρωπολόγοι. Στο δέκατο κεφάλαιο, συνεχίζεται η
μελέτη της δομής της ομάδας. Στο ενδέκατο κεφάλαιο, Νούμερα, μελετώνται ορισμένες κλασικές αριθμητικές
σπαζοκεφαλιές, ενώ στο δωδέκατο κεφάλαιο, παρουσιάζονται τα βασικά στοιχεία της θεωρίας των
πιθανοτήτων. Το κεφάλαιο τελειώνει με μια σύντομη επίσκεψη στην κριτική μαθηματική εκπαίδευση.
Κάθε κεφάλαιο ξεκινά με φύλλα εργασίας. Η αντίστοιχη προς μελέτη μαθηματική θεωρία αναπτύσσεται
μέσω της επίλυσης των σπαζοκεφαλιών και των προβλημάτων που αυτά περιέχουν.

Το παρόν σύγγραμμα δημιουργήθηκε στο πλαίσιο του Έργου ΚΑΛΛΙΠΟΣ+


Χρηματοδότης Υπουργείο Παιδείας και Θρησκευμάτων,
Προγράμματα ΠΔΕ, ΕΠΑ 2020-2025
Φορέας υλοποίησης ΕΛΚΕ ΕΜΠ
Φορέας λειτουργίας ΣΕΑΒ/Παράρτημα ΕΜΠ/Μονάδα Εκδόσεων
Διάρκεια 2ης Φάσης 2020-2023
Σκοπός Η δημιουργία ακαδημαϊκών ψηφιακών συγγραμμάτων
ανοικτής πρόσβασης (περισσότερων από 700)
• Προπτυχιακών και μεταπτυχιακών εγχειριδίων
• Μονογραφιών
• Μεταφράσεων ανοικτών textbooks
• Βιβλιογραφικών Οδηγών
Επιστημονικά Υπεύθυνος Νικόλαος Μήτρου, Καθηγητής ΣΗΜΜΥ ΕΜΠ

ISBN: 978-618-228-126-0 DOI: http://dx.doi.org/10.57713/kallipos-361

Το παρόν σύγγραμμα χρηματοδοτήθηκε από το Πρόγραμμα Δημοσίων Επενδύσεων του Υπουργείου Παιδείας .

Ψυχαγωγικά Μαθηματικά 229

You might also like